You are on page 1of 114

3

INDEX
02-08-2021 .................................................................................................................................................................. 4

03-08-2021 .................................................................................................................................................................. 8

04-08-2021 ................................................................................................................................................................. 13

05-08-2021 ................................................................................................................................................................. 17

06-08-2021 ................................................................................................................................................................ 22

07-08-2021 .................................................................................................................................................................27

09-08-2021 ................................................................................................................................................................ 32

10-08-2021 ................................................................................................................................................................ 36

11-08-2021 ................................................................................................................................................................. 39

12-08-2021 .................................................................................................................................................................. 41

13-08-2021 ................................................................................................................................................................. 45

14-08-2021 ................................................................................................................................................................. 50

16-08-2021 ................................................................................................................................................................. 54

17-08-2021 ................................................................................................................................................................. 58

18-08-2021................................................................................................................................................................. 63

19-08-2021 ..................................................................................................................................................................67

19-08-2021 (IPCC Special).........................................................................................................................................72

20-08-2021 ................................................................................................................................................................ 78

21-08-2021 ................................................................................................................................................................. 83

23-08-2021 ................................................................................................................................................................ 85

24-08-2021 ................................................................................................................................................................ 89

25-08-2021 ................................................................................................................................................................ 94

26-08-2021 ................................................................................................................................................................ 96

27-08-2021 ................................................................................................................................................................ 98

28-08-2021 ............................................................................................................................................................... 101

30-08-2021 ...............................................................................................................................................................106

31-08-2021 ................................................................................................................................................................ 110

www.shankariasacademy.com | www.iasparliament.com
4

02-08-2021 b. 1 and 2 only


1) Consider the following statements with respect to e- c. 1 and 3 only
RUPI d. 2 and 3 only
1. It is a cashless and contactless digital
payments medium, which will be delivered to
mobile phones of beneficiaries in form of an 4) Consider the following statements with respect to
SMS-string or a QR code. Central Bank Digital Currency
2. It has been developed by the National 1. It is a form of fiat currency that is backed by
Payments Corporation of India in the central bank and does not possess any
collaboration with Ministry of Health and intrinsic value.
Family Welfare and National Health Authority. 2. India is the first country in the world to launch
Which of the statement(s) given above is/are correct? the Central Bank Digital Currency.
a. 1 only Which of the statement(s) given above is/are correct?
b. 2 only a. 1 only
c. Both 1 and 2 b. 2 only
d. Neither 1 nor 2 c. Both 1 and 2
d. Neither 1 nor 2
2) Consider the following statements with respect to
Controller General of Accounts (CGA) 5) Amagarh Fort, sometimes seen in the news recently,
1. The Office of Controller General of Accounts is located in?
derives mandate from Article 150 of the Indian a. Jaipur, Rajasthan
Constitution.
b. Mysuru, Karnataka
2. It works under the Department of
Expenditure, Ministry of Finance. c. Hyderabad, Telangana
Which of the statement(s) given above is/are correct? d. Gwalior, Madhya Pradesh
a. 1 only
b. 2 only 6) Which of the following statement(s) is/ are correct
with respect to All India Quota (AIQ) Scheme
c. Both 1 and 2
1. It comprises 20% of UG seats in government
d. Neither 1 nor 2 engineering colleges and 35% of UG seats in
government medical colleges.
3) Consider the following statements with respect to 2. In Abhay Nath v University of Delhi and
Kuthiran Tunnel Others, the SC directed that reservation of 15%
for Scheduled Castes and 7.5% for Scheduled
1. It is the first-ever road tunnel in Kerala which Tribes be introduced with in the AIQ.
improves connectivity in North – South
Corridor. Select the correct answer using the codes given below:
2. The tunnel runs through the Wayanad wildlife a. 1 only
sanctuary. b. 2 only
3. Kuthiran gradient is situated in the western c. Both 1 and 2
part of Rajamala Hills.
d. Neither 1 nor 2
Which of the statement(s) given above is/are correct?
a. 1 only

www.shankariasacademy.com | www.iasparliament.com
5

7) Consider the following statements with respective 10) Consider the following statements with respective
to Airports Economic Regulatory Authority of India to Sujal-Drink from Tap Mission
(AERA) Amendment Bill 2021
1. It is launched by Rajasthan government to
1. It proposes to amend the definition of major provide drink water to all urban households.
airport to include a group of airports.
2. The initiative has been launched under the
2. It empowers central government to designate Ministry of Jalshakthi.
any airport as a major airport by a notification.
Which of the statement(s) given above is/are correct?
Which of the statement(s) given above is/are correct?
a. 1 only
a. 1 only
b. 2 only
b. 2 only
c. Both 1 and 2
c. Both 1 and 2
d. Neither 1 nor 2
d. Neither 1 nor 2

Answers
8) Consider the following statements with respective
to Factoring Regulation (Amendment) Bill 2021 1. c

1. It has incorporated factoring business  Prime Minister Shri Narendra Modi will
suggestions from UK Sinha Committee. launch e-RUPI, a person and purpose specific
digital payment solution today.
2. It empowers RBI to allow non-bank finance
companies to remain in factoring business only e-RUPI
if it was their principal business.  It is a cashless and contactless instrument for
Which of the statement(s) given above digital payment.
is/are incorrect?
 It has been developed by National Payments
a. 1 only Corporation of India on its UPI platform, in
collaboration with the Department of Financial
b. 2 only Services, Ministry of Health & Family Welfare
c. Both 1 and 2 and National Health Authority.
d. Neither 1 nor 2 Working
 It is a QR code or SMS string-based e-Voucher,
which is delivered to the mobile of the
9) Which of the following statement(s) is/ are correct
beneficiaries.
with respect to I-MESA Scheme
 The users of this seamless one-time payment
mechanism will be able to redeem the voucher
1. It aims to conduct Social Audits for all the
without a card, digital payments app or
schemes of the Department starting FY 2021-
internet banking access, at the service
22.
provider.
2. It is formulated by Ministry of Social Justice
and Empowerment.  e-RUPI connects the sponsors of the services
with the beneficiaries and service providers in
Select the correct answer using the codes given below: a digital manner without any physical
interface.
a. 1 only
b. 2 only  It also ensures that the payment to the service
provider is made only after the transaction is
c. Both 1 and 2 completed.
d. Neither 1 nor 2

www.shankariasacademy.com | www.iasparliament.com
6

 Being pre-paid in nature, it assures timely  It is Kerala's first-ever tunnel for road
payment to the service provider without transport and South India‘s longest 6-lane
involvement of any intermediary. road tunnel.
Benefits  It will drastically improve connectivity to
Tamil Nadu and Karnataka.
 It is expected to be a revolutionary initiative in
the direction of ensuring a leak-proof delivery  The 1.6 km long tunnel is designed through
of welfare services. Peechi- Vazahani wildlife sanctuary.
 It can also be used for delivering services  The road will improve connectivity to
under schemes meant for providing drugs and important ports and towns in North – South
nutritional support under Mother and Child Corridor without endangering wildlife.
welfare schemes, TB eradication programmes,
drugs & diagnostics under schemes like  Kuthiran gradient is situated in the Kuthiran
Ayushman Bharat Pradhan Mantri Jan Arogya Hills, located in the western part of Anaimalai
Yojana, fertilizer subsidies etc. Hills

 Even the private sector can leverage these 4. a


digital vouchers as part of their employee Central Bank Digital Currency
welfare and corporate social responsibility
programmes.  The Reserve Bank of India (RBI) recently said
it will begin testing its own digital currency.
2. c
 Several economies, including the U.S., the
Controller General of Accounts (CGA) European Union and China, have been
 Deepak Das recently took charge as the new working to issue their own Central Bank
Controller General of Accounts. Digital Currency (CBDC).

 He is the 25th officer to hold the position of  The Bahamas launched the world‘s first
Controller General of Accounts (CGA). CBDC last year.

 The office of Controller General of Accounts CBDC


(CGA) works under the Department of  A CBDC is no different from the cash that we
Expenditure, Ministry of Finance. hold in our wallets, except that it exists in a
 It is the Principal Accounting Adviser to digital form in a digital wallet supervised by
Government of India and is responsible for the central bank.
establishing and maintaining a technically  Economically, the digital rupee is similar to
sound Management Accounting System. rupee coins and notes.
 The Office of CGA prepares monthly and  It is simply a form of fiat currency that is
annual analysis of expenditure, revenues, backed by the central bank and does not
borrowings and various fiscal indicators for the possess any intrinsic value.
Union Government.
 In fact, it is similar to private digital currencies
3. a in the sense that people accept these
Kuthiran Tunnel currencies merely because they believe that
others will also accept them.
 Union Minister for Road Transport and
Highways Shri Nitin Gadkari has recently  The RBI‘s digital rupee will not directly replace
instructed to open one side of the Kuthiran demand deposits held in banks.
Tunnel in Kerala.  Physical cash will continue to be used by banks
 It is a twin-tube tunnel, with three lanes in and people wishing to withdraw cash from
each tube, located at Kuthiran in Thrissur banks can still do so.
district of Kerala.  But they can also opt to convert their bank
deposits into the new digital rupee.

www.shankariasacademy.com | www.iasparliament.com
7

5. a  In January, 2007, in Abhay Nath v University


of Delhi and Others, the SC directed that
Amagarh Fort
reservation of 15% for Scheduled Castes and
 The fort, located in Jaipur, Rajasthan, is at the 7.5% for Scheduled Tribes be introduced in the
centre of a conflict between the tribal Meena AIQ.
community and local Hindu groups.
 Until 2007, no reservation was implemented
 Members of the Meena community say the within the All India Quota for medical
Amagarh Fort was built by a Meena ruler admission.
predating Rajput rule in Jaipur, and this has
 When the Central Educational Institutions
been their holy site for centuries where they
(Reservation in Admission) Act became
worship Amba Mata.
effective in 2007, providing for uniform 27%
Meena Community reservation to the OBCs, the scheme was
implemented in all the Central Educational
 They are also known as Meos or Mewati. Institutions.
 The tribal members accused Hindu groups of  However, this was not extended to the AIQ
trying to appropriate tribal symbols into the seats of State medical and dental colleges.
Hindutva fold, and of changing the name of
Amba Mata to Ambika Bhawani. 7. c
Amagarh Fort Airports Economic Regulatory Authority of
India (AERA) Amendment Bill 2021
 The present form of the Amagarh Fort was
given in the 18th century by Maharaja Sawai  Recently, the Lok Sabha passed the Airports
Jai Singh II, founder of Jaipur. Economic Regulatory Authority of India
(AERA) Amendment Bill, 2021.
 It has always been believed that there was
some construction at the place before Jai Singh  It was first introduced in March 2021 and
II built the fort. subsequently referred to a parliamentary
standing committee on transport, tourism and
 Prior to Rajput rule by the Kachhwaha culture, which approved it without any
dynasty, Jaipur and its nearby regions were changes.
ruled by Meenas, who had political control.
 It seeks to amend the Airports Economic
 It is believed that the fort was built by a Meena Regulatory Authority of India Act, 2008.
Sardar from the Nadla gotra, now known as
Badgoti Meenas.  AERA 2008 Act designates an airport as a
major airport if it has an annual passenger
 Sardars from the Meena community ruled traffic of at least 35 lakh.
large parts of Rajasthan till around 1100 AD.
 Recent amendment will allow AERA to
6. b regulate tariff and other charges for
All India Quota (AIQ) Scheme aeronautical services for not just major
airports with annual passenger traffic of more
 The AIQ was introduced in 1986 under the than 35 lakh, but also a group of airports.
directions of the Supreme Court (SC) to
provide for domicile-free merit-based  The government will be able to club profitable
opportunities to students from any State to and non-profitable airports as a
aspire to study in a medical college located in combination/package to bidders to make it a
another State. viable combination for investment under PPP
(Public-Private Partnership) mode.
 It comprises 15% of UG seats and 50% of PG
seats in government medical colleges. 8. b

 Remaining chunk of the seats in state Factoring Regulation (Amendment) Bill


medical/dental colleges is reserved for
students domiciled in their respective states.

www.shankariasacademy.com | www.iasparliament.com
8

 Factoring business is a business where an women self-help groups (SHGs) of Mission


entity acquires the receivables of another Shakti in community-led water supply
entity for an amount. management.
 Factor can be a bank, a registered non-banking  Odisha's Puri became the first Indian city to
financial company or any company registered achieve citywide ‗drink from tap‘ water for 24
under the Companies Act. hours with 100 per cent metered household
connections.
 Receivables is the total amount that is owed or
yet to be paid by the customers (referred as the  The project has been implemented under the
debtors) to the assignor for the use of any 5T initiative of the State government.
goods, services or facility.
 With this, Puri also joined the league of cities
 The bill amends the definitions of like London, Los Angeles, and Singapore to
"receivables", "assignment", and "factoring provide 24-hour clean drinking water.
business" to bring them at par with
international definitions.
 The bill empowers Reserve Bank of India 03-08-2021
(RBI) to make regulations for granting 1) Consider the following statements with respect to
registration certificates to a factor, filing of PM CARES for Children Scheme
transaction details with the Central Registry
and all other matters. 1. It aims to support children who have lost both
the Parents or legal Guardian or Adoptive
9. c Parents or Surviving Parent to COVID-19
I-MESA Scheme pandemic.

 Social audit is an audit of a scheme jointly by 2. The Ministry of Finance shall be the nodal
the Government and the people, especially by Ministry for execution of the scheme at the
those who are affected by the scheme or its Central level.
beneficiaries. 3. The Scheme will provide a monthly stipend up
 Information-Monitoring, Evaluation and to the age of 18 years and a lump sum of Rs. 10
Social Audit (I-MESA) in FY 2021-22 is lakh on turning 18 years of age.
launched by Ministry of Social Justice and Which of the statement(s) given above is/are correct?
Empowerment.
a. 1 only
 The social audits will be done through Social
Audit Units (SAU) of the States and National b. 3 only
Institute for Rural Development and c. 1 and 3 only
Panchayati Raj.
d. 1, 2 and 3
10. d
Sujal-Drink from Tap Mission
2) Consider the following statements with respect to
 ‗Sujal-Drink from Tap Mission‘ is launched in Scheme for Promotion of Culture of Science (SPoCS)
the state of Odisha.
1. The Scheme provides for setting up of Science
 It aims to provide quality drinking water that Cities and Science Centres in all the States of
would be suitable for consumption from the the country.
tap.
2. The Scheme has been operated by the Ministry
 It had been launched to ensure quality tapped of Culture.
drinking water for more than 1.5 million 3. States willing to avail of this Scheme have to
people in more than 15 urban areas. provide land and share the cost of setting up of
 The initiative has been launched under the facility.
‗Jalsathi‘ programme by partnering with Which of the statement(s) given above is/are correct?

www.shankariasacademy.com | www.iasparliament.com
9

a. 1 only d. Neither 1 nor 2


b. 1 and 3 only
c. 2 and 3 only 6) Consider the following statements with respective
to Pangolin
d. 1, 2 and 3
1. Indian pangolin is found throughout North
Eastern states, where Chinese Pangolin is
3) Which of the following statements is incorrect? found in Kumaon Himalayas.
a. Natural rubber is not naturally suited for colder 2. Indian pangolin is listed as ―endangered‖ and
conditions during winter in the Northeast region of the Chinese pangolin has been listed as
India ―critically endangered‖.
b. Kerala is the major producer of rubber in India Which of the statement(s) given above
is/are incorrect?
c. Rubber production seen a significant increase in
the North-Eastern parts of India since 2017 a. 1 only
d. World‘s first genetically modified (GM) rubber b. 2 only
plant was recently planted in Arunachal Pradesh
c. Both 1 and 2
d. Neither 1 nor 2
4) Consider the following statements with respect to
Eutelsat Quantum Satellite
7) Which of the following statement(s) is/ are correct
1. It is the world's first commercial fully
with respect to Global Conservation Assured | Tiger
reprogrammable satellite.
Standards (CA|TS)
2. It was launched by the European Space Agency
1. It is a globally accepted conservation tool that
(ESA).
sets best practices and standards to manage
Which of the statement(s) given above is/are correct? tigers and assessments to benchmark progress.
a. 1 only 2. This tool is launched by Ministry of
Environment, Forest and Climate Change.
b. 2 only
Select the correct answer using the codes given below:
c. Both 1 and 2
a. 1 only
d. Neither 1 nor 2
b. 2 only
c. Both 1 and 2
5) Consider the following statements with respect to
Office of the Controller General of Patents, Designs & d. Neither 1 nor 2
Trade Marks
1. It functions under the Department of
8) Consider the following statements with respective
Promotion of Industry and Internal Trade
to Employee Stock Option Plan
(DPIIT), Ministry of Commerce & Industry.
1. It give employees the right to buy shares of
2. Geographical Indications Registry, established
their company at a future date at a pre-decided
in Chennai, works under the Office of the
price.
Controller General of Patents, Designs & Trade
Marks 2. It help businesses, particularly cash-strapped
start-ups, attract and retain talented people.
Which of the statement(s) given above is/are correct?
Which of the statement(s) given above is/are correct?
a. 1 only
a. 1 only
b. 2 only
b. 2 only
c. Both 1 and 2

www.shankariasacademy.com | www.iasparliament.com
10

c. Both 1 and 2 pandemic during the period starting from


11th March 2020.
d. Neither 1 nor 2
 The objective of the Scheme is to ensure
comprehensive care and protection of Children
9) Which of the following statement(s) is/ are correct in a sustained manner, and enable their
with respect to National Internet Exchange of India wellbeing through health insurance, empower
(NIXI) them through education and equip them for
self-sufficient existence with financial support
1. It is an autonomous agency under Ministry of
on reaching 23 years of age.
Electronics and Information Technology.
2. It is managing and operating the .IN country  Fixed Deposit - PM CARES will contribute
code domain and Internationalized Domain through a specially designed scheme to create
Name for India. a corpus of Rs 10 lakh for each child when s/he
reaches 18 years of age.
Select the correct answer using the codes given below:
 This corpus will be used to give a monthly
a. 1 only financial support/ stipend from 18 years of
b. 2 only age, for the next five years to take care of his or
her personal requirements during the period of
c. Both 1 and 2 higher education.
d. Neither 1 nor 2  On reaching the age of 23 years, s/he will get
the corpus amount as one lump-sum for
personal and professional use.
10) Which of the following statement(s) is/ are correct
with respect to Galileo Project  School Education - Child under 10 years will
be given admission in the nearest Kendriya
1. The objectives of the project is to search for Vidyalaya or in a private school as a day
extra-terrestrial civilisations using artificial scholar.
intelligence (AI).
 Child between 11-18 years will be admitted in
2. The project module will be integrated with any Central Government residential school
International Space Station (ISS) and observe such as Sainik School, Navodaya Vidyalaya etc.
nearby asteroids.
 Support for Higher Education - The child
Select the correct answer using the codes given below: will be assisted in obtaining education loan for
a. 1 only Professional courses / Higher Education in
India as per the existing Education Loan
b. 2 only norms.
c. Both 1 and 2  The interest on this loan will be paid by the PM
d. Neither 1 nor 2 CARES.
 Health Insurance - All children will be a
beneficiary under Ayushman Bharat Scheme
Answers (PM-JAY) with a health insurance cover of Rs.
1. a 5 lakhs.

PM CARES for Children Scheme  The premium amount for these children till the
age of 18 years will be paid by PM CARES.
 The PM CARES for Children Scheme was
launched by Prime Minister Shri Narendra  Implementing Agency - The Ministry of
Modi on 29th May, 2021 Women and Child Development shall be the
nodal Ministry for execution of the scheme at
 It aims to support children who have lost both the central level.
the Parents or legal Guardian or Adoptive
Parents or Surviving Parent to COVID-19  Department of the State/UT government
dealing with the Juvenile Justice set up in the

www.shankariasacademy.com | www.iasparliament.com
11

State shall be the nodal agency at State level.  Growth of young rubber plants remains
The District Magistrates shall be the nodal suspended during the winter months, which
authority at District level for execution of the are also characterized by progressive drying of
scheme. the soil.
2. d  This is the reason for the long immaturity
period of this crop in the region.
 National Council of Science Museums (NCSM)
has developed a chain of science museums and  The GM rubber has additional copies of the
science centers throughout the Country gene MnSOD, or manganese-containing
including Science Cities through Scheme for superoxide dismutase, inserted in the plant,
Promotion of Culture of Science (SPoCS). which is expected to tide over the severe cold
conditions during winter — a major factor
 Currently, 25 Science Museums/Science
affecting the growth of young rubber plants in
Centres in the country are functioning under
the region.
the administrative control of National Council
of Science Museums (NCSM). 4. c
 NCSM is an autonomous organization under Eutelsat Quantum Satellite
the Ministry of Culture, Govt. of India.
 This is the world's first commercial fully
Scheme for Promotion of Culture of Science reprogrammable satellite launched by the
(SPoCS) European Space Agency (ESA) from French
Guiana.
 The Ministry of Culture operates Scheme for
Promotion of Culture of Science (SPOCS).  Eutelsat Quantum, developed under an ESA
partnership project with satellite operator
 The Scheme provides for setting up of Science
Eutelsat and prime manufacturer Airbus, is the
Cities and Science Centres in all the States of
world‘s first commercial fully flexible software-
the country subject to availability of funds for
defined satellite.
the purpose.
 Eutelsat Quantum weighs 3.5 tonnes and
 States willing to avail of this Scheme have to
consists of eight communication beams.
provide land and share the cost of setting up of
facility and corpus for upkeep and  Each of the beams can be modified to make
maintenance. changes to the coverage area and its
telecommunications signal.
3. d
Reprogrammable Satellite
Rubber Production in India
 These satellites allow the user to change the
 Kerala is the major producer of rubber in
communications as per need, in real time.
India.
 Even while orbiting in a fixed position at
 Rubber production seen a significant increase
35,000 kms above Earth, the satellite can be
in the North-Eastern parts of India since 2017.
reprogrammed.
 A Rubber Board research farm in Guwahati,
 Because it can be reprogrammed in orbit, it
Assam has recently planted the World‘s first
can respond to changing demands for data
genetically modified (GM) rubber plant
transmission and secure communications
tailored for the climatic conditions in the
during its 15-year lifetime.
Northeast.
5. c
 Natural rubber is a native of warm humid
Amazon forests and is not naturally suited for  Union Minister for Commerce & Industry
the colder conditions in the Northeast, which reviewed the functioning of the Office of
is one of the largest producers of rubber in Controller General of Patents, Designs and
India. Trademarks in Mumbai.

www.shankariasacademy.com | www.iasparliament.com
12

Office of the Controller General of Patents,  CA|TS has been agreed upon as an
Designs & Trade Marks (CGPDTM) accreditation tool by the global coalition of
Tiger Range Countries (TRCs) and has been
 The Office of the Controller General of Patents,
developed by tiger and protected area experts,
Designs & Trade Marks (CGPDTM) is located
officially launched in 2013.
at Mumbai.
 There are currently 13 tiger range countries -
 It functions under the Department of
India, Bangladesh, Bhutan, Cambodia, China,
Promotion of Industry and Internal Trade
Indonesia, Lao PDR, Malaysia, Myanmar,
(DPIIT), Ministry of Commerce & Industry.
Nepal, Russia, Thailand and Vietnam.
 The Controller General supervises the working
 CA|TS is a set of criteria which allows tiger
of the Patents Act, 1970, the Designs Act, 2000
sites to check if their management will lead to
and the Trade Marks Act, 1999 and also
successful tiger conservation.
renders advice to the Government on matters
relating to these subjects.  It sets minimum standards for effective
management of target species and encourages
 The Head Office of the ‗Patent office‘ is in
assessment of these standards in relevant
Kolkata, ‗Trade Mark Registry‘ is in Mumbai
conservation areas.
and the ‗GI Registry‘ is in Chennai.
 Until CA|TS, there has not been a set of criteria
 The Offices of ‗The Patent Information System‘
which not only provide clarity on management
(PIS) and ‗National Institute of Intellectual
of tiger sites, but also encourage further
Property Management‘ (NIIPM) are at Nagpur.
development and sharing of best practice
 In order to protect the Geographical across the tiger range countries.
Indications of goods a Geographical 8. c
Indications Registry has been established in
Chennai to administer the Geographical Employee Stock Option Plan
Indications of Goods (Registration and
 ESOPs give employees a chance to buy their
Protection) Act, 1999 under the CGPDTM.
firm‘s shares at an attractive price.
6. a
 Listed companies may offer these at a discount
Pangolin to the current market price.
 The Indian Pangolin is found throughout the  ESOPs are intended to give employees a sense
country south of the Himalayas, excluding the of ownership in a company and to work
north-eastern region while the Chinese keeping in mind the interests of the company‘s
Pangolin ranges through Assam and the shareholders.
eastern Himalayas.
 Companies grant ESOPs to their employees as
 The Chinese pangolin is distinguished from part of their compensation package.
other Asian pangolins by its almost helmeted
appearance, smaller scales than the Indian  While ESOPs have traditionally been granted
pangolin. to senior employees, many firms, particularly
start-ups, are now extending them beyond the
 All pangolin species are listed in Convention top echelons.
on International Trade in Endangered Species
(CITES) Appendix I.  A significant proportion of the salary at many
start-ups may come in the form of ESOPs.
 In India, pangolins, both Indian and Chinese,
are protected under Schedule 1 of the Wildlife  While this may imply lower cash in hand for
(Protection) Act 1972. the employees, the potential for windfall gains
on sale of these shares at some point in the
7. a future makes employees value them.
Global Conservation Assured | Tiger 9. b
Standards (CA|TS)
National Internet Exchange of India (NIXI)

www.shankariasacademy.com | www.iasparliament.com
13

 NIXI is a not-for-profit organization (section 8 application money and payment of dividend or


of the Companies Act 2013) working since interest warrants.
2003 for spreading the internet infrastructure
2. Non-scheduled payments banks are not
to the citizens of India through the following
eligible to act as a banker to an issue (BTI).
activities:
Which of the statement(s) given above is/are correct?
1. Internet Exchanges through which the internet
data is exchanged amongst Internet Service a. 1 only
Providers (ISP‘s), Data Centers and Content
b. 2 only
Delivery Network (CDNs).
c. Both 1 and 2
2. .IN Registry, managing and operation of .IN
country code domain and .BHARAT IDN d. Neither 1 nor 2
(Internationalized Domain Name) domain for
India.
3. Indian Registry for Internet Names and 2) Consider the following statements with respect to
Coir Geo Textiles
Numbers (IRINN), managing and operating
Internet Protocol (IPv4/IPv6). 1. It is an impermeable fabric used for
improvement of sub-grade soil strength in
 NIXI has developed an IPv6 index portal for
road pavements and stabilization of side
the Internet community.
slopes.
 NIXI-IP-INDEX portal will showcase the IPv6 2. Coir Geo Textiles are being used for the
adoption rate in India and across the world. construction of rural roads under the Pradhan
 It can be used to compare IPv6 Indian Mantri Gram Sadak Yojana (PMGSY-III).
adoption rate with other economies in the Which of the statement(s) given above is/are correct?
world.
a. 1 only
 It will also include details about web adoption
in IPv6, IPv6 traffic etc. b. 2 only
10. a c. Both 1 and 2
Galileo Project d. Neither 1 nor 2

 Researchers from Harvard University are


using artificial intelligence (AI) to search for 3) Consider the following statements:
extra-terrestrial civilisations.
1. Article 243 G of Indian Constitution mandated
 The project named Galileo Project will use Panchayats to prepare and implement plans
extensive AI object recognition and for economic development and social justice.
algorithmic approaches to search for terrestrial
objects or satellites. 2. Sabki Yojana Sabka Vikas is a campaign for
inclusive and holistic preparation of Gram
 The team will use a network of mid-sized, Panchayat Development Plan (GPDP).
high-resolution telescopes and detector arrays
with suitable cameras and computer systems Which of the statement(s) given above is/are correct?
as part of their research to find alien a. 1 only
civilisations
b. 2 only
c. Both 1 and 2
04-08-2021 d. Neither 1 nor 2
1) Consider the following statements with respect to
Banker to an Issue (BTI)
4) Consider the following statements:
1. Banker to an Issue means a bank carrying
activities, including acceptance and refund of

www.shankariasacademy.com | www.iasparliament.com
14

1. India shares its border with eight countries d. Neither 1 nor 2


and Bangladesh shares the longest border with
India.
7) Which of the following statement(s) is/ are correct
2. India has fenced its borders more along the
with respect to Skyglow
Indo-Bangladesh border than the Indo-
Pakistan border. 1. It is an omnipresent sheet of light across the
night sky in and around cities that can block all
Which of the statement(s) given above is/are correct?
but the very brightest stars from view.
a. 1 only
2. Sunlight reflected off the moon and earth, and
b. 2 only faint air glow in the upper atmosphere are few
natural sources of sky glow.
c. Both 1 and 2
Select the correct answer using the codes given below:
d. Neither 1 nor 2
a. 1 only
b. 2 only
5) Consider the following statements with respect to
Positive Pay System c. Both 1 and 2
1. It is a process of reconfirming the key details, d. Neither 1 nor 2
such as date, name of the beneficiary/payee
amount, of large value cheques.
8) Consider the following statements with respective
2. It has been developed by the National
to Minervarya Pentali
Payments Corporation of India and
implemented by the Reserve Bank of India 1. It is a newly discovered frog belongs to family
(RBI). of Semi-Aquatic frogs.
3. It is mandatory for all account holders issuing 2. It is largest known Minervarya (genus) which
cheques for Rs. 50,000 and above. is endemic to the Nagaland.
Which of the statement(s) given above is/are correct? Which of the statement(s) given above is/are correct?
a. 3 only a. 1 only
b. 1 and 2 only b. 2 only
c. 2 and 3 only c. Both 1 and 2
d. 1, 2 and 3 d. Neither 1 nor 2

6) Consider the following statements with respective to 9) Which of the following statement(s) is/ are correct
Genetically Modified (GM) crops with respect to Hypervitaminosis
1. India allows the import of GM soybean, where 1. It is a condition of abnormally high storage
import of GM soya bean seeds has not been levels of vitamins, which can lead to various
approved in India. symptoms as over excitement, or even toxicity.
2. In India Bt cotton and DMH-11 mustard are 2. It is caused only water soluble vitamins like
only two GM crops that are allowed of vitamin C, Vitamin B6 and not by fat soluble
commercial cultivation. vitamins.
Which of the statement(s) given above Select the correct answer using the codes given below:
is/are incorrect?
a. 1 only
a. 1 only
b. 2 only
b. 2 only
c. Both 1 and 2
c. Both 1 and 2

www.shankariasacademy.com | www.iasparliament.com
15

d. Neither 1 nor 2  They are natural eco friendly, erosion control


blankets in woven and non-woven
preparations.
10) Which of the following statement(s) is/ are correct
with respect to Halam Tribes  It protect land surface and promote quick
vegetation.
1. They are scheduled tribes of Tripura, who
belong to the Kuki-Chin tribes of Tibeto-  They are bio-degradable geo-textiles used for
Burmese ethnic group. improvement of sub-grade soil strength in
road pavements and stabilization of side
2. They live in typical Tong Ghar, specially made slopes, as it can retard soil erosion.
of bamboo and Chan grass.
 It has been used in various places for
Select the correct answer using the codes given below: improving the properties and strength of sub-
a. 1 only grade soil layer by providing a physical
separation of sub-base and sub-grade layers.
b. 2 only
 The Indian Road Congress (IRC) has also
c. Both 1 and 2 accredited the use of coir geo-textiles in
d. Neither 1 nor 2 construction of roads.
 The coir geo textiles are being used for
construction of rural roads under the Pradhan
Answers Mantri Gram Sadak Yojana (PMGSY-III).
1. a 3. c
 In order to provide easy access to investors to Sabki Yojna Sabka Vikas Campaign
participate in public and rights issues by using
various payment avenues, Securities and  Under Article 243 G of Indian Constitution,
Exchange Board of India (SEBI) has recently Panchayats have been mandated for
allowed payments banks to carry out the preparation and implementation of plans for
activities of investment bankers. economic development and social justice.

 According to SEBI, Non-scheduled payments  Sabki Yojna Sabka Vikas is a campaign for
banks, which have prior approval from the inclusive and holistic preparation of Gram
Reserve Bank of India (RBI), will be eligible to Panchayat Development Plan (GPDP).
act as a banker to an issue (BTI) subject to  In this endeavor, convergence was sought with
fulfilment of the conditions stipulated in the all Departments relating to 29 devolved
BTI Regulations. subjects listed in XIth Schedule of the
Bankers to an Issue Constitution.

 It means a scheduled bank carrying on all or  The objectives of ‗Sabki Yojna Sabka Vikas‘
any of the following issue related activities broadly include strengthening of elected
namely: representatives and Self Help Groups.

1. acceptance of application and application 4. b


monies
Borders Covered by fence in Kms
2. acceptance of allotment or call monies
3. refund of application monies Indo-Pakistan 2041
4. payment of dividend or interest warrants
Indo-Bangladesh 3141
2. c
5. b
Coir Geotextiles

www.shankariasacademy.com | www.iasparliament.com
16

 Banks have been informing customers about  In DMH-11 mustard, genetic modification
making mandatory, the positive pay system to allows cross-pollination in a crop that self-
safeguard against cheque frauds. pollinates in nature.
Positive Pay System  The poultry industry is demanding a permit for
the import of crushed genetically modified
 It is a process of reconfirming the key details of
(GM) soy seeds for captive consumption of
large value cheques.
farmers from the Central government.
 It has been developed by the National 7. c
Payments Corporation of India.
Skyglow
 Under this system, a person issuing the high-
value cheque submits certain essential details  The brightening of the night sky over inhabited
of that cheque like date, name of the areas because of streetlights, security
beneficiary/payee amount etc. to the drawee floodlights and outdoor ornamental lights
bank. cause the Skyglow, it is one of the components
of light pollution.
 The details can be submitted through
electronic means such as SMS, mobile app,  This light floods directly into the eyes of the
internet banking, ATM etc. Nocturnal (active at night) and also into the
skies and misleads their path.
 The details are cross-checked while issuing the
cheque and any discrepancy is flagged.  The natural component of sky glow has five
sources:
Limits
1. Sunlight reflected off the moon and earth,
 RBI has told banks to enable the facility for all
account holders issuing cheques for amounts 2. Faint air glow in the upper atmosphere (a
of Rs.50, 000 and above. permanent, low-grade aurora),

 It has also said that while availing of the 3. Sunlight reflected off interplanetary dust
facility is at the discretion of the account (zodiacal light),
holder, banks may consider making it 4. Starlight scattered in the atmosphere,
mandatory in case of cheque values of Rs. 5
lakh and above. 5. Background light from faint, unresolved stars
and nebulae (celestial objects or diffuse masses
6. b of interstellar dust and gas that appear as hazy
Genetically Modified (GM) crops in India smudges of light).

 A GM or transgenic crop is a plant that has a  A recent study has shown that the Skyglow
novel combination of genetic material obtained forces dung beetles in the city to abandon the
through the use of modern biotechnology. Milky Way as their compass, they rely instead
on earthbound artificial lights as beacons.
 Bt cotton is the only GM crop that is allowed in
India. It has alien genes from the soil 8. a
bacterium Bacillus thuringiensis (Bt) that Minervarya Pentali
allows the crop to develop a protein toxic to
the common pest pink bollworm.  The new frog species named Minervarya
Pentali belongs to the family of Dicroglossidae.
 Herbicide Tolerant Bt (Ht Bt) cotton, on the
other hand is derived with the insertion of an  The family Dicroglossidae comprises 202
additional gene, from another soil bacterium, species of semiaquatic frogs distributed by the
which allows the plant to resist the common tropical and subtropical regions of Africa and
herbicide glyphosate. Asia and Papua New Guinea.

 In Bt brinjal, a gene allows the plant to resist  It was discovered from the Western Ghats
attacks of fruit and shoot borers. biodiversity hotspot, extending along the
southwest coast of the Indian Peninsula.

www.shankariasacademy.com | www.iasparliament.com
17

 This new species is endemic to the southern 05-08-2021


Western Ghats, it is also among the smallest
known Minervarya (genus) frogs. 1) Which of the following statements regarding the
Commission for Air Quality Management in National
9. a Capital Region (NCR) and Adjoining Areas Bill, 2021,
Hypervitaminoses is incorrect?
a. The Chairperson of the Commission,
 It are primarily caused by fat-soluble vitamins
established through this bill, will have a tenure of 5
(D and A), as these are stored by the body for
years or till the age of 60 years, whichever is earlier
longer than the water-soluble vitamins.
b. The Bill dissolves the Environment Pollution
 Generally, toxic levels of vitamins stem from Prevention and Control Authority established in
high supplement intake and not always from
the NCR in 1998
natural sources but rather the mix of natural,
derived vitamins and enhancers (vitamin c. In case of conflicts, directions of the
boosters). Commission, established through this bill, will
prevail over the orders of the respective state
 Toxicities of fat-soluble vitamins can also be governments, and the CPCB
caused by a large intake of highly fortified
foods, but natural food in modest levels rarely d. The Commission may collect an environmental
deliver extreme or dangerous levels of fat- compensation from farmers causing pollution by
soluble vitamins. stubble burning
 Recently, activists have warned the Food
Safety and Standards Authority of India 2) Consider the following statements:
(FSSAI) of the adverse impacts of Food
Fortification on health and livelihoods. 1. Under Article 30 of Indian Constitution,
Parliament, by law, can restrict or abrogate the
10. c rights of the members of the armed forces.
Halam Tribes 2. For the first time, Essential Defence Services
 Halams are also known as Mila Kuki, though Bill, 2021, made strikes by government
they are not at all Kukis in terms of language, employees explicitly illegal in India.
culture and living style. Which of the statement(s) given above is/are correct?
 As per 2011 Census, their total population is a. 1 only
57,210 and distributed throughout the State of
Tripura. b. 2 only

 Halams are divided into several sub-clans c. Both 1 and 2


which are referred to as ―Barki-Halam‖. d. Neither 1 nor 2
 Their language is also more or less similar to
that of the Tibeto-Burman family.
3) Consider the following statements with respect to
 Major sub-clans of Halams are Koloi, Korbong, Fast Track Special Court (FTSCs)
Kaipeng, Bong, Sakachep, Thangachep,
1. It is a Centrally Sponsored Scheme and the
Molsom, Rupini, Rangkhowl, Chorai, Lankai,
Central Share would be funded from Nirbhaya
Kaireng (Darlong), Ranglong, Marchafang and
Fund.
Saihmar.
2. They were established through the Protection
 Halams live in typical "Tong Ghar" specially
of children against sexual offences (POCSO)
made of bamboo and Chan grass.
Act, 2012, for expeditious disposal of cases of
 Apart from plain land cultivation they still rape.
practice Jhum cultivation and depend on both Which of the statement(s) given above is/are correct?
the activities beside other substitute works.
a. 1 only

www.shankariasacademy.com | www.iasparliament.com
18

b. 2 only 2. It states that State-level medical board‘s


opinion is essential for a pregnancy to be
c. Both 1 and 2
terminated after 24 weeks in case of
d. Neither 1 nor 2 substantial foetal abnormalities.
Which of the statement(s) given above is/are correct?
4) Consider the following statements with respect to a. 1 only
Commercial Crew Programme
b. 2 only
1. It is a partnership to develop safe, reliable, and
c. Both 1 and 2
cost-effective human transportation to and
from the International Space Station (ISS). d. Neither 1 nor 2
2. It is an initiative of National Aeronautics and
Space Administration (NASA) in partnership
7) Which of the following statement(s) is/ are correct
with American private industry.
with respect to Global Positioning System (GPS)
Which of the statement(s) given above is/are correct? Clocks
a. 1 only 1. It is based on atomic clock technology which is
synchronized and guided by a single master
b. 2 only
clock.
c. Both 1 and 2
2. Ministry of Railways amended few rulebook
d. Neither 1 nor 2 provisions to provide GPS clocks for train
operations to ensure uniformity in time.
Select the correct answer using the codes given below:
5) Consider the following statements with respect to
National Child Labour Project (NCLP) a. 1 only
1. It is a Central Sector Scheme in which children b. 2 only
in the age group of 5-8 years are withdrawn
c. Both 1 and 2
from work and put into NCLP Special Training
Centres. d. Neither 1 nor 2
2. PENCiL is a dedicated online portal to ensure
the effective enforcement of the provisions of
8) Consider the following statements with respective
the Child Labour Act and smooth
to Strait Island
implementation of NCLP Scheme.
1. It is a small island of Andaman & Nicobar,
Which of the statement(s) given above is/are correct?
separated from Ritchie's Archipelago through
a. 1 only Diligent Strait.
b. 2 only 2. The island is inhabited by Jarawas and
Sentinelese tribes of Andaman & Nicobar.
c. Both 1 and 2
Which of the statement(s) given above is/are correct?
d. Neither 1 nor 2
a. 1 only
b. 2 only
6) Consider the following statements with respective
to Medical Termination of Pregnancy (Amendment) c. Both 1 and 2
Act, 2021
d. Neither 1 nor 2
1. According to the act opinion of three registered
medical practitioner is required for
termination of pregnancy up to 20 weeks of 9) Which of the following statement(s) is/ are correct
gestation. with respect to Tribunal Reform Bill 2021

www.shankariasacademy.com | www.iasparliament.com
19

1. It seeks to provide for uniform terms and of pollution may cause adverse impact on air
conditions of the various members of the quality in the NCR.
Tribunal and abolish certain tribunals.
 The Bill also dissolves the Environment
2. It empowers the Central Government to make Pollution Prevention and Control Authority
rules for qualifications, appointment, and established in the NCR in 1998.
other terms and conditions of service of
Members of Tribunals.  The Chairperson and members of the
Commission will have a tenure of three years
Select the correct answer using the codes given below: or till the age of seventy years, whichever is
a. 1 only earlier.

b. 2 only  The central government will constitute a


selection committee to recommend
c. Both 1 and 2 appointments of members of the Commission.
d. Neither 1 nor 2  The Committee will be headed by the Minister
of Environment, Forest and Climate Change.

10) Which of the following statement(s) is/ are correct  The Commission will be the sole authority with
with respect to SATNAV Policy jurisdiction over matters defined in the Bill
(such as air quality management).
1. It aims to ensure the continuous availability of
secured navigation signals for civilian uses and  In case of conflicts, directions of the
free-to-air navigation signals for strategic uses. Commission will prevail over the orders of the
respective state governments, the Central
2. It aims to work towards compatibility and Pollution Control Board (CPCB), state PCBs,
interoperability of Indian satellite navigation and state-level statutory bodies.
and augmentation signals with other Global
Navigation Satellite Systems (GNSS).  Contravention of provisions of the Bill, or
orders and directions of the Commission will
Select the correct answer using the codes given below: be punishable with imprisonment of up to five
a. 1 only years, or fine of up to one crore rupees, or
both.
b. 2 only
 The Bill excludes farmers from the scope of
c. Both 1 and 2
these penalties.
d. Neither 1 nor 2
 However, the Commission may collect an
environmental compensation from farmers
causing pollution by stubble burning.
Answers
 This compensation will be prescribed by the
1. a central government.
The Commission for Air Quality Management
 Appeals against the Commission‘s orders will
in National Capital Region (NCR) and lie with the National Green Tribunal.
Adjoining Areas Bill, 2021
2. d
 The Bill provides for the constitution of a
Commission for better co-ordination, research, Essential Defence Services Bill, 2021
identification, and resolution of problems
 The Minister of Defence has recently
related to air quality in the NCR and adjoining
introduced the Essential Defence Services Bill,
areas.
2021, in Lok Sabha.
 Adjoining areas have been defined as areas in
 The bill provides for the maintenance of
Haryana, Punjab, Rajasthan, and Uttar
essential defence services so as ―to secure the
Pradesh, adjoining the National Capital
security of nation and the life and property of
Territory of Delhi and NCR, where any source
the public at large‖ and prevent staff of the

www.shankariasacademy.com | www.iasparliament.com
20

government-owned ordnance factories from  The Central Share is to be funded from


going on strike. Nirbhaya Fund.
 The Bill seeks to empower the government to 4. c
declare services mentioned in it as ―essential
defence services‖ and prohibit strikes and  The launch of Boeing‘s uncrewed Starliner
lockouts in any industrial establishment or Orbital Flight Test-2 (OFT-2) or the Crew
unit engaged in such services. Space Transportation-100 (CST-100) has been
postponed.
 This is not for the first time that strikes by
government employees are being made  This spacecraft is part of an uncrewed test
explicitly illegal by the government. flight to the International Space Station.

 The Madhya Pradesh (and Chhattisgarh) Civil  The mission is part of NASA‘s Commercial
Services Rules, 1965, prohibit demonstrations Crew Program.
and strikes by government servants and direct NASA’s Commercial Crew Program
the competent authorities to treat the
durations as unauthorised absence.  It is a partnership to develop safe, reliable, and
cost-effective human transportation to and
Constitutional Provisions from the International Space Station from the
 Under Article 33 of the Constitution, United States through a partnership with
Parliament, by law, can restrict or abrogate the American private industry.
rights of the members of the armed forces or  NASA selected Boeing and SpaceX as
the forces charged with the maintenance of commercial partners for this program in
public order so as to ensure the proper September 2014 to develop transportation
discharge of their duties and maintenance of systems for transferring crew from the US to
discipline among them. the ISS.
 Thus, for the armed forces and the police, 5. b
where discipline is the most important
prerequisite, even the fundamental right to National Child Labour Project (NCLP) Scheme
form an association can be restricted under  It is a Central Sector Scheme.
Article 19(4) in the interest of public order and
other considerations.  Under this Scheme the District Project
Societies (DPS) are set up at the district level
3. a under the Chairmanship of the
Fast Track Special Courts Collector/District Magistrate for overseeing
the implementation of the project.
 The Union Cabinet has approved the
continuation of Fast Track Special Court  Under this Scheme, the children in the age
(FTSCs) including exclusive POCSO Courts group of 9-14 years are withdrawn from work
from 2021 to 2023. and put into NCLP Special Training Centres.

 The Union Government enacted the Criminal  The NCLP Training Centres will provide them
Law (Amendment) Act, 2018 to bring more with bridge education, vocational training,
stringent provisions and expeditious trial and mid-day meal, stipend, health care etc. before
disposal of cases of rape and Protection of being mainstreamed them into formal
children against sexual offences (POCSO) Act. education system.

 This led to the establishment of the Fast Track  The children in the age group of 5-8 years are
Special Courts (FTSCs) in 2019 as Centrally directly linked to the formal education system
Sponsored Scheme. through a close coordination with the Sarva
Shiksha Abhiyan.
 FTSCs are dedicated courts that ensure swift
dispensation of justice and, it strengthens the  Further, to ensure the effective enforcement of
deterrence framework for sexual offenders. the provisions of the Child Labour Act and
smooth implementation of NCLP Scheme, a

www.shankariasacademy.com | www.iasparliament.com
21

 dedicated online portal named PENCiL GPS Clocks


(Platform for Effective Enforcement for No
 Each GPS satellite contains multiple atomic
Child Labour) is developed.
clocks that contribute very precise time data to
 Under this Scheme, funds are provided directly the GPS signals.
to the District Project Societies who in turn
 These atomic clocks are synchronized and
engage and allocate the funds to
guided by a single master clock on the ground
NGOs/Voluntary Agencies/Civil Societies
which is controlled by the United States Naval
Organisation etc. for running of Special
Observatory.
Training Centres.
6. b  GPS receivers decode these signals, effectively
synchronizing each receiver to the atomic
Medical Termination of Pregnancy clocks.
(Amendment) Act, 2021
 This enables users to determine the time to
 Under the Act, a pregnancy may be terminated within 100 billionths of a second, without the
up to 20 weeks by a married woman in the case cost of owning and operating atomic clocks.
of failure of contraceptive method or device.
GPS Clocks & Indian Railways
 The Bill allows unmarried women to also
terminate a pregnancy for this reason.  Indian Railways will provide Global
Positioning System (GPS) clocks for control
 Opinion Needed for Termination of offices to enable section controllers coordinate
Pregnancy: with station masters, loco pilots and other staff
directly involved in train operations to ensure
1. Opinion of one registered medical practitioner
uniformity in time.
(instead of two or more) for termination of
pregnancy up to 20 weeks of gestation.  The enhanced safety measure will replace
analog clocks and train operations will soon be
2. Opinion of two registered medical
guided by satellite supported timers known for
practitioners for termination of pregnancy of
precision and reliability.
20-24 weeks of gestation.
3. Opinion of the State-level medical board is  Section controllers will set the time for station
essential for a pregnancy to be terminated masters, loco pilots and others to correct their
after 24 weeks in case of substantial foetal clocks/ watches to uniformity.
abnormalities.  The Ministry of Railways amended relevant
 Every state government is required to provisions in the rule book to make way for the
constitute a Medical Board, These Medical technology upgrade.
Boards will consist of the following members:  The huge clocks at major railway stations
1. A gynaecologist, would also be modified to GPS technology by
replacing the analog mechanism.
2. A paediatrician,
8. a
3. A radiologist or sonologist,
Strait Island
4. Any other number of members, as may be
notified by the state government.  The island belongs to the East Baratang Group
of North & Middle Andaman region and lies
 It enhances the upper gestation limit from 20 east of Colebrooke Island.
to 24 weeks for special categories of women
which will be defined in the amendments to  The island is comma-shaped, and heavily
the MTP Rules and would include survivors of forested it is separated from the Ritchie's
rape, victims of incest and other vulnerable Archipelago through Diligent Strait.
women (like differently-abled women, minors)  Great Andamanese are one of five PVTGs that
etc. reside in ‗Strait Island‘ of Andaman and
7. d Nicobar Islands.

www.shankariasacademy.com | www.iasparliament.com
22

 The Jarwas of Anadaman live in Western coast coverage and its supporting infrastructure
of Middle Andaman and South Andaman designed to provide all weather passive 3-D
Islands. positioning, velocity and timing data .
 The Sentinelese tribes live in small North  Development of communication & information
Sentinel Island. and mobile phone technology, are dependent
on PVT (Position Velocity and Time) based
9. c services.
Tribunal Reform Bill 2021
 Also aims to continue and upgrade Space
 It seeks to dissolve certain existing appellate based Navigation Services (SBNS) and
bodies and transfer their functions to other Satellite-Based Augmentation System (SBAS)
existing judicial bodies. as part of government's Atmanirbhar bharat
 It seeks to empower the Central Government  Secured SBNS exclusively for the Indian
to make rules for qualifications, appointment, Strategic community is provided through
term of office, salaries and allowances, navigation with Indian Constellation (NAVIC)
resignation, removal and other terms and
 SBAS Services via GPS Aided Geo Augmented
conditions of service of Members of Tribunals.
Navigation (GAGAN) for Indian airspace.
 It provides that the Chairperson and Members
of the Tribunals will be appointed by the
Central Government on the recommendation 06-08-2021
of a Search-cum-Selection Committee.
1) Consider the following statements with respect to
 It also provides the composition of the Rashtriya Mahila Kosh (RMK)
Committee, to be headed by the Chief Justice
of India or a Judge of Supreme Court 1. It was established in 1993 as a national body
nominated by him. for socio-economic empowerment of women,
especially those in the rural and unorganized
 It aims to abolish the following Appellate sector.
tribunals and their functions are to be
2. It is an autonomous body under the aegis
transferred to the existing judicial bodies.
of Ministry of Finance.
1. Film Certification Appellate Tribunal,
3. It is a facilitating agency which provides loans
2. Airports Appellate Tribunal, to Intermediary Organizations (IMO) which
further on-lend to SHGs of women.
3. Authority for Advanced Rulings,
4. Union Minister for Women & Child
4. Intellectual Property Appellate Board
Development is the ex-officio Chairperson of
5. Plant Varieties Protection Appellate Tribunal the Governing board which administers the
RMK.
10. b
Which of the statements given above are correct?
SATNAV Policy
a. 1 and 3 only
 In order to achieve the goal of self-reliance in
India's Satellite based navigation and b. 3 and 4 only
augmentation services sector, a comprehensive c. 1, 3 and 4 only
SATNAV Policy has been proposed by the
Department of Space d. 1, 2, 3 and 4
 It aims to ensure the continuous availability of
free-to-air navigation signals for civilian uses 2) Maritime India Vision, 2030 outlines 10 key themes
and secured navigation signals for strategic which are essential for India to secure its place at the
uses. forefront of the Global Maritime Sector. Which of the
 Satellite based navigation is a constellation of following is not among them?
navigation satellites with global or regional 1. Develop best-in-class Port infrastructure

www.shankariasacademy.com | www.iasparliament.com
23

2. Promote Ocean, Coastal and River Cruise a. 1 and 2 only


Sector
b. 1 and 3 only
3. Enhance Global Share in Ship Building, Repair
c. 2 and 3 only
and Recycling
d. 1, 2 and 3
4. Enhance Cargo and Passenger Movement
through Inland Waterways
5. Become Top Seafaring Nation with World 5) With respect to Employee Stock Option Plan
Class Education & Training (ESOP), consider the following statements:
Select the correct answer using the codes given below: 1. It gives employees the right to buy shares of
their company at a future date at a pre-decided
a. 4 and 5 only
price.
b. 2 and 5 only
2. It would be granted only to senior employees
c. 2, 3 and 4 only with work experience more than 7 years.
d. None of the above Which of the statement(s) given above is/are correct?
a. 1 only
3) The Ministry of New and Renewable Energy has b. 2 only
been implementing a Scheme to support promotion of
c. Both 1 and 2
biomass based cogeneration in sugar mills and other
industries. Under the Scheme, which of the following d. Neither 1 nor 2
can be used as Biomass Resources?
1. Weeds
6) Consider the following statements with respective
2. Bagasse to Pardoning Powers of the Governor
3. Municipal Solid Waste 1. Under Article 161, the Governor has pardoning
powers, which also extends to death sentences.
4. Agro-based industrial residue
2. This power will override the restrictions
5. Wood waste produced in industrial operations
imposed under Section 433-A of the Criminal
Select the correct answer using the codes given below: Procedure Code.
a. 2, 3 and 4 only Which of the statement(s) given above is/are correct?
b. 3, 4 and 5 only a. 1 only
c. 1, 2, 4 and 5 only b. 2 only
d. 1, 2, 3, 4 and 5 c. Both 1 and 2
d. Neither 1 nor 2
4) Consider the following statements with respect to
Atlantic Meridional Overturning Circulation (AMOC)
7) Which of the following statement(s) is/
1. It is a large system of Ocean Currents driven are incorrect with respect to Prosopis Juliflora
by differences in temperature and salt content.
1. It is a shrub, and a contributing factor to
2. It transports warm water masses from the malaria transmission, especially during dry
tropics northward at the ocean surface and periods.
cold water southward at the ocean bottom.
2. It is an invasive tree species which has
3. It ensures the world‘s oceans are continually encroached over half of Shola Grasslands, in
mixed, and that heat and energy are Western Ghats range.
distributed around the earth.
Select the correct answer using the codes given below:
Which of the statements given above are correct?

www.shankariasacademy.com | www.iasparliament.com
24

a. 1 only b. 2 only
b. 2 only c. Both 1 and 2
c. Both 1 and 2 d. Neither 1 nor 2
d. Neither 1 nor 2
Answers
8) Consider the following statements with respective 1. c
to Floating rate funds
 The Government had set up Rashtriya Mahila
1. It is a fund that invests in financial Kosh (RMK) in 1993 as a national level
instruments that pays a variable or floating autonomous body under the aegis
interest rate. of the Ministry of Women
2. According to SEBI rules, 65% of the corpus of and Child Development.
floating rate funds has to be invested in  It was established for socio-economic
floating rate instruments. empowerment of women, especially those in
Which of the statement(s) given above is/are correct? the rural and unorganized sector.
a. 1 only  It provides them concessional, collateral free
micro-credit.
b. 2 only
 The operating model currently followed by
c. Both 1 and 2
RMK is that of a facilitating agency wherein
d. Neither 1 nor 2 RMK provides loans to NGO-MFIs termed as
Intermediary Organizations (IMO) which on-
lend to Self Help Groups (SHGs) of women.
9) Which of the following statement(s) is/ are correct
 The Kosh is being administered by a
with respect to Swinhoe's Softshell Turtle
Governing Board of sixteen members
1. It is found in the temple tanks of Nagshankar consisting of the ex-officio Chairperson who is
Temple in north-eastern Assam. Minister in the Ministry of Women and Child
Development.
2. It is listed as Critically Endangered under
IUCN Red List. 2. d
Select the correct answer using the codes given below: Maritime India Vision 2030
a. 1 only  With the objective of propelling India to the
forefront of the Global Maritime Sector,
b. 2 only
Ministry of Ports, Shipping and Waterways has
c. Both 1 and 2 formulated Maritime India Vision 2030 (MIV
2030).
d. Neither 1 nor 2
 It is a blueprint to ensure coordinated and
accelerated growth of India‘s maritime sector
10) Consider the following statements with respective in the next decade.
to Polyhouse Technology
 MIV 2030 projects cargo traffic to reach 2,570
1. Temperature, humidity and ventilation can be million tonnes per annum by 2030.
controlled or automated in the polyhouse.
 MIV 2030 outlines 10 key themes which are
2. It can help farmers to cultivate off-season essential for India to secure its place at the
crops and reduce threats such as extreme heat forefront of the Global Maritime Sector:
and pest attacks in crops.
Key Themes
Which of the statement(s) given above is/are correct?
1. Develop best-in-class Port infrastructure
a. 1 only

www.shankariasacademy.com | www.iasparliament.com
25

2. Drive E2E Logistics Efficiency and Cost weakening to such an extent that it could soon
Competitiveness bring big changes to the world's weather.
3. Enhance Logistics Efficiency through Atlantic Meridional Overturning Circulation
Technology and Innovation (AMOC)
4. Strengthen Policy and Institutional  It is a large system of ocean currents, like a
Framework to Support all Stakeholders conveyor belt, driven by differences in
temperature and salt content – the water‘s
5. Enhance Global Share in Ship Building, Repair
density.
and Recycling
6. Enhance Cargo and Passenger Movement  It transports warm water from the tropics
through Inland Waterways northwards into the North Atlantic.

7. Promote Ocean, Coastal and River Cruise  As warm water flows northwards it cools and
Sector some evaporation occurs, which increases the
amount of salt.
8. Enhance India‘s Global stature and Maritime
Co-operation  Low temperature and a high salt content make
the water denser, and this dense water sinks
9. Lead the World in Safe, Sustainable & Green deep into the ocean.
Maritime Sector
 The cold, dense water slowly spreads
10. Become Top Seafaring Nation with World southwards, several kilometres below the
Class Education, Research & Training surface.
3. c  Eventually, it gets pulled back to the surface
Scheme to support promotion of biomass and warms in a process called ―upwelling‖ and
based cogeneration in sugar mills and other the circulation is complete.
industries
 This global process makes sure that the world‘s
 The Ministry of New & Renewable Energy has oceans are continually mixed, and that heat
been implementing a scheme to support and energy are distributed around the earth.
promotion of biomass based cogeneration in
 This, in turn, contributes to the climate we
sugar mills and other industries for power
experience today.
generation in the country.
Causes
 The scheme was applicable for projects set up
across India.  Climate models have shown that the AMOC is
at its weakest in more than a 1,000 years.
 The programme will provide Central Financial
Assistance (CFA) for projects utilizing biomass  As the atmosphere warms due to increased
like bagasse, agro-based industrial residue, greenhouse gas emissions, the surface ocean
crop residues, wood produced through energy beneath retains more of heat.
plantations, weeds, wood waste produced in
industrial operations, etc.  However, it has not been known whether the
weakening is due to a change in circulation or
 Municipal Solid Waste is not covered it is to do with the loss of stability.
under the programme.
 A potential collapse of the system could have
 CFA will be provided at the rate of Rs.25 severe consequences for the world's weather
Lakh/MW (for bagasse cogeneration projects) systems.
and Rs.50 Lakh/MW (Nonbagasse
Effects
Cogeneration projects) under the scheme.
If the AMOC collapsed, it would create the following
4. d
effects:
 According to a recent study, the Atlantic
1. Sea level rise in the Atlantic
Ocean's current system, an engine of the
Northern Hemsiphere's climate, could be 2. Increase cooling of the Northern Hemisphere

www.shankariasacademy.com | www.iasparliament.com
26

3. A shift in monsoons in South America and  Recently, the Supreme Court has observed that
Africa the power of Governor under Article 161 of the
Constitution to commute sentence or to
4. An overall fall in precipitation over Europe and
pardon will override the restrictions imposed
North America
under Section 433-A of the Criminal Procedure
5. a Code.
Employee Stock Option Plan (ESOP)  The court held that the power conferred on the
Governor, though exercised on the aid and
 It gives employees the right to buy shares of
advice of the State, is without any restriction of
their company at a future date at a pre-decided
the actual period of imprisonment undergone
price.
by the prisoner.
 Companies grant ESOPs to their employees as
 The remission under Article 161 of the
part of their compensation package.
Constitution will override Section 433-A of the
 While ESOPs have traditionally been granted Code, if the State Government decides to be
to senior employees, many firms, particularly governed of its constitutional power.
start-ups, are now extending them beyond the 433-A of the Criminal Procedure Code
top echelons.
 It emphasis on restriction on powers of
 Since ESOPs grant employees only an option, remission or Commutation in certain cases.
they can choose not to buy the shares.
 Where a sentence of death imposed on a
 As ESOPs are part of employees‘ person has been commuted under section 433
compensation, these are taxed, when they into one of imprisonment for life, such person
exercise the option to buy and when they make
capital gains on selling them. shall not be released from prison unless he had served
at least fourteen years of imprisonment.
 ESOPs are intended to give employees a sense
of ownership in a company and to work 7. b
keeping in mind the interests of the company‘s Prosopis Juliflora
shareholders.
 It is a shrub or small tree in the family
 ESOPs however do have flip side. If the Fabaceae, it is native to Mexico, South America
financial performance of the company an and the Caribbean.
employee working with turns out to be subpar,
not just his/her salary but also his/her wealth  It has become established as an invasive weed
will be put at risk. in Africa, Asia, Australia and elsewhere.
6. b  It is a contributing factor to continuing
transmission of malaria, especially during dry
Pardoning Powers of the Governor
periods when sugar sources from native plants
 Under Article 161, the Governor too has are largely unavailable to mosquitoes.
pardoning powers, but these do not extend to
 Recently it has encroached over half Banni,
death sentences.
one of Asia‘s largest grasslands in Gujarat.
 The power of the President to grant pardon
 It has many harmful flipsides to ecology, there
extends in cases where the punishment or
are several flipsides to growing this tree
sentence is by a Court Martial but Article 161
species, such as
does not provide any such power to the
Governor. 1. It depletes groundwater availability;
 The Governor has a power to grant pardons, 2. Increases soil salinity,
reprieves, respites and remissions of
3. Makes the grassland more susceptible to
punishment or to suspend, remit or commute
wildfires.
the sentence of any person if the prisoner has
not undergone 14 years or more of actual
imprisonment.

www.shankariasacademy.com | www.iasparliament.com
27

 The invasion is also a threat for habitat  The freshwater Black softshell turtle (Nilssonia
specialist species such as the desert fox, nigricans) is extinct in the wild, but very few
Houbara bustard and spiny-tailed lizard that survive in constricting temple ponds, mostly in
have evolved to life in grasslands over Assam.
centuries.
10. c
8. c
Polyhouse Technology
Floating rate funds
 A Polyhouse is a specially constructed
 A floating rate fund, which can be a mutual structure like a building where specialised
fund or an exchange-traded fund (ETF), polythene sheet is used as a covering material
invests in bonds and debt instruments whose under which crops can be grown in partially or
interest payments fluctuate with an underlying fully controlled climatic conditions.
interest rate level.
 It is covered with a transparent material as to
 Typically, a fixed-rate investment will have a permit the entry of natural light.
stable, predictable income.
 Its ongoing development will be useful in
 However, as interest rates rise, fixed-rate India, which has 15 different agro-climatic
investments lag behind the market since their zones and will help farmers to cultivate off-
returns remain fixed. season crops that can fetch higher value and
income.
 Floating rate funds aim to provide investors
with a flexible interest income in a rising rate
environment.
07-08-2021
 Floating rate funds buy bonds whose interest
rates change according to the changing rates in 1) With respect to Foreign Direct Investment (FDI) in
the economy. Indian Textiles sector, consider the following
 This feature is thus supposed to insulate them statements:
from losses because of the rate hike and can
even increase their returns as rates rise. 1. Textile sector is open to FDI under 100%
automatic route.
9. b
2. Nearly 80% of textile units in India are Micro
Swinhoe's Softshell Turtle
Small & Medium Enterprises (MSME).
 Swinhoe's Softshell Turtle (Rafetus swinhoei) 3. Japan ranks first as the major FDI investor in
is native to China and Vietnam, its natural Indian Textiles sector.
habitat are wetlands and large lakes.
Which of the statements given above are correct?
 The species is endemic to eastern and southern
China and northern Vietnam. a. 1 and 2 only

 Its protection status includes IUCN Red List: b. 1 and 3 only


Critically Endangered and CITES: Appendix II c. 2 and 3 only
 In Vietnam, these animals have great cultural d. 1, 2 and 3
significance as people in Hanoi revere this
creature as a living god.
 Some researchers have highlighted their 2) Consider the following statements with respect to
importance to the seafloor biosystem, where PM Dakshta Aur Kushalta Sampann Hitgrahi (PM-
they contribute by enriching soil nutrients and DAKSH) Yojana
facilitating seed dispersion. 1. It is a National Action Plan exclusively for the
 They have been driven to the brink by hunting welfare of sanitation workers including waste
for its meat and eggs, as well as by destruction pickers and manual scavengers.
of its habitat.

www.shankariasacademy.com | www.iasparliament.com
28

2. It aims to provide long term and short term a. 1 and 2 only


skills, followed by settlement in
b. 1 and 3 only
employment/self-employment.
c. 2 and 3 only
3. The Scheme is being implemented by the
Ministry of Social Justice and Empowerment. d. 1, 2 and 3
Which of the statement(s) given above is/are correct?
a. 1 and 2 only 6) Consider the following statements with respective
to Abanindranath Tagore
b. 1 and 3 only
1. He mastered the Western art of oil painting,
c. 2 and 3 only
but painted themes from Indian mythology
d. 1, 2 and 3
2. He belonged to the Renaissance School or the
Revivalist School, as it represented the first
modern movement of Indian art.
3) The Pensilungpa Glacier, sometimes seen in the
news recently, is located in? Which of the statement(s) given above is/are correct?
a. Alaska, USA a. 1 only
b. Ladakh, India b. 2 only
c. Santa Cruz, Argentina c. Both 1 and 2
d. Southern Alps, New Zealand d. Neither 1 nor 2

4) Agarose is a natural polymer derived from seaweed 7) Which of the following statement(s) is/ are correct
agar. It is used for which of the following purposes? with respect to Collateral Margin
a. It is an oil-eating polymer that act as a natural 1. It is extra fund an investor receives for trading
remedy for oil spills or investing by pledging securities based on
demat account.
b. It is a wound dressing material used for
treatment of diabetic wounds 2. It is restricted only for trading shares and a
trader can share it with others if they have
c. It is a polymer modified bitumen material
power of attorney (POA).
widely used for road construction
Select the correct answer using the codes given below:
d. It is referred as a floating gold and used for
traditional medicine and in the perfume markets a. 1 only
b. 2 only
5) Consider the following statements with respect to c. Both 1 and 2
Giraffes
d. Neither 1 nor 2
1. Giraffes have socially complex structure and
cooperative care as elephants.
8) Consider the following statements with respective
2. Giraffes need to drink water on a daily basis
to Reverse Merger
and are not suitable to live in areas with scarce
water. 1. It is a combination where a smaller company
merges into a larger one.
3. Grandmother hypothesis, an idea that some
animals survive well beyond their reproductive 2. In the case of small finance banks, the holding
years to ensure their grandchildren thrive may company is expected to be merged into the
apply to giraffes. subsidiary bank, which is referred as
downstream merger.
Which of the statements given above are correct?
Which of the statement(s) given above is/are correct?

www.shankariasacademy.com | www.iasparliament.com
29

a. 1 only  As per Government FDI Policy for Textiles


Sector, Textiles is open to FDI under 100%
b. 2 only
automatic route.
c. Both 1 and 2
 Nearly 80% of textiles units in India are
d. Neither 1 nor 2 MSME resulting in fragmented nature of
industry.

9) Which of the following statement(s) is/ are correct  Japan has invested the maximum amount US$
with respect to Project REPLAN (REducing PLAstic 381.47 million as FDI in textiles sector.
from Nature)  There are no FDI specific incentives for
1. Its primary objective is to remove the existing artisans, weavers and labourers provided by
waste plastic material from nature and use it in the Government.
a semi-permanent manner.  Weavers, artisans, cooperative societies,
2. It aims to collect waste plastic from nature, handloom and handicraft agencies were being
and mixed with paper pulp in a ratio of 80 is to facilitated to register on Government e-
20. Marketplace (GeM) in all States.
Select the correct answer using the codes given below:  This arrangement would help in marketing of
their products and getting reasonable price by
a. 1 only eliminating intermediaries.
b. 2 only 2. c
c. Both 1 and 2 Pradhan Mantri Dakshta Aur Kushalta
d. Neither 1 nor 2 Sampann Hitgrahi’ (DAKSH) Yojana
 It is a National Action Plan for the
marginalized persons of SC, OBC, EBC, DNTs.
10) Consider the following statements with respective
to Preventive Detention  The programme also engages with skilling the
Safai Karamcharis including Waste Pickers,
1. It is taken under Criminal Procedure Code,
Transgenders and Women so that they can
1973 (CrPC) on grounds of suspicion that some
engage in self-employment activities.
wrong actions may be done by the person
concerned.  The main purpose of the scheme is to increase
the skill levels of the target youth by providing
2. Only Parliament have powers to enact such
for long term and short term skills, followed by
laws for the reasons related to maintenance of
settlement in employment/selfemployment.
public order or maintenance of supplies or
services essential to the community.  The Scheme is being implemented by the
Which of the statement(s) given above is/are correct? Ministry of Social Justice & Empowerment.

a. 1 only The scheme would be implemented through three


Corporations namely:
b. 2 only
1. National Scheduled Castes Finance and
c. Both 1 and 2 Development Corporation (NSFDC)
d. Neither 1 nor 2 2. National Backward Classes Finance &
Development Corporation (NBCFDC)
3. National Safai Karamcharis Finance and
Answers Development Corporation (NSKFDC)
1. d 3. b
FDI in Textiles Sector Pensilungpa Glacier
 It is located in Zanskar, Ladakh.

www.shankariasacademy.com | www.iasparliament.com
30

 Researchers have recently found that it is  But a recent study suggests that giraffes have
being retreating, and attributed the retreat to been misunderstood and are in fact a highly
an increase in the temperature and decrease in complex and social species.
precipitation during winters.
 Female giraffes maintain long-term
 The study also suggests that due to continuous relationships with other females and their own
rise in the air temperature in line with the offspring, the study found.
global trend, the melting would increase, and it
is possible that the precipitation of summer  Close bonds form between females and their
periods at higher altitudes will change from calves, who are sometimes cared for by other
snow to rain, and that may influence the females in a kind of creche.
summer and winter pattern.  Female giraffes show distress when a calf in
4. b the group dies even if it's not their own.

Agarose  Male giraffes, however, only associate


consistently with their mothers.
 An Indian scientist has recently developed an
advanced wound dressing based on agarose. Grandmother effect

 Agarose is a natural polymer derived from  The study also found that the grandmother
seaweed agar. hypothesis - the idea that some animals
survive well beyond their reproductive years to
 It is used for the treatment of infected diabetic ensure their grandchildren thrive - may apply
wounds and patients suffering from chronic to giraffes.
wounds.
 It's only been observed in a handful of species -
 This indigenous dressing will allow cost- including orcas, elephants and of course,
effective dressings for chronic wound patients human beings.
and will also pave the way for business
incubation. 6. c
Abanindranath Tagore
 The advanced wound dressing market in India
is largely monopolized by foreign companies.  He was a nephew of Rabindranath Tagore, in
his youth, hereceived training in European and
 This indigenous dressing will not only allow
Academic style from European artists.
cost-effective dressings for chronic wound
patients but also will pave way for business  However, during the last decade of the 19th
incubation. century, he developed distaste for the
5. b corporeality of European naturalism (which
represented things closer to the way one sees
Giraffe them - inspired by the principles of natural
science).
 They are the world‘s tallest mammals native to
Africa.  Mughal miniatures influenced his visual ideas
deeply.
 They are uniquely adapted to reach vegetation
inaccessible to other herbivores.  Another source of inspiration came from the
visit of the Japanese philosopher and
 Giraffes drink water when it is available, but
aesthetician Okakura Kakuzo to Kolkata in
they don‘t need to drink water on a daily basis,
1902.
which allows them to survive in areas with
scarce water. 7. a
 Traditionally, giraffes were thought to have Collateral Margin
little or no social structure, and only fleeting,
 The facility is not restricted to shares and can
weak relationships.
be availed against mutual fund units, bonds,
gold ETFs and fixed deposits too.

www.shankariasacademy.com | www.iasparliament.com
31

 Earlier brokers had pledged client securities  It is the first of its kind project in India, where
without their knowledge, to meet collateral plastic waste is de-structured, degraded,
requirements of other clients. diluted and used with paper pulp while making
handmade paper and thus reduces plastic
 Clients gave their brokers a power of attorney waste from nature.
(POA) to deal with their demat on their behalf.
 Recently, the Khadi & Village Industries
 But the misuse of this facility has led SEBI to Commission (KVIC) has secured Patent
introduce more steps in the pledging process. registration for its plastic-mixed handmade
 Before September 2020, the process of using paper. Under Project REPLAN
the shares lying in your demat account as  The patent certificate was issued to KVIC‘s
collateral margin was automatic. Kumarappa National Handmade Paper
 According SEBI‘s new rules for availing Institute (KNHPI), Jaipur.
collateral margin, control over the securities  The technology developed by KVIC uses both
held in demat account is completely with client high & low density waste polythene that not
and broker. only adds extra strength to the paper but also
 Client cannot transfer them if they have given reduces the cost by up to 34%.
a POA, this also prevents the potential misuse 10. a
of securities by brokers through pooling of
collateral. Preventive Detention
8. c  Under Section 151 of The Criminal Procedure
Code, 1973 (CrPC) preventive detention is
Reverse Merger action taken on grounds of suspicion that some
 A merger is a corporate action where two wrong actions may be done by the person
companies decide to bring together their assets concerned.
and liabilities to create a single entity that is  A police officer can arrest an individual
bigger and better than either of them. without orders from a Magistrate and without
 While a merger is usually proposed between any warrant if he gets any information that
equals, a reverse merger is a combination such an individual can commit any offense.
where a smaller company merges into a larger  The Article 22 of the Indian Constitution
one, or a loss-making company merges into a provides safeguards against the misuse of
profitable one. police powers to make arrests and detentions.
 When NBFCs were originally allowed by the  Under Entry 9 of List I (Union List),
RBI to bag small finance bank licences to offer Parliament has the exclusive power to enact a
credit to unbanked segments of the law for preventive detention for the reasons
population, the RBI required them to set up connected with defence, foreign affairs, or
their banks under the non-operating financial security of India.
holding company (NOFHCs) structure.
 Under Entry 3 of List III (Concurrent List),
 This was so that the NBFC would separate its both Parliament and State Legislature have
new banking operations from its other powers to enact such laws for the reasons
businesses. related to maintenance of public order or
 While reverse mergers usually create maintenance of supplies or services essential
uncertainty for the merging companies, to the community.
especially when loss-making entities are  According to SC preventive detention order
involved, small finance banks look set to can only be passed if the activities of the
benefit from this reverse merger. detainee affects, or are likely to adversely
9. c affect, the maintenance of public order.
Project REPLAN (REducing PLAstic from
Nature)

www.shankariasacademy.com | www.iasparliament.com
32

09-08-2021 2. Failure of Cripps Mission to solve the


constitutional deadlock
1) Under the Indian Passports Act, 1967, the passport
authority can refuse to issue passport for which of the 3. Discontent because of rising prices and
following reasons? shortage of rice and salt
1. The applicant‘s departure may be detrimental 4. Fears of Britain following a scorched earth
to the security of India policy in Assam, Bengal and Orissa against
possible Japanese advance
2. The person‘s presence abroad may prejudice
India‘s friendly relations with a foreign country Select the correct answer using the codes given below:
3. If, in the opinion of the President, the issue of a. 2 only
a passport to the applicant will not be in the b. 1 and 2 only
public interest
c. 1, 2 and 3 only
4. Conviction for any offence that involves ―moral
turpitude‖ with a sentence of not less than two d. 1, 2, 3 and 4
years in the five preceding years
Select the correct answer using the codes given below: 4) Which of the following factors are responsible for
a. 2 and 3 only increase in price of edible oils in India?
b. 3 and 4 only 1. Increased transportation costs
c. 1, 2 and 4 only 2. Mismatch in demand & supply
d. 1, 2, 3 and 4 3. Reduction of International prices of edible oils
4. Boost in domestic production owing to
favourable weather conditions
2) Indo-Pacific Oceans‘ Initiative (IPOI) is an open,
non-treaty based initiative that seeks to ensure security Select the correct answer using the codes given below:
and stability of the regional maritime domain. It a. 1 and 2 only
focuses on seven pillars of maritime security. Which of
the following is not among them? b. 1, 2 and 4 only
1. Maritime Ecology c. 1, 2 and 3 only
2. Trade Connectivity & Maritime Transport d. 1, 2, 3 and 4
3. Disaster Risk Reduction and Management
4. Science, Technology and Academic 5) Coal refers to a whole range of combustible
Cooperation sedimentary rock materials spanning over a
continuous quality scale. For convenience, this
Select the correct answer using the codes given below: continuous series is often divided into Hard coal and
a. 1 and 4 only Brown coal. Which of the following is/are not Hard
Coal?
b. 3 and 4 only
1. Lignite
c. 1, 3 and 4 only
2. Anthracite
d. None of the above
3. Coking coal
4. Bituminous coal
3) Which of the following is/are the probable reason(s)
for the onset of Quit India Movement? Select the correct answer using the codes given below:
1. News of reverses suffered by the British in a. 1 only
South-East Asia b. 3 only

www.shankariasacademy.com | www.iasparliament.com
33

c. 1 and 2 only Which of the statement(s) given above is/are correct?


d. 3 and 4 only a. 1 only
b. 2 only
6) Consider the following statements with respective c. Both 1 and 2
to IPCC Assessment Reports
d. Neither 1 nor 2
1. The First Assessment Report led to the setting
up of the UN Framework Convention on
Climate Change (UNFCCC). 9) Which of the following statement(s) is/ are correct
with respect to Crew Health and Performance
2. The Second Assessment Report was the basis
Exploration Analog (CHAPEA)
for the 1997 Kyoto Protocol.
1. It is a unique 3D printed habitat designed to
3. Fifth Assessment Report which came out in
serve as an analog for one-year missions to the
2014, guided the Paris Agreement.
Martian surface.
Which of the statement(s) given above is/are correct?
2. It is a project of European Space Agency,
a. 1 only which will be attached to the International
Space Station by 2022
b. 1 & 2 only
Select the correct answer using the codes given below:
c. 2 & 3 only
a. 1 only
d. All of the above
b. 2 only
c. Both 1 and 2
7) Which of the following statement(s) is/ are correct
with respect to Tribal Population in Arunachal d. Neither 1 nor 2
Pradesh
1. Nearly 64.2% of the total population of
10) Which of the following statement(s) is/ are correct
Arunachal Pradesh is Scheduled Tribes (STs)
with respect to Single Spotted Lanternfly
according to the 2001 Census.
1. It feeds on agricultural crops such as grapes,
2. Constitution (Scheduled Tribes) Order
apples and hops as well as maple, walnut and
(Amendment) Bill, removes the Abor tribe
willow trees.
from the list of identified STs in Arunachal
Pradesh. 2. It is an invasive species in India which has
affected more than 80 hectares of Vine yards
Select the correct answer using the codes given below:
in Nashik, Maharashtra.
a. 1 only
Select the correct answer using the codes given below:
b. 2 only
a. 1 only
c. Both 1 and 2
b. 2 only
d. Neither 1 nor 2
c. Both 1 and 2
d. Neither 1 nor 2
8) Consider the following statements with respective
to Snake Bite Envenoming
1. It is classified by the World Health Answers
Organisation (WHO) as a High-Priority 1. c
Neglected Tropical Disease (NTD).
Refusal of Passports
2. India has the highest number of snakebite
cases in the world, accounting for nearly 50%
of the global snakebite deaths.

www.shankariasacademy.com | www.iasparliament.com
34

 Under Section 6(2) of the Indian Passports 5. Disaster Risk Reduction and Management
Act, 1967, the passport authority can refuse to
6. Science, Technology and Academic
issue a passport for specific reasons such as:
Cooperation
1. The applicant being not a citizen of India
7. Trade Connectivity and Maritime Transport
2. The applicant may engage, outside India, in
3. d
activities prejudicial to the country‘s
sovereignty and integrity  After Cripps‘ departure, Gandhi framed a
resolution calling for British withdrawal and a
3. The applicant‘s departure may be detrimental
non-violent non-cooperation movement
to the country‘s security
against any Japanese invasion.
4. The person‘s presence abroad may prejudice
India‘s friendly relations with a foreign  The CWC meeting at Wardha (July 14, 1942)
country. accepted the idea of a struggle.

Refusal of passport may also be related to a person‘s Reasons for Quit India Movement
antecedents.  The failure of the Cripps Mission to solve the
1. Conviction for any offence that involves ―moral constitutional deadlock
turpitude‖ with a sentence of not less than two  There was popular discontent because of rising
years in the five preceding years prices and shortage of rice, salt, etc.,
2. If any proceedings are pending in a criminal  There were fears of Britain following a
court against the applicant scorched earth policy in Assam, Bengal and
3. If any arrest warrant or summons for Orissa against possible Japanese advance.
appearance is pending.
 News of reverses suffered by the British in
4. There is a general power to deny a passport or South-East Asia.
travel document to anyone ―if in the opinion of
 The leadership wanted to condition the masses
the Central government, it is not in public
for a possible Japanese invasion.
interest‖.
4. a
2. d
Indo-Pacific Oceans’ Initiative (IPOI)  The prices of edible oils increase rapidly in the
last 5 years during 2016-2021.
 In November 2019, Prime Minister Narendra
Modi launched the Indo-Pacific Oceans  RBD palmolein is one of the edible oils that
Initiative (IPOI) at the East Asia Summit. saw a steep increase in the prices in the last
five years.
 IPOI seeks to ensure security and stability of
the regional maritime domain.  Groundnut oil saw minimum increase in the
prices compared to other edible oils in the
 IPOI is an open, non-treaty based initiative for market.
countries to work together for cooperative and
Factors responsible for increase in prices of
collaborative solutions to common challenges
edible oils
in the region.
 Seasonality
 IPOI draws on existing regional architecture
and mechanisms to focus on seven pillars.  Supply chain constraints
Seven Pillars  Increased transportation costs
1. Maritime Security  Mismatch in demand and supply
2. Maritime Ecology  Increase in the international prices of edible
3. Maritime Resources oils
4. Capacity Building and Resource Sharing

www.shankariasacademy.com | www.iasparliament.com
35

 Shortfall in domestic production owing to  Fourth Assessment Report (2007) -


adverse weather conditions Greenhouse gas emissions increased by 70 per
cent between 1970 and 2004.
5. a
 The report won the 2007 Nobel Peace Prize for
 Coal refers to a whole range of combustible
IPCC and was the scientific input for the 2009
sedimentary rock materials spanning over a
Copenhagen climate meeting.
continuous quality scale.
 Fifth Assessment Report (2014) - Rise in
 For convenience, this continuous series is
global temperatures by 2100 could be as high
often divided into two main categories, namely
as 4.8 degree Celsius from pre-industrial times
hard coal and brown coal.
 This report formed the scientific basis for
Hard Coal
negotiations of the Paris Agreement in 2015.
1. Anthracite
 Sixth Assessment Report – It is attempting
2. Bituminous coal to provide more actionable information to help
3. Coking coal governments take policy decisions.

4. Other bituminous coal  It would likely state what the scenarios for sea-
level rise in the Bay of Bengal region is, not
Brown Coal just what the average sea-level rise across the
1. Sub-bituminous coal world is likely to be.

2. Lignite 7. c

6. d Tribal Population in Arunachal Pradesh

IPCC Assessment Reports  Nearly 64.2% of the total population of


Arunachal Pradesh is Scheduled Tribes (STs)
 The IPCC assessment reports have been according to the 2001 Census.
extremely influential in directing the dialogue
and action on climate change.  The state has registered a decadal growth of
28.1% of ST population in 1991-2001 Census.
 First Assessment Report (1990) – It
found that global temperatures have risen by  Constitution (Scheduled Tribes) Order
0.3 to 0.6 degree Celsius in last 100 years. (Amendment) Bill, 2021, provides for
modifying Part-XVIII of the Schedule to the
 This report formed the basis for negotiation of Constitution (Scheduled Tribes) Order, 1950,
the UN Framework Convention on Climate relating to the state of Arunachal Pradesh.
Change in 1992.
 It replaces certain STs with other tribes (as
 Second Assessment Report (1995) - indicated below):
Global rise in temperature by 0.3 to 0.6
degree Celsius since late 19th century, 1. Abor has been deleted from the list
―unlikely to be entirely natural in origin‖. 2. Khampti will be replaced with Tai Khamti
 This report was the scientific underpinning for 3. Mishmi, Idu, and Taroan will be replaced with
Kyoto Protocol in 1997. Mishmi-Kaman (Miju Mishmi), Idu (Mishmi),
and Taraon (Digaru Mishmi)
 Third Assessment Report (2001) – It has
predicts that by 2100, the sea level is likely to 4. Momba will be replaced with Monpa, Memba,
rise by as much as 80 cm from 1990 levels. Sartang, and Sajolang (Miji)
Glaciers to retreat during the 21st century.
5. Any Naga Tribes will be replaced with
 Presented new and stronger evidence to Nocte, Tangsa, Tutsa, and Wancho.
suggest that global warming is mostly 8. c
attributable to human activities.
Snake Bite Envenoming

www.shankariasacademy.com | www.iasparliament.com
36

 It is a potentially life-threatening disease that 10-08-2021


typically results from the injection of a mixture
of different toxins (venom) following the bite 1) Elderly in India 2021 report, found India‘s elderly
of a Venomous Snake. population to rise 41% over next decade to touch 194
million in 2031, Which of the following statements
 It is a particularly important public health is/are correct with respective to the report?
problem in rural areas of tropical and
subtropical countries situated in Africa, the 1. Currently, Kerala has the highest elderly
Middle-East, Asia, Oceania and Latin America. population (16.5%) followed by Tamil Nadu.
2. The report cited data from NITI Aayog‘s inputs
 India has seen an estimated 1.2 million
and it is published by Ministry of Health and
snakebite deaths from 2000 to 2019, an
Family Welfare.
average of 58,000 per year.
Select the correct answer using the codes given below:
9. a
a. 1 only
Crew Health and Performance Exploration
Analog (CHAPEA) b. 2 only
 It is NASA‘s habitat in which the four crew c. Both 1 and 2
members will stay will be as Mars-realistic as
d. Neither 1 nor 2
possible.
 The results will provide scientific data that will
help in validating the systems that will be used 2) Which of the following is/are impacts or outcomes
for actual missions to Mars. of Swadeshi Movement?
 The habitat will simulate what it feels like to 1. Morley-Minto Reforms
carry out missions on Mars including resource 2. Rise of Extreme Nationalism
limitations, equipment failure, communication
delays and any other environmental stressors. 3. Revival of Indian Cottage Industry
 The crew will be expected to perform 4. Setup of National Council of Education
simulated spacewalks, scientific research and 5. Setup of Banaras Hindu University
use virtual reality and robotic controls and
exchange communications. Select the correct answer using the codes given below
10. a a. 1,2,3 & 4 only
Single Spotted Lanternfly b. 2,3, 4 & 5 only
 It has black spots on its wings, scarlet c. 3 & 5 only
underwings, yellow markings on its abdomen, d. All of the above
and tan semi-transparent forewings.
 It is an invasive insect that can cause damage
to native trees and agricultural crops. 3) Consider the following statements with respective
to PM-DAKSH Yojana
 It is native to Asia and feeds on agricultural
crops such as grapes, apples and hops as well 1. Under the scheme skill development training
as maple, walnut and willow trees. programmes are provided only for
marginalized persons of SC (Scheduled Caste),
 It is most commonly associated with "Tree of Denotified tribes.
Heaven" (Ailanthus altissima) plants which it
uses has its primary host. 2. It is implemented by National Scheduled
Castes Finance and Development Corporation
(NSFDC) and few other corporations under
Ministry of Skill Development and
Entrepreneurship.
Which of the statement(s) given above is/are correct?

www.shankariasacademy.com | www.iasparliament.com
37

a. 1 only  The report published cited findings from the


Technical Group on Population Projections for
b. 2 only
India and States 2011-2036 for its predictions.
c. Both 1 and 2
 India‘s elderly population (aged 60 and above)
d. Neither 1 nor 2 is projected to touch 194 million in 2031 from
138 million in 2021, a 41 per cent increase over
a decade.
4) Which of the following statement(s) is/ are correct
with respect to Mineral surveys in India  According to the report, Kerala currently has
the highest elderly population (16.5 per cent),
1. Geological Survey of India (GSI) is the nodal followed by Tamil Nadu (13.6 per cent),
agency to formulate mineral exploration Himachal Pradesh (13.1 per cent), Punjab (12.6
programmes. per cent) and Andhra Pradesh (12.4 per cent)
2. GSI publishes the National Mineral Inventory in 2021.
(NMI) of mineral resources once in five years.  Bihar, Uttar Pradesh and Assam have the least
3. The extraction of minerals depends on the proportion with 7.7 per cent, 8.1 per cent and
grant of the mineral concessions by respective 8.2 per cent, respectively.
State government.  The old-age dependency ratio is defined as the
Select the correct answer using the codes given below: number of persons aged 60+ per 100 persons
relative to the age group 15-59.
a. 1 and 2 only
2. a
b. 1 and 3 only
Swadeshi Movement
c. 2 and 3 only
 The movement had its roots in the anti-
d. All of the above partition movement which was started to
oppose Lord Curzon‘s decision of dividing the
province of Bengal.
5) Consider the following statements with respective
to Capital Expenditure  In August 1905, at Calcutta Townhall, a
massive meeting was held and the formal
1. It is a payment for goods or services recorded proclamation of the Swadeshi Movement was
or capitalized on the balance sheet instead of made.
expensed on the income statement.
 Impact of Swadeshi Movement
2. It creates employment especially for poor and
unskilled due to high multiplier effect.  Decline in Imports - It resulted in
significant decline in the foreign imports
Which of the statement(s) given above is/are correct?
during 1905-1908.
a. 1 only
 Growth of Extremism - Movement resulted
b. 2 only in growth of extreme nationalism amongst
youth which took to violence and wanted to
c. Both 1 and 2
bring an instant end to British dominance.
d. Neither 1 nor 2
 Morley-Minto Reforms - It forced British
dispensation to offer some concessions to
Indians in forms of Morley-Minto reforms in
Answers 1909.
1. a  Establishment of Swadeshi Institutions -
Elderly in India 2021 Report In August 1906, the National Council of
Education was set up to organise the national
 It is published by Ministry of Statistics and education system.
Programme Implementation.

www.shankariasacademy.com | www.iasparliament.com
38

 A Bengal Institute of Technology was set up for  Indian Bureau of Mines (IBM) an attached
technical education. office of Ministry of Mines collects the
exploration data from various agencies.
 Growth in Swadeshi Industries - It led to
establishments of swadeshi textile mills, soap  Indian Bureau of Mines (IBM) publishes the
and match factories, tanneries, banks, National Mineral Inventory (NMI) of mineral
insurance companies, shops, etc., also revived resources once in five years.
the Indian Cottage Industry.
 Under the Mines and Minerals (Development
Banaras Hindu University and Regulation) Amendment Act, 2015, for
major minerals, the mineral concession can be
 It was established jointly in 1916 by the
allotted through auctioning.
Maharaja of Darbhanga, Maharaja of Banaras
Prabhu Narayan Singh, Madan Mohan  The power to grant these mineral concessions
Malaviya, British Theosophist and Home Rule vests in the State Governments.
League founder Annie Besant.
 The extraction of minerals depends on the
3. b grant of the mineral concessions by the State
PM-DAKSH Yojana Government and the utilisation of minerals
depends on economic viability of the minerals.
 Under this, eligible target groups are provided
5. c
with the skill development training
programmes. Capital Expenditure
 Marginalized persons of SC (Scheduled Caste),  Capital expenditures (CapEx) are funds used
OBC (Other Backward Classes), Economically by to acquire, upgrade, and maintain physical
Backward Classes, Denotified tribes, assets such as property, plants, buildings,
Sanitation workers including waste pickers, technology, or equipment as well as
manual scavengers, transgender and other investment in shares.
similar categories are eligible under the
scheme.  It is important for companies / organization to
maintain existing property and equipment,
 It is implemented by the three Corporations and invest in new technology and other assets
under the Ministry of Skill Development and for growth.
Entrepreneurship.
 In view of a government, it creates
1. National Scheduled Castes Finance and employment especially for unskilled due to
Development Corporation (NSFDC), high multiplier effect.
2. National Backward Classes Finance &  It enhances the future productive capacity of
Development Corporation (NBCFDC), the economy and results in a higher rate of
3. National Safai Karamcharis Finance and economic growth.
Development Corporation (NSKFDC).  The multiplier effect refers to the effect on
4. b national income and product of an exogenous
increase in demand.
Mineral surveys in India
Government initiatives for Capital
 GSI carries out mapping and systematic Expenditure
exploration for various mineral commodities
based on the mineral potential following the  India experienced low growth rates for decades
guidelines as it failed to develop physical and social
infrastructure the key to achieving high
 United Nations Framework Classification economic growth.
(UNFC) and Mineral Evidence and Mineral
Content Rules (MEMC-2015) with an aim to  To address this union government launched
augmenting mineral resource. scheme of Financial assistance to states for
capital expenditure

www.shankariasacademy.com | www.iasparliament.com
39

 Under the scheme financial assistance is d. Neither 1 nor 2


provided to the state governments in the form
of 50 year interest free loan.
3) Which of the following statement(s) is/ are correct
 Funds provided to the states under the scheme with respect to National Edible Oil Mission-Oil Palm
by the government of India shall be used for (NMEO-OP)
new and ongoing capital project for long term
benefit to the state. 1. Under the scheme, oil palm farmers will be
provided financial assistance and will get
remuneration under a price and viability
11-08-2021 formula.

1) Global Youth Tobacco Survey (GYTS-4) was 2. The special emphasis of the scheme will be in
conducted in 2019 by the International Institute for Central India due to the conducive weather
Population Sciences (IIPS). Which of the following conditions in the regions.
statements is/are correct with respect to Tobacco Select the correct answer using the codes given below:
usage and cultivation in India?
a. 1 only
1. Arunachal Pradesh and Mizoram has the
highest use of tobacco among students. b. 2 only

2. The lowest use of tobacco among students in c. Both 1 and 2


Himachal Pradesh and Karnataka d. Neither 1 nor 2
3. Andhra Pradesh, Karnataka, and Gujarat are
the major producers of Tobacco in India.
4) Consider the following statements with respective
4. In the global scenario, Indian tobacco accounts to Marburg virus
for 10% of the area and 9% of the total
production. 1. It is a highly fatal virulent disease that causes
haemorrhagic fever, with bats as vectors.
Select the correct answer using the codes given below:
2. It belongs to the same family as the Nipha
a. 1 , 3, 4 only virus.
b. 2 & 4 only 3. There is no human to human transmission
c. 1,2, & 3 only reported so far.

d. All of the above Which of the statement(s) given above


is/are incorrect?
a. 1 only
2) Consider the following statements with respective
to Operation Greens b. 1 and 2 only

1. It is a price fixation scheme that allows only c. 2 and 3 only


Farmer Producer Organizations (FPO), d. All of the above
Cooperatives, Self-help groups to avail the
financial assistance.
2. It provides for short term intervention through 5) Consider the following statements with respective
value addition projects with Grant-in-aid at to Mission Innovation (MI)
35% to 70% of eligible project cost. 1. It is a global initiative of 24 countries and
Which of the statement(s) given above is/are correct? UNDP for catalysing energy trade.

a. 1 only 2. It aims to demonstrate clean energy


affordable, attractive and accessible for all.
b. 2 only
Which of the statement(s) given above is/are correct?
c. Both 1 and 2
a. 1 only

www.shankariasacademy.com | www.iasparliament.com
40

b. 2 only groups, food processors etc. can avail the


financial assistance under it.
c. Both 1 and 2
 The scheme provides for short term
d. Neither 1 nor 2
intervention by way of providing
transportation and storage subsidy @ 50%.
Answers  Where long term intervention is provided
through value addition projects in identified
1. d
 production clusters with Grant-in-aid @ 35%
Tobacco cultivation in India
to 70% of eligible project cost subject to
 Tobacco is a drought tolerant, hardy and short maximum of Rs. 50 crore per project.
duration crop which can be grown on soils
 Under the scheme, state-wise funds are not
where other crops cannot be cultivated
allocated as the scheme is demand driven and
profitably.
projects are approved as per scheme guidelines
 India is the 2nd largest producer and exporter on the basis of applications received for setting
after China and Brazil respectively. up of projects in eligible production clusters.

 The distinctive and positive features of Indian  As per budget Announcement 2021-22,
tobacco include the lower levels of heavy expanded operation greens scheme covers 22
metals, very low levels of Tobacco Specific perishables including shrimp.
Nitrosamines (TSNAs) and pesticide residues 3. a
compared to the other tobacco producing
countries in the world. National Edible Oil Mission-Oil Palm (NMEO-
OP)
Global Youth Tobacco Survey (GYTS-4)
 It is launched by Ministry of Agriculture &
 It is a self-administered, school-based survey Farmers' Welfare.
of students in grades associated with 13 to 15
years of age.  It will ensure that farmers get all facilities,
from quality seeds to technology to promote
 It is developed by World Health Organization farming to produce palm oil and other oil
(WHO), the Tobacco Free Initiative (TFI), the seeds.
United Nations Children‘s Fund (UNICEF) and
the Office on Smoking and Health at the  It is expected to incentivize production of palm
Centres for Disease Control and Prevention oil to reduce dependence on imports and help
(OSH-CDC). farmers cash in on the huge market.
 The current use of tobacco among students  It will involve raising the area under oil palm
across the States/ UTs ranged from the highest cultivation to 10 lakh hectares by 2025-26 and
in Arunachal Pradesh and Mizoram (58% 16.7 lakh hectares by 2029-30.
each) to the lowest in Himachal Pradesh
(1.1%).  The special emphasis of the scheme will be in
North East India, and A&N islands, due to the
2. d conducive weather conditions in the regions.
Operation Greens 4. c
 Ministry of Food Processing Industries Marburg virus
launched Operation Greens scheme, it focuses
on organized marketing of Tomatoes, Onions  It is in the same family as the virus that causes
and Potatoes (TOP vegetables) by connecting Ebola virus disease.
farmers with consumers.  Human infection with Marburg virus disease
 State Agriculture and other Marketing initially results from prolonged exposure to
Federations, Farmer Producer Organizations mines or caves inhabited by Rousettus bat
(FPO), cooperatives, companies, Self-help colonies.

www.shankariasacademy.com | www.iasparliament.com
41

 Marburg can spread through human-to-


human transmission via direct contact
2) With respect to Steel sector in India, consider the
(through broken skin or mucous membranes)
following statements:
with the blood, secretions, organs or other
bodily fluids of infected people, and with 1. Steel is a de-regulated sector and the role of
contaminated surfaces & materials. the Ministry of Steel is that of a facilitator.
 Two large outbreaks that occurred 2. The modernisation of steel plants is being
simultaneously in Marburg and Frankfurt in funded by the respective steel sector company
Germany, and in Belgrade, Serbia, in 1967, led from its internal resources / borrowings.
to the initial recognition of the disease. Which of the statement(s) given above is/are correct?
 There have been 12 major Marburg outbreaks a. 1 only
since 1967, mostly in southern and eastern
Africa. b. 2 only
5. b c. Both 1 and 2
Mission Innovation (MI) d. Neither 1 nor 2
 Mission Innovation (MI) is a global initiative
of 22 countries and the European Commission 3) Teram Shehr Glacier, sometimes seen in the news
(on behalf of the European Union). recently, is a tributary of?
 It was launched during 21st Conference of the a. Zemu Glacier
Parties (COP 21) UNFCCC in November 2015.
b. Milam Glacier
 It catalyses a decade of action and investment
in research, development and demonstration c. Siachen Glacier
to make clean energy affordable, attractive and d. Chhota Shigri Glacier
accessible for all.
 India is a member of this mission.
4) With respect to the indigenous aircraft carrier (IAC-
 Department of Biotechnology is the nodal 1), Vikrant, consider the following statements:
agency for coordinating national efforts in
clean energy R&D under the mission. 1. It is India‘s first indigenous aircraft carrier
with the largest and most complex platform so
far designed by the Directorate of Naval
Design.
12-08-2021
2. It is being propelled by nuclear reactors which
1) With reference to Quality of Life for Elderly Index, helps it to operate at high speed for long
which of the following statements is correct? periods of time.
a. The Index has been created by the NITI Aayog Which of the statement(s) given above is/are correct?
at the request of Ministry of Social Justice
Department. a. 1 only
b. The Index framework includes four pillars b. 2 only
namely Financial Well-being, Social Well-being, c. Both 1 and 2
Health System and Job Security.
d. Neither 1 nor 2
c. According to the index, Puducherry and Ladakh
are top-scoring regions in Aged and Relatively
Aged States, respectively.
5) With reference to the term ―Junk DNA‖, consider
d. Aged States refer to States with an elderly the following statements:
population of more than 5 million, whereas
1. It refers to the regions of DNA that does not
Relatively Aged States refer to less than 5 million.
encode protein sequences.

www.shankariasacademy.com | www.iasparliament.com
42

2. The proportion of coding versus noncoding 8) Which of the following are reasons behind Wild
DNA varies significantly between species. Fires in India?
Which of the statement(s) given above is/are correct? 1. Hot and Dry seasons
a. 1 only 2. Lightning strikes
b. 2 only 3. Lack of soil moisture
c. Both 1 and 2 4. Stubble Burning
d. Neither 1 nor 2 Select the correct answer using the codes given below:
a. 1 & 3 only
6) NCPCR Survey found out that 62.5% of students in b. 1 & 2 only
minority schools belonged to non-minority
c. 1, 2 & 3 only
communities. Which of the following statements is/are
correct with respect to Minority Educational d. All of the above
Institutions
1. Minority institutions cannot ignore the
regulations recommended by the state while 9) Consider the following statements with respective
establishing educational institutions of their to Kurumba & Irula Tribes
choice. 1. Ministry of Tribal Affairs has launched
2. According to SC, Right to Education Act is ―Namath Basai‖ program to teach mother
entirely inapplicable to minority schools. tongue to these tribes.

Select the correct answer using the codes given below: 2. Irula tribes worship Vishnu under the name of
Rangaswami and Siva without differentiation.
a. 1 only
Which of the statement(s) given above
b. 2 only is/are incorrect?
c. Both 1 and 2 a. 1 only
d. Neither 1 nor 2 b. 2 only
c. Both 1 and 2
7) Consider the following statements with respective d. Neither 1 nor 2
to Carbon Sinks
1. Mangroves, seagrass beds and salt marshes
store at least ten times more carbon than 10) Consider the following statements with respective
continental forests. to Rana Punja Bhil

2. According to recent findings, Amazon 1. He was a local chieftain lived in 16th century
rainforest, being the best carbon sink in the and was a contemporary of Akbar.
world, absorbs more carbon than it releases. 2. He participated in the Battle of Haldighati and
Which of the statement(s) given above supported the forces led by Man Singh I of
is/are incorrect? Amber

a. 1 only Which of the statement(s) given above is/are correct?

b. 2 only a. 1 only

c. Both 1 and 2 b. 2 only

d. Neither 1 nor 2 c. Both 1 and 2


d. Neither 1 nor 2

www.shankariasacademy.com | www.iasparliament.com
43

Answers 2. c
1. d Steel Sector in India
Quality of Life for Elderly Index  Steel is a de-regulated sector and the role of
the Ministry of Steel is that of a facilitator.
 It was released recently by Dr Bibek Debroy,
Chairman, Economic Advisory Council to the  Decision regarding modernization of steel
Prime Minister (EAC-PM). plant is taken by individual company based on
techno-commercial considerations.
 The Index has been created by the Institute for
Competitiveness at the request of EAC-PM.  Government of India has not allocated any
funds for modernisation of steel plants.
 The report identifies the regional patterns of
ageing across Indian States and assesses the  The modernisation of steel plants is funded by
overall ageing situation in India. the respective steel sector company from its
internal resources / borrowings.
 The Index framework includes four pillars and
eight sub-pillars. 3. c
Main Pillars of the index  An expedition to simultaneously scale five
virgin peaks located in the Teram Shehr
1. Financial Well-being Glacier, near Siachen Glacier was flagged off
2. Social Well-being recently.
3. Health System  Teram Shehr Glacier is a tributary of Siachen
Glacier.
4. Income Security
4. a
Key Highlights from the Report
INS VIKRANT
 The Health System pillar observes the highest
national average, 66.97 at an all-India level,  India‘s first indigenous aircraft carrier (IAC-1),
followed by 62.34 in Social Well-being. Vikrant, set out to sea for its maiden set of
trials.
 Financial Well-being observes a score of 44.7,
which is lowered by the low performance of 21  The vessel to be named, Vikrant, after the first
States across the Education Attainment & aircraft carrier operated by the Indian Navy.
Employment pillar, which showcases scope for
 It is the largest and the most complex platform
improvement.
so far designed by the Directorate of Naval
 States have performed particularly worse in Design, and is slated to join the Navy next
the Income Security pillar because over half of year.
the States have a score below the national
 The IAC-1 is an advanced platform in
average, i.e., 33.03 in Income Security, which
comparison with Vikramaditya, the only
is the lowest across all pillars.
aircraft carrier in service with the Indian Navy,
 Rajasthan and Himachal Pradesh are top- in terms of capabilities, automation and net-
scoring regions in Aged and Relatively Aged centricity.
States, respectively.
 Unlike Vikramaditya, which is steam-
 Chandigarh and Mizoram are top-scoring propelled, Vikrant is propelled by four gas
regions in Union Territory and North-East turbines.
States category.
 Its diesel alternators, eight of them, generate
 The Aged States refer to States with an elderly as much as 24 MW power, which is enough to
population of more than 5 million, whereas light up an entire city.
Relatively Aged States refer to States with an 5. c
Elderly population of less than 5 million.
Junk DNA

www.shankariasacademy.com | www.iasparliament.com
44

 In genetics, the term junk DNA refers to  Carbon sinks are natural systems that suck up
regions of DNA that are noncoding. and store carbon dioxide from the atmosphere.
 DNA contains instructions (coding) that are  Nature provided trees, the oceans, earth and
used to create proteins in the cell. the animals themselves as carbon sinks, or
sponges.
 However, the amount of DNA contained inside
each cell is vast and not all of the genetic  Ocean sequestration, geological sequestration
sequences present within a DNA molecule are artificial carbon sinks
actually code for a protein.
 Sedimentary rocks in the Earth‘s crust contain
 Some of this noncoding DNA is used to loads of carbon compounds, including the
produce non-coding RNA components such as hydrocarbons we use as fossil fuels that leads
transfer RNA, regulatory RNA and ribosomal to excess carbon dioxide in our atmosphere.
RNA.
 A forest is considered a carbon sink if it
 However, other DNA regions are not absorbs more carbon than it releases.
transcribed into proteins, nor are they used to
produce RNA molecules and their function is  According to recent findings Amazon
unknown. rainforest is now releasing instead of
absorbing carbon dioxide.
 The proportion of coding versus noncoding
8. d
DNA varies significantly between species.
Reasons behind Forest Fires
 In the human genome for example, almost all
(98%) of the DNA is noncoding, while in  Fires of longer duration, increasing intensity,
bacteria, only 2% of the genetic material does higher frequency and highly inflammable
not code for anything. nature are all being linked to climate change.
6. c  In Uttarakhand, the lack of soil
Minority Educational Institutions moisture too is being seen as a key factor.

 Minority institutions have the fundamental  Since dry leaves, shrubs, grass and deadwood
right under Article 30 of the Constitution to are easily combustible, they are easy to ignite.
establish and administer their educational  In India highest incidents of forest fires used
institutions according to their choice. to occur in the months of May and June but
 However, they cannot ignore the regulations since the past few years one part of the state or
recommended by the state. the other is affected almost all the year round
by wildfires.
 Further, the Supreme Court in the TMA Pai
Foundation case, 2002 said that Article 30(1)  Ignition can either happen naturally, such as
was neither absolute nor above the law. through lightning strikes, or triggered
accidentally, such as from cigarette stubs.
 Minority schools are outside the purview of the
RTE Act, Further, in 2014, the Supreme Court  Extreme heat can increase drought, and hot
in Pramati judgment made the whole RTE Act dry conditions can in turn create wildfire.
inapplicable to minority schools.  Authorities in Uttarakhand and Nepal have
 National Commission for Protection of Child said several forest fires have been caused by
Rights (NCPCR) conducted a survey, which stubble burning in agricultural lands adjoining
highlighted that as many schools and forests.
institutions have registered as minority 9. b
institutions, simply because they don‘t have to
implement RTE. Kurumba Tribe

7. b  Popularly known as Mala Pulayans, Hill


Pulayans and PambaPulayans.
Carbon Sinks

www.shankariasacademy.com | www.iasparliament.com
45

 Their traditional occupation includes foraging MaharanaPratap, and the Mughal emperor
and shifting cultivation Akbar's forces, led by Man Singh I of Amber.
 They Collect forest products like honey, wax,  The Mughals were the victors and inflicted
soapnut, turmeric, ginger and wild significant casualties among the Mewaris but
cardamoms. failed to capture Pratap, who escaped.
 Their religion includes Animism (belief in  When MaharanaPratap was readying for the
spirituality of objects, places, and creatures) battle with Akbar, the tribal Bhil community
and Totemism (Worship of any species of voluntarily came to his assistance and at the
plants or animals thought to possess time the Bhil army was commanded by Punja.
supernatural powers)
 Owing to his status as a commander, he was
Irula Tribe bestowed the title of Rana.
 They are Agriculturists, who grow Paddy, Ragi,
dhal, plantains, chillies and turmeric
13-08-2021
 Their majority worship is Vishnu under the
name of Rangaswami and Siva without 1) With reference to Anti-dumping duty, consider the
differentiation. following statements:
1. It is a measure to protect domestic industry
 Some practice animism and worship the tiger.
and act as an instrument to establish fair trade.
Namath Basai Scheme
2. Import of cheap products through illegal trade
 Kerala State government is carrying out a channels like smuggling falls within the
unique programme called ―Namath Basai‖ of purview of dumping.
teaching tribal children in their mother 3. The use of anti-dumping measure as an
tongue. instrument of fair competition is permitted by
 The programme is being implemented by the the World Trade Organization (WTO).
SamagraShiksha Kerala (SSK). Which of the statements given above is/are correct?
 SSK is an overarching programme for the a. 3 only
school education sector extending from pre-
school to class 12. b. 1 and 2 only

 It offers pre-recorded classes through a c. 1 and 3 only


YouTube channel in three tribal languages in d. 1, 2 and 3
Attappady valley in Palakkad District Kerala.
 These languages belong to the Irula, Muduka
and Kurumba tribes. 2) Which of the following is the nodal agency for
landslide studies in India?
 Classes are offered in the Oorali, Muthuvan
and Paniya languages in Idukki. a. Geological Survey of India

10. a b. National Disaster Management Authority

Rana Punja Bhil c. National Institute of Public Finance and Policy


d. Building Materials and Technology Promotion
 Rana Punja Bhil was a contemporary of 16th
Council
century ruler of Mewar, MaharanaPratap.
 He is considered to be a significant character
who bolstered the strength of Pratap during 3) Consider the following statements:
his battles with Mughal emperor Akbar.
1. Lion and Tiger surveys are usually held once in
 The Battle of Haldighati was a battle fought on four years whereas elephants are counted once
18 June 1576 between cavalry and archers in five years.
supporting the Rana of Mewar,

www.shankariasacademy.com | www.iasparliament.com
46

2. In India, 90% of the area occupied by 7) Consider the following statements with respective
elephants and tigers is common. to Biomethanation
Which of the statements given above is/are correct? 1. Fermenting bacteria, Organic acid oxidizing
bacteria, and Methanogenic archaea are group
a. 1 only
of microorganisms involved in the process.
b. 2 only
2. Ministry of Environment, Forest and Climate
c. Both 1 and 2 change has launched a loan interest
subvention scheme for biomethanation
d. Neither 1 nor 2 projects.
Which of the statement(s) given above
4) Ningbo Port, often seen in the news recently, is is/are incorrect?
situated in? a. 1 only
a. Sea of Japan b. 2 only
b. East China Sea c. Both 1 and 2
c. South China Sea d. Neither 1 nor 2
d. Strait of Malacca

8) About 32 % of the Indian coastline is under varying


5) With reference to Asian Elephants, which of the degrees of erosion (low, moderate or high), which of
following statements is incorrect? the following factors affect the rate of Coastline
Erosion?
a. More than 90% of the world‘s elephant
population is in India. 1. Nature of the seafloor.
b. Elephants have a gestation period of 22 months. 2. Resource extraction in shore areas
c. Elephant calves are able to walk within one to 3. Power of the waves crossing the beach.
two hours of birth. 4. Vegetation in shores.
d. Asian elephants are listed as ―Endangered‖ on Select the correct answer using the codes given below:
the IUCN Red List of threatened species.
a. 1 and 3 only
b. 2 and 4 only
6) The richest 10% depleted the global carbon budget
by 31% and the poorest 50% used only 4% of the c. 1, 2 and 3 only
carbon budget, which of the following factors
d. 1, 2, 3 and 4
impact Carbon Budget?
1. Effect of non-CO2 greenhouse gases (GHG)
9) Consider the following statements with respective
2. Cooling effect of Aerosols
to Dara Shikoh
3. Tropical cyclones
1. He was designated with the title Padshahzada-
4. Earth-system feedbacks i-Buzurg Martaba (Prince of High Rank).
Select the correct answer using the codes given below: 2. He authored a work which argues for the
harmony of Sufi philosophy in Islam and
a. 1 and 3 only Vedanta philosophy in Hinduism.
b. 2 and 3 only Which of the statement(s) given above is/are correct?
c. 1, 2 and 4 only a. 1 only
d. 1, 2, 3 and 4 b. 2 only
c. Both 1 and 2

www.shankariasacademy.com | www.iasparliament.com
47

d. Neither 1 nor 2 downstream sectors — weaving and


garmenting.
 Indian textile units will be able to export
10) Consider the following statements with respective
viscose products at internationally competitive
to Asiatic Lion
prices and produce more of viscose-based
1. Asiatic lions in Gir wildlife sanctuary has low products.
genetic diversity, making it vulnerable to
threats of extension from epidemics.  Viscose is largely used by the textile and
clothing industry to make apparel, mainly for
2. Ministry of Environment, Forests and Climate women and children, in the domestic market.
Change (MoEFCC) created Project Lion to
conserve Asiatic lions in Gir forest. 2. a

Which of the statement(s) given above is/are correct?  The Geological Survey of India (GSI), an
attached office under the Ministry of Mines
a. 1 only (MoM), is the nodal agency for landslide
b. 2 only studies in India providing quality geo-scientific
information in order to minimize loss of life
c. Both 1 and 2 and damage to property from landslide
d. Neither 1 nor 2 hazards.
 The Landslide Hazard Zonation Map of India
marks over 70% of the State as ‗high risk‘ and
Answers 14% as ‗severe‘ to ‗very high risk‘.
1. a
Anti-Dumping Duty
 Dumping is said to occur when the goods are
exported by a country to another country at a
price lower than its normal value.
 This is an unfair trade practice which can have
a distortive effect on international trade.
 Anti-dumping is a measure to rectify the
situation arising out of the dumping of goods
and its trade distortive effect.
 In fact, anti-dumping is an instrument for
ensuring fair trade and is not a measure of
protection per se for the domestic industry.
 It provides relief to the domestic industry
against the injury caused by dumping.
 The use of anti-dumping measure as an
instrument of fair competition is permitted by
the WTO.
Recent Developments
 The Union government recently revoked anti-
dumping duty (ADD) on viscose staple fibre
originating in or imported from China and 3. b
Indonesia.
 India is planning to move to a system that will
 As an impact, the fibre will be available in count tigers and elephants as part of a
India at competitive prices and thus benefit common survey.

www.shankariasacademy.com | www.iasparliament.com
48

 Given that 90% of the area occupied by  Elephants have a gestation period of 22
elephants and tigers is common, and once months.
estimation methods are standardised, having a
common survey can significantly save costs.  Elephant calves are able to walk within one or
two hours of birth.
 The tiger survey is usually held once in four
years and elephants and lions are counted once  Asian elephants are listed as ―Endangered‖ on
in five years. the IUCN Red List of threatened species.

 According to the most recent 2018-19 survey,  This has been done as most of the range States
there were 2,997 tigers in India. except India have lost their viable elephant
populations due to loss of habitat, poaching,
 According to the last count in 2017, there were etc.
29,964 elephants in India.
 Current population estimates indicate that
4. b there are about 50,000-60,000 Asian
elephants in the world.
 China has partially shut down the world‘s
third-busiest container port after a worker  More than 60% of the world‘s elephant
there tested positive for COVID-19. population is in India.
 The Meishan terminal at Ningbo-Zhoushan 6. c
port, which is south of Shanghai, accounts for
Carbon Budgets
over a fourth of the container cargo handled at
the Chinese port.  A carbon budget is a cumulative amount of
carbon dioxide (CO2) emissions permitted
 The closure could potentially threaten global
supply chains and impact maritime trade.  over a period of time to keep within a certain
temperature threshold.
 The port is located on the coast of the East
China Sea.  The carbon budget is the amount of CO2 that
humanity can emit while still having a chance
to contain global warming within 1.5 degrees
centigrade compared with preindustrial levels,
as advocated by the Paris Agreement.
 Carbon budgets are complex estimates and are
typically subject to several uncertainties such
as,
1. The effect of non-CO2 greenhouse gases
(GHG) [methane and nitrous oxide].
2. The cooling effect of aerosols.
3. Earth system feedbacks such as thawing of
permafrost would release additional carbon,
and which were not considered in
methodology earlier, now it‘s been added.
7. b
Biomethanation
 It is a process by which organic material is
microbiologically converted under anaerobic
conditions to biogas.
5. a
 Three main physiological groups of
 Elephas maximus is the scientific name of microorganisms are involved: fermenting
Asian elephants.

www.shankariasacademy.com | www.iasparliament.com
49

bacteria, organic acid oxidizing bacteria, and  The study was done along nine coastal states
methanogenic archaea. and two Union territories (UT) to provide
information for coastal management strategy.
 Biomethanation has strong potential for the
production of energy from organic residues  Around 41 per cent of the coastline of Kerala is
and wastes. experiencing erosion, 31 per cent is stable and
21 per cent is accreting, as per the National
 It will help to reduce the use of fossil fuels and
Centre of Coastal Research studies.
thus reduce CO(2) emission.
9. c
 The Ministry of New and Renewable Energy
(MNRE) has launched a loan interest Dara Shikoh
subvention scheme in association with United
 Dara Shikoh was Mughal emperor Shah
Nations Industrial Development Organization
Jahan‘s son, was an expected heir of Mughal
(UNIDO) and Global Environment Facility
rule.
(GEF) to provide financial assistance for
innovative waste to energy bio-methanation  In the war of succession which ensued after
projects. Shah Jahan‘s illness in 1657, Dara was
8. d defeated by his younger brother Prince
Muhiuddin (Aurangzeb).
Coastline Erosion
 Dara was a liberal-minded unorthodox Muslim
 Coastline erosion is the loss or displacement of as opposed to the orthodox Aurangzeb.
land, or the long-term removal of sediment
and rocks along the coastline due to the action  He authored the work ―The Confluence of the
of waves, currents, tides, wind-driven water, Two Seas‖, which argues for the harmony of
waterborne ice, or other impacts of storms. Sufi philosophy in Islam and Vedanta
philosophy in Hinduism.
 The ability of waves to cause erosion of the cliff
face depends on many factors such as, 10. a
Asiatic Lion
 Primary Factors
1. The hardness of sea-facing rocks  The population in Gir has low genetic
diversity, making it vulnerable to threats of
2. Power of the waves crossing the beach. extension from epidemics.
3. Configuration of the seafloor.  It is listed as ‗Endangered‘ under the IUCN
4. Rising sea levels globally. Red List.

 Secondary Factors  Its population is restricted to the state of


Gujarat in India (Gir National Park).
1. Weathering
 The Wildlife Institute of India, along with the
2. Slope hydrology Gujarat Forest Department, had created a
3. Vegetation Project Lion.

4. Cliff foot sediment accumulation & erosion  The project will involve habitat development
by engaging modern technologies in
5. Resource extraction in shore areas management as well as in addressing the issue
Mapping for the Indian coast of diseases in lion.

 The National Centre for Coastal Research has  Under Project Lion following sanctuaries are
carried out a national shoreline change identified for conservation of Lions
assessment mapping for the Indian coast, 1. Kuno-Palpur Wildlife Sanctuary, Madhya
using 28 years of satellite data from 1990- Pradesh.
2018.
2. Madhav National Park, Madhya Pradesh.
3. Sitamata Wildlife Sanctuary, Rajasthan.

www.shankariasacademy.com | www.iasparliament.com
50

4. Mukundra Hills Tiger Reserve, Rajasthan. scrubland, which helps other mammals to co-
exist.
5. Gandhi Sagar Wildlife Sanctuary, Madhya
Pradesh. Which of the statement(s) given above is/are correct?
6. Kumbhalgarh Wildlife Sanctuary, Rajasthan. a. 1 only
7. Jessore-Balaram Ambaji WLS and adjoining b. 2 only
landscape, Gujarat.
c. Both 1 and 2
d. Neither 1 nor 2
14-08-2021
1) Which of the following site(s) has been recently 4) With respect to the Constitution (One Hundred and
included in the Ramsar Convention as Wetlands of Twenty Seventh Amendment) Bill, 2021, consider the
International Importance? following statements:
1. Wadhvana Wetland 1. The Bill empowers the states and union
2. Sultanpur National Park territories to prepare their own list of socially
and educationally backward classes.
3. Thane Creek Flamingo Sanctuary
2. This list must be made by law, and may differ
Select the correct answer using the codes given below: from the central list.
a. 1 only Which of the given statements above is/are correct?
b. 1 and 2 only a. 1 only
c. 2 and 3 only b. 2 only
d. 1, 2 and 3 c. Both 1 and 2
d. Neither 1 nor 2
2) Consider the following statements with respect to
Bhindawas Wildlife Sanctuary
5) Operation Blue Freedom, sometimes seen in the
1. It is the largest natural freshwater wetland news recently, is associated with?
located in the Indian State of Haryana.
a. India‘s scientific expedition program to
2. The site supports globally threatened species Antarctica under the Mission Sakthi
including the endangered Egyptian Vulture
and the Steppe Eagle. b. An expedition exercise till Siachen Glacier by
People with disabilities from across the country
Which of the statement(s) given above is/are correct?
c. A campaign launched by the Ministry of
a. 1 only Environment to attain blue flag tag for India‘s
b. 2 only beaches

c. Both 1 and 2 d. An exercise launched by the Ministry of


Fisheries to eliminate ghostnets and bottom
d. Neither 1 nor 2 trawling fishing

3) Consider the following statements with respect to 6) Factors like guaranteed sales of sugarcane and
Thol Lake Wildlife Sanctuary public distribution of sugar have helped India become
1. It lies on the East Asian-Australasian Flyway in the second-largest producer of sugar worldwide. Which
the Indian State of Gujarat. of the following statements are correct with respect to
Sugar production in India?
2. The wetland is an open water habitat
surrounded by cropland, fallow land and

www.shankariasacademy.com | www.iasparliament.com
51

1. North India produces sugar canes with higher a. 1 only


sucrose content and higher yield per unit area
b. 2 only
due to nutrient rich soil.
c. Both 1 and 2
2. About 60% of the sugar production comes
from Maharashtra and Uttar Pradesh. d. Neither 1 nor 2
3. South India‘s co-operative sugar mills are
better managed when compared with north
India. 9) Consider the following statements with respective to
Taxation in India
Select the correct answer using the codes given below:
1. Constitution makes it clear that no one has the
a. 1 and 2 only right to levy or charge taxes except by the
authority of law.
b. 2 and 3 only
2. The State's sovereign right to tax is not
c. 1 and 3 only
absolute under international law.
d. 1, 2 and 3
Which of the statement(s) given above is/are correct?
a. 1 only
7) Consider the following statements with respective to
b. 2 only
Taxation Laws (Amendment) Bill, 2021
c. Both 1 and 2
1. It will nullify IT Act to impose tax liability on
the income earned from the sale of shares of a d. Neither 1 nor 2
foreign company on a retrospective basis.
2. It aims to refund retrospective taxes on
offshore transactions involving Indian assets 10) Consider the following statements with respective
without any interest. to Gol Gumbaz

Which of the statement(s) given above is/are correct? 1. It has a biggest dome structure in India which
stands unsupported by pillars.
a. 1 only
2. It is the tomb Yusuf Adil Sha, the founder Adil
b. 2 only Shahi dynasty constructed in 1630.
c. Both 1 and 2 Which of the statement(s) given above is/are correct?
d. Neither 1 nor 2 a. 1 only
b. 2 only
However offshore transaction involving Indian assets c. Both 1 and 2
executed after 28th may 2012 are still taxable as there
is no retrospective application of the law. d. Neither 1 nor 2

8) The amount of ocean warming observed since 1971


will likely at least double by 2100, which of the Answers
following statements are incorrect with respect to
Outcomes of Ocean warming? 1. b
1. It can create both zones where waters that Wetlands of International Importance (Ramsar
have no dissolved oxygen and zones with low Sites)
oxygen concentration.
 Four more wetlands from India get recognition
2. It can decrease the range and increase diseases from the Ramsar Secretariat as Ramsar sites
in commonly eaten fish sea fishes like recently.
sardines, pilchards, and herring.
The sites are:
Select the correct answer using the codes given below:
1. Wadhwana Wetland (Gujarat)

www.shankariasacademy.com | www.iasparliament.com
52

2. Thol Lake Wildlife Sanctuary (Gujarat) Egyptian Vulture, Steppe Eagle, Pallas‘s Fish
Eagle, and Black-bellied Tern.
3. Sultanpur National Park (Haryana)
3. b
4. Bhindawas Wildlife Sanctuary (Haryana)
Thol Lake Wildlife Sanctuary
Sultanpur National Park
 It is located in the State of Gujarat.
 It is located in the state of Haryana and
supports more than 220 species of resident,  It lies on the Central Asian Flyway and more
winter migratory and local migratory water than 320 bird species can be found here.
birds at critical stages of their life cycles.
 The wetland supports more 30 threatened
 More than ten of these are globally threatened, waterbird species, such as the critically
including the critically endangered sociable endangered White-rumped Vulture and
lapwing, and the endangered Egyptian Vulture, Sociable Lapwing, and the vulnerable Sarus
Saker Falcon, Pallas‘s Fish Eagle and Black- Crane, Common Pochard and Lesser White-
bellied Tern. fronted Goose.
Wadhvana Wetland 4. c
 It is located in the state of Gujarat. The Constitution (One Hundred and Twenty
Seventh Amendment) 2021 Bill
 It is internationally important for its birdlife as
it provides wintering ground to migratory  The bill amends this to provide that
water birds, including over 80 species that the President may notify the list of socially
migrate on the Central Asian Flyway. and educationally backward classes only for
purposes of the central government.
 They include some threatened or near-
threatened species such as the endangered  This central list will be prepared and
Pallas‘s fish-Eagle, the vulnerable Common maintained by the central government.
Pochard, and the near-threatened Dalmatian
Pelican, Grey-headed Fish-eagle and  The Bill enables states and union
Ferruginous Duck. territories to prepare their own list of
socially and educationally backward
Thane Creek classes.
 The biodiversity-rich Thane Creek, which is  This list must be made by law, and may
one of the largest creeks in Asia receiving huge differ from the central list.
congregation of birds, has been proposed for
Ramsar site designation. 5. b

At present, Maharashtra has two Ramsar sites Operation Blue Freedom

1. Nandur Madhmeshwar in Nashik  Recently, the Government of India has


permitted a team of people with disabilities to
2. Lonar crater in Buldhana district scale Siachen Glacier.
2. b  Operation Blue Freedom is an expedition till
Bhindawas Wildlife Sanctuary Siachen Glacier undertaken by the People with
disabilities from across the country to create a
 It is the largest wetland in Haryana. new World Record for the largest team of
people with disabilities to reach the world‘s
 It is a human-made freshwater wetland.
highest battlefield.
 Over 250 bird species use the sanctuary 6. b
throughout the year as a resting and roosting
site. Sugar production in India
 The site supports more than ten globally  Sugar industry is broadly distributed over two
threatened species including the endangered major areas of production- Uttar Pradesh,
Bihar, Haryana and Punjab in the north and

www.shankariasacademy.com | www.iasparliament.com
53

Maharashtra, Karnataka, Tamil Nadu and  The latest Intergovernmental Panel on Climate
Andhra Pradesh in the south. Change (IPCC) report has warned that ocean
warming will continue over the 21st century
 South India has tropical climate which is
and is likely to continue until at least the year
suitable for higher sucrose content giving
2300 even if we minimise carbon emissions.
higher yield per unit area as compared to north
India.  Ocean warming can help create both anoxic
(waters that have no dissolved oxygen) and
 The crushing season is also much longer in the
hypoxic (low oxygen concentration) zones.
south than in the north.
 The report adds that these oxygen-deficient
 For example, crushing season is of nearly four
areas are expected to persist for thousands of
months only in the north from November to
years.
February, whereas it is of nearly 7-8 months in
the south where it starts in October and  Ocean warming and acidification may drag
continues till May and June. down the commercial Arctic cod fishery by
2100.
 The co-operative sugar mills are better
managed in the south than in the north.  Several species were noted to migrate
poleward or to deeper waters to stay in their
 Most of the mills in the south are new which
ideal temperature range.
are equipped with modern machinery.
 Fish like sardines, pilchards and herring will
 Over half of sugar mills are located in
become smaller in size and not be able to move
Maharashtra and Uttar Pradesh and about 60
to better environments.
per cent of the production comes from these
two states. 9. c
7. a Taxation in India
Taxation Laws (Amendment) Bill, 2021  Constitution gives the government the right to
levy taxes on individuals and organisations.
 The Bill amends the Income Tax Act, 1961 (IT
Act) and the Finance Act, 2012.  Any tax being charged has to be backed by a
law passed by the legislature or Parliament
 The 2012 act, aimed to check practice of Indian
(Article 265).
business held under entities incorporated
abroad changing hands without involving a  Taxes in India come under a three-tier system
capital gains tax liability in India and the Act based on the Central, State and local
was to be applicable retrospectively. governments, and the Seventh Schedule of the
Constitution puts separate heads of taxation
 The Bill proposes to nullify this retrospective
under the Union and State list.
basis for taxation.
 There is no separate head under the
 Features of the bill are as follows
Concurrent list, meaning Union and the States
1. No tax demand shall be raised in future on the have no concurrent power of taxation.
basis of the said retrospective amendment for
 India has a sovereign right to tax, however
any indirect transfer of Indian Assets , if the
sovereign right is subject to certain limitations,
transaction was undertaken before 28th may
and under international law the sovereign
2012
right to tax is not absolute.
2. Demand raised for indirect transfer of Indian
Assets made before 28th may 2012 shall be  Recently, the Government of India introduced
nullified on fulfilment of specified conditions. The Taxation Laws (Amendment) Bill, 2021 in
the Lok Sabha which seeks to withdraw tax
3. It has also proposed to refund the amount paid demands made using a 2012 retrospective
in these cases without any interest thereon. legislation to tax the indirect transfer of Indian
8. d assets.

Outcomes of Ocean warming

www.shankariasacademy.com | www.iasparliament.com
54

 The government has stressed the need to 1. It works as a standard Massive Open Online
establish its sovereign right to taxation. Course (MOOC) platform and offers access to
various courses in the field of social defence.
10. a
2. It is an initiative of Department of School
Gol Gumbaz Education and Literacy, Ministry of Education.
 Gol Gumbaz is the tomb of Mohammed Adil Which of the statement(s) given above is/are
Shah, the 7th ruler of Adil Shahi dynasty. incorrect?
 It was commissioned by Mohammed Adil shah a. 1 only
in 1626 and completed in 1656, its architect
was Yaqut of Dabul. b. 2 only

 It has biggest domes in India, with a diameter c. Both 1 and 2


of 44 metres. d. Neither 1 nor 2
 The amazing feature of this dome is that it
stands unsupported by pillars.
4) Consider the following statements with respect to
Jaipur Blue Pottery
16-08-2021 1. It is an indigenous pottery technique native to
the states of Rajasthan and Gujarat.
1) ‗IndiGau‘, sometimes seen in the news recently,
refers to? 2. It was conferred the Geographical Indication
Status in 2008.
a. Indigenously developed vaccine for cattles
3. It is one of the few pottery techniques in the
b. Cattle genomic chip for preserving indigenous world that does not use clay.
breeds
Which of the statements given above are correct?
c. Animal health card issued under the National
Mission for Bovine productivity a. 1 and 2 only
d. A web portal to boost dairy productivity in India b. 1 and 3 only
by organizing the livestock market
c. 2 and 3 only
d. 1, 2 and 3
2) Match the following:
Indigeneous breeds – States
5) With respect to Jajmani System, consider the
1. Ongole – a. Punjab following statements:
2. Kangayam – b. Gujarat 1. It is a share cropping system followed by
Jotedars, where sharecroppers handed over
3. Sahiwal – c. Tamil Nadu half of the crops to the Jotedars, post
4. Kankrej – d. Andhra Pradesh harvesting.
Select the correct answer using the codes given below: 2. Ganadevata, a novel written by Tarashankar
Bando-padhyay, portrayed the decline of the
a. 1-c; 2-d; 3-b; 4-a Jajmani system.
b. 1-d; 2-c; 3-b; 4-a Which of the statement(s) given above is/are correct?
c. 1-a; 2-b; 3-c; 4-d a. 1 only
d. 1-d; 2-c; 3-a; 4-b b. 2 only
c. Both 1 and 2
3) With respect to TAPAS Portal, consider the d. Neither 1 nor 2
following statements:

www.shankariasacademy.com | www.iasparliament.com
55

6) Which of the following statements are correct with b. 2 only


respective to Bioremediation?
c. Both 1 and 2
1. It involve oxidation-reduction reactions where
d. Neither 1 nor 2
oxygen is added to stimulate oxidation of a
reduced pollutant (e.g. hydrocarbons).
2. Paraperlucidibaca, Cycloclasticus are some 9) Consider the following statements with respective to
bacteria that can help remove several classes of Greater Adjutant Stroke
contaminants from oil spills.
1. They are listed Endangered under IUCN Red
Select the correct answer using the codes given below: list and Schedule IV under Wildlife
(Protection) Act 1972.
a. 1 only
2. Their breeding grounds are found in Odisha,
b. 2 only
West Bengal & Uttar Pradesh.
c. Both 1 and 2
Which of the statement(s) given above is/are correct?
d. Neither 1 nor 2
a. 1 only
b. 2 only
7) Consider the following statements with respective to
c. Both 1 and 2
Socio Economic Caste Census
d. Neither 1 nor 2
1. All the personal information collected under
SECC is open for use by Government
departments.
10) Consider the following statements with respective
2. It is conducted under the overall coordination to Post Devolution Revenue Deficit Grant
of Office of the Registrar General and Census
Commissioner under Ministry of Home 1. It is provided by the union government to the
Affairs. States under Article 275 of the Constitution.

Which of the statement(s) given above is/are 2. It is released in monthly instalments as per the
incorrect? recommendations of the Finance Commission.

a. 1 only 3. It is paid out of the Consolidated Fund of India


in each year, and different sums may be fixed
b. 2 only for different States.
c. Both 1 and 2 Which of the statement(s) given above is/are correct?
d. Neither 1 nor 2 a. 1 and 2 only
b. 1 and 3 only
8) Out of 9.46 million tonnes of plastic waste c. 2 and 3 only
generated every year in our country, 43% is single use
plastic. Which of the following statements are d. 1, 2 and 3
incorrect with respective to Plastic Waste Management
Amendment Rules, 2021?
Answers
1. It aims to ban all types of single use plastics
which also includes commodities made of 1. b
compostable plastic. IndiGau
2. Ban of single use plastics under the rules will
 It is purely indigenous and the largest cattle
be monitored by respective state governments.
chip of the world.
Select the correct answer using the codes given below:
 It is a great example of self-reliant India.
a. 1 only

www.shankariasacademy.com | www.iasparliament.com
56

 This chip will have practical utility in the Training for Augmenting Productivity and
Governments schemes to achieve the goal of Services (TAPAS)
conservation of our own breeds with
 It is an initiative of National Institute of Social
better characters and help towards doubling
Defence (NISD), Ministry of Social Justice and
of farmers’ income by 2022.
Empowerment.
2. d
 It aims to provide access to lectures by subject
Kankrej experts, study material and more, but in a
manner that it supplements the physical
 It is otherwise called as Wadad or Waged,
classroom without compromising on the
Wadhiar.
quality of teaching.
 Originated from Southeast Rann of Kutch of
 The main objective of introducing the course
Gujarat and adjoining Rajasthan (Barmer and
modules is to impart training and enhance the
Jodhpur district).
knowledge and skills for the capacity building
 Kankrej is valued for fast, powerful, draught of the participants.
cattle.
 It can be taken up by anyone who wishes to
 Useful in ploughing and carting. enhance his or her knowledge on the topics
and there is no fee for joining.
 The cows are good milkers, yielding about
1360 kgs.  The five basic courses are on Drug (Substance)
Abuse Prevention, Geriatric/Elderly Care, Care
Sahiwal and Management of Dementia, Transgender
 This breed otherwise known as Lola (loose Issues and on comprehensive course on Social
skin), Lambi Bar, Montgomery, Multani, Teli. Defence Issues.

 Originated in Montgomery district in present 4. c


Pakistan and Indian states of Punjab, Haryana, Jaipur Blue Pottery
& Rajasthan.
 The technique travelled from Iran and Turkey
 The average milk yield of this breed is between to India in the 14th century.
2,725 and 3,175 kgs in lactation period of 300
days.  It was used primarily for souvenirs or
decorative tableware such as coasters, vases,
Ongole trays, plates and doorknobs, its applications
 Otherwise known as Nellore. have remained limited since the 1850s when it
was patronised by the ruling house of Jaipur.
 Home tract is Ongole taluk in Gantur district
of Andhra Pradesh.  A century later, the dying craft and was revived
by the royal family and patrons of art and
 Average milk yield is 1000 kgs. Age at first culture.
calving is 38-45 months with an intercalving
period of 470 days.  In 2005, the Central Glass and Ceramic
Institute of India (CGCII) introduced a new
Kangayam glaze finish.
 Originated in Kangayam, Dharapuram,  It was conferred Geographical Index Status
Perundurai, Erode, Bhavani and part of (GIS) in 2008.
Gobichettipalayam taluk of Erode and
Coimbatore district. Procedure

 The Kangayam breed was developed by the  It is one of the few pottery techniques in the
efforts of the late Pattogar of Palayakottai, Sri world that does not use clay.
N. Nallathambi Sarkari Manradiar.  Locally-sourced quartz powder, recycled glass,
3. b plant-based gum (katera), and Multani Mitti

www.shankariasacademy.com | www.iasparliament.com
57

are ground together and kneaded to make a to stimulate growth of microorganisms and
dough. degrade the target pollutants.
 This is then moulded like a flatbread, sun  Most bioremediation processes involve
dried, and finished with intricate motifs done oxidation-reduction reactions where either an
with oxide pigments, as an underglaze. electron acceptor (commonly oxygen) is added
to stimulate oxidation of a reduced pollutant
 The glaze is applied as the last step before (e.g. hydrocarbons) or an electron donor
firing at around 800 degrees. (commonly an organic substrate) is added to
5. b reduce oxidized pollutants (nitrate,
perchlorate, oxidized metals, chlorinated
The Jajmani System solvents, explosives and propellants)
 In areas where the use of money had not been  Specific bacteria can be used to bio remediate
introduced, service relationships and specific contaminants, such as hydrocarbons,
exchanges in kind may have existed. which are present in oil and gasoline.
 It is likely that the jajmani system evolved  Bacteria such as Paraperlucidibaca,
from these service relations. Cycloclasticus, Oleispira, Thalassolituus
 In many parts of India the jajmani system Zhongshania and some others can help remove
defined most of the transactions in the craft several classes of contaminants.
sector. 7. b
 Jajmani System is a reciprocal arrangement Socio Economic Caste Census
between craft-producing castes and the wider
village community, for the supply of goods and  SECC was conducted in 2011 for the first time
services. since 1931.

 The caste system did not permit the upper  SECC 2011 has three census components
castes to practise certain occupations. which were conducted by three separate
authorities but under the overall coordination
 As a result the patrons or Jajman were of Department of Rural Development in the
dependent on purjans (cultivators, craftsmen, Government of India.
barbers, washermen, cobblers, sweepers, etc.)
to provide essential goods and services for the 1. Census in Rural Area has been conducted by
village/urban economy. the Department of Rural Development
(DoRD).
 In return a fixed payment in kind was assured.
2. Census in Urban areas is under the
 This could be rent-free land, residence sites, administrative jurisdiction of the Ministry of
credit facilities, food or even dung! Housing and Urban Poverty Alleviation
(MoHUPA).
 Since most upper-caste people owned land, the
jajmani system provided them with a stable 3. Caste Census is under the administrative
supply of labour. control of Ministry of Home Affairs: Registrar
General of India (RGI) and Census
 Today this system still holds sway over several Commissioner of India.
parts of the country, though colonialism,
competition, better communications and  The Decennial Census provides a portrait of
improved civil laws have all transformed it in the Indian population, while the SECC is a tool
their own ways. to identify beneficiaries of state support.
6. c  Since the Decennial Census falls under the
Census Act of 1948, all data are considered
Bioremediation
confidential, whereas all the personal
 It is a process used to treat contaminated information given in the SECC is open for use
media, including water, soil and subsurface by Government departments to grant and/or
material, by altering environmental conditions restrict benefits to households.

www.shankariasacademy.com | www.iasparliament.com
58

8. c  Article 275 provides for the payment of such


sums as Parliament may by law provide as
Plastic Waste Management Amendment Rules,
grants-in aid to such States as Parliament may
2021
determine to be in need of assistance.
 Recently, the Ministry Of Environment Forest
 The grants are paid out of the Consolidated
And Climate Change has notified the Plastic
Fund of India in each year, and different sums
Waste Management Amendment Rules, 2021.
may be fixed for different States.
 These rules prohibit specific single-use plastic
 Grants are primarily intended to correct Inter-
items which have ―low utility and high littering
State disparities in financial resources and to
potential‖ by 2022.
coordinate the maintenance and expansion of
 The ban will not apply to commodities made of the welfare schemes of the State Governments
compostable plastic. on a uniform national level.
 The Central Pollution Control Board, along  The grants are released as per the
with state pollution bodies, will monitor the recommendations of the Finance Commission
ban, identify violations, and impose penalties in monthly instalments to meet the gap in
already prescribed under the Environmental Revenue Accounts of the States post-
Protection Act, 1986. devolution (of the divisible tax pool of the
Centre).
Compostable Plastics
 The eligibility of States to receive this grant
1. Compostable Plastics are derived from
and the quantum of grant was decided by the
renewable materials like corn, potato, and
Commission based on the gap between
tapioca starches, cellulose, soy protein, and
assessment of revenue and expenditure of the
lactic acid.
State.
2. These are non-toxic and decompose back into
carbon dioxide, water, and biomass when
composted. 17-08-2021
9. a 1) With reference to the Defence Testing Infrastructure
Greater Adjutant Stroke Scheme (DTIS), consider the following statements:

 The greater adjutant is a member of the stork 1. The Scheme aims at setting up of Greenfield
family, Ciconiidae, there are about 20 species Defence Testing Infrastructure required for
in the family. defence and aerospace related production.
2. Under the scheme, 75% of the project cost will
 Once found across South and Southeast Asia,
be borne by the Special Purpose Vehicle
the Greater Adjutant is one of the most
(SPV) which constitutes the Indian private
threatened stork species in the world.
entities and State governments.
 There are only three known breeding grounds,
3. The remaining 25% will be provided through
one in Cambodia and two in India i.e. Assam
Government funding in the form of ‗Grant-in-
and Bihar.
Aid‘.
 They are protected under IUCN Red List: Which of the statement(s) given above is/are correct?
Endangered and Wildlife (Protection) Act
1972: Schedule IV. a. 2 only
 Recently, Bihar has decided to tag greater b. 1 and 3 only
adjutant storks locally known as ‗Garuda‘ with c. 1 only
GPS trackers to monitor their movement as
part of efforts to conserve them. d. 1, 2 and 3
10. d
Post Devolution Revenue Deficit Grant 2) In the administrative records of the British East
India Company, ―Koots‖ refers to?

www.shankariasacademy.com | www.iasparliament.com
59

a. A Peasant uprising 6) Which of the following statements are correct with


respective to India‘s National Flag
b. A form of Land tax
1. After undergoing several changes, the
c. A title given to Indians
Tricolour was adopted as our national flag at a
d. Right to trade duty free Congress Committee meeting in Karachi in
1931.
2. The flag code of India, 2002 replace all pre-
3) Consider the following statements with respect to existing rules governing the correct display of
Visceral Leishmaniasis the flag.
1. It is a complex infectious disease transmitted 3. Only flags that conform to the specifications of
by the bite of female Anopheles Mosquito. Bureau of Indian Standards can be used for
2. It is a neglected tropical disease that affects official display.
millions annually, making it the second most Select the correct answer using the codes given below:
common parasitic killer after malaria.
a. 1 and 2 only
Which of the statement(s) given above is/are correct?
b. 1 and 3 only
a. 1 only
c. 2 and 3 only
b. 2 only
d. 1, 2 and 3
c. Both 1 and 2
d. Neither 1 nor 2
7) Which of the following statements are correct with
respective to Symplocos Mohananii?
4) The Arctic is warming twice as fast as the rest of the 1. It is an endemic fruit plant which is found only
globe. This warming differential between the poles and in Assam and Nagaland.
the tropics is known as?
2. It was spotted from a hill which was about
a. Arctic Polarisation 3,000 feet above sea level, and the plant
b. Arctic Sublimation carries white flowers.
c. Arctic Amplification Select the correct answer using the codes given below:
d. Arctic Differentiation a. 1 only
b. 2 only
5) Assertion (A): Polar Amplification is much stronger c. Both 1 and 2
in the Arctic than in Antarctica. d. Neither 1 nor 2
Reason (R): The Arctic is an ocean covered by sea ice,
while Antarctica is an elevated continent covered in
more permanent ice and snow. 8) Which of the following statements are incorrect with
respective to Navroz festival?
Select the correct answer using the codes given below:
1. It is celebrated at the time of the vernal
a. Both A and R are true and R is the correct equinox, marking the start of spring in the
explanation of A Northern Hemisphere.
b. Both A and R are true but R is not the correct 2. In India it is celebrated twice a year based on
explanation of A both Iranian calendar and based on
c. A is true but R is false Shahenshahi calendar.
d. A is false but R is true Select the correct answer using the codes given below:
a. 1 only
b. 2 only

www.shankariasacademy.com | www.iasparliament.com
60

c. Both 1 and 2  It envisages to set-up six to eight new test


facilities in partnership with private industry.
d. Neither 1 nor 2
 This will facilitate indigenous defence
production, consequently reduce imports of
9) Consider the following statements with respective to military equipment and help make the country
Fan Tokens self-reliant.
1. The fan tokens are volatile assets and their  The projects under the Scheme will be
value can drastically change overnight similar provided with up to 75% government funding
to digital currencies. in the form of ‗Grant-in-Aid‘.
2. They represent the ownership of shares in  The remaining 25% of the project cost will
game clubs and used for any financial have to be borne by the Special Purpose
transactions within the club. Vehicle (SPV) whose constituents will be
Which of the statement(s) given above is/are correct? Indian private entities and State Governments.

a. 1 only  The SPVs under the Scheme will be registered


under Companies Act 2013 and shall also
b. 2 only operate and maintain all assets under the
c. Both 1 and 2 Scheme, in a self-sustainable manner by
collecting user charges.
d. Neither 1 nor 2
2. a
Koots
10) Consider the following statements with respective
to Slender Lorises (Loris)  These are peasant uprisings that broke out in
South Kanara District during 1830-31.
1. They are found in tropical rainforests, scrub
forests, semi-deciduous forests, and swamps.  The uprising broke out against the East India
Company in the coastal belt reached the stage
2. In south India viable population of the animal of an armed struggle in 1837.
is found in Kadavur Reserve Forest, Tamil
Nadu.  The revolt by peasants was not related to the
freedom movement, though, but against the
3. It is listed as Critically Endangered under
high land revenue (tax) assessment and the
IUCN red list
absence of a lucrative market for farm
Which of the statement(s) given above is/are correct? produce.
a. 1 and 2 only  But it did motivate people to join the freedom
movement after 1850, say researchers.
b. 1 and 3 only
 The peasants were mainly agitated over the
c. 2 and 3 only
company for introducing a system that
d. 1, 2 and 3 mandated that taxes should be paid only in
cash and not in kind.
3. b
Answers
Strategy for Visceral Leishmaniasis
1. c
 Indian researchers have developed a non-
Defence Testing Infrastructure Scheme (DTIS) invasive, easy to administer, cost-effective, and
 It aims to give a boost to domestic defence and patient compliant potential therapeutic
aerospace manufacturing. strategy against Visceral Leishmaniasis.

 It has an outlay of Rs 400 crore for creating  The conventional treatment therapy of VL
state of the art testing infrastructure for mainly involves painful intravenous
defence testing sector. administration, which imposes many

www.shankariasacademy.com | www.iasparliament.com
61

treatment complications, including prolonged  This amplification is primarily caused by


hospitalization, high cost, and high risk of melting ice - a process that is increasing in the
infection. Arctic at a rate of 13% per decade.
 Their strategy based on nano carrier-based  Ice is more reflective and less absorbent of
oral drugs coated with Vitamin B12 enhanced sunlight than land or the surface of an ocean.
oral bioavailability and efficacy of the therapy
by more than 90%.  When ice melts, it typically reveals darker
areas of land or sea, and this results in
Visceral Leishmaniasis increased sunlight absorption and associated
warming.
 Leishmaniasis is caused by protozoan parasites
which are transmitted by the bite of infected Effects of Arctic warming
female phlebotomine sandflies.
 One of the most significant effects of Arctic
 There are 3 main forms of leishmaniases: amplification is the weakening of west-to-east
jet streams in the northern hemisphere.
1. Visceral (also known as kala-azar which is the
most serious form of the disease)  As the Arctic warms at a faster rate than the
2. Cutaneous (the most common), and tropics, this results in a weaker atmospheric
pressure gradient and hence lower wind
3. Mucocutaneous speeds.
 Visceral Leishmaniasis is a neglected tropical 5. a
disease that affects millions annually, making
Polar Amplification
 it the second most common parasitic killer
 It is much stronger in the Arctic than in
after malaria.
Antarctica.
 The disease affects some of the poorest people
 This difference is because the Arctic is an
and is associated with malnutrition,
ocean covered by sea ice, while Antarctica is an
population displacement, poor housing, a weak
elevated continent covered in more permanent
immune system and lack of financial
ice and snow.
resources.
 In fact, the Antarctic continent has not
 Leishmaniasis is also linked to environmental
warmed in the past seven decades, despite a
changes such as deforestation, building of
steady increase in the atmospheric
dams, irrigation schemes and urbanization.
concentrations of greenhouse gases.
4. c
 The exception is the Antarctic peninsula,
Arctic Warming which juts out further north into the Southern
Ocean and has been warming faster than any
 The Arctic is warming twice as fast as the rest
other terrestrial environment in the southern
of the world.
hemisphere during the latter half of the 20th
 The rapid Arctic warming is referred to as century.
Arctic Amplification.
 Satellite data also show that between 2002 and
 Researchers suggests that it may be blamed for 2020, Antarctica lost an average of 149 billion
some of the extreme weather episodes. metric tonnes of ice per year, partly because
the oceans around the continent are warming.
Arctic Amplification
6. b
 It occurs whenever there is any change in the
net radiation balance of Earth, and this The Flag Code of India, 2002
produces a larger change in temperature near  It allowed the unrestricted display of the
the poles than the global average. Tricolour as long as the honour and dignity of
 It is typically measured as the ratio of polar the flag were being respected.
warming to tropical warming.

www.shankariasacademy.com | www.iasparliament.com
62

 The flag code did not replace the pre-existing Navroz festival
rules governing the correct display of the flag.
 It is a tradition is observed by Iranians and
 It was, however, an effort to bring together all Zoroastrian around the world, also known as
the previous laws, conventions and practices. Parsi new-year
 It mentions that the tricolour cannot be used  Though celebrated in March globally, Navroz
for commercial purposes, and cannot be arrives 200 days later in India and is
dipped in salute to any person or thing. celebrated in the month of August as the Parsis
here follow the Shahenshahi calendar that
 For official display, only flags that conform to doesn‘t account for leap years.
the specifications as laid down by the Bureau
of Indian Standards and bearing their mark  Interestingly in India, people celebrate it twice
can be used. a year - first according to the Iranian calendar
and the second according to the Shahenshahi
Adoption of national flag calendar which is followed by people here and
1. 1906 - The first national flag, which consisted in Pakistan. The festival falls between July and
of three horizontal stripes of red, yellow and August.
green, is said to have been hoisted on 7th
 It was inscribed in the list of UNESCO
August, 1906, in Calcutta (now Kolkata).
Intangible Cultural Heritage of Humanity of
2. 1921 - Later, in 1921, freedom fighter Pingali India in 2009.
Venkayya met Mahatma Gandhi and proposed
9. a
a basic design of the flag, consisting of two red
and green bands. Fan Tokens
3. 1931 - After undergoing several changes, the  Fan Tokens are a type of NFT (non-fungible
Tricolour was adopted as our national flag at a token) which is a digital-only asset.
Congress Committee meeting in Karachi in
1931.  The Fan Tokens are digital assets that
represent the ownership of a voting right and
4. 1947 - The Indian flag was adopted in its give you access to earn unique club-specific
present form during a meeting of the rewards and experiences.
Constituent Assembly held on 22nd July, 1947.
 Fans can purchase said crypto tokens with
7. b real-world money to gain access to exclusive
Symplocos Mohananii content and augmented-reality games.

 Symplocos Mohananii has been discovered at  The fan token holders also get to vote on
Kerala‘s Ponmudi hills which is part of the mostly minor decisions related to their clubs.
Western Ghats.  The more tokens a fan has, the more votes they
 The tree is very endemic in nature and we get in the polls but ultimately, the polls are
could spot only five of them during our decided upon by the clubs.
research.  Fan Tokens aren‘t about transaction but they
 It was spotted from a hill which was about are about ownership as a utility, which gives
3,000 feet above sea level. the access to a global community of fandom,
rewards & team influence.
 The plant carries white flowers but we could
not record opening of the flowers so far, the 10. a
white flowers usually open in night. Slender Lorises (Loris)
 Plants of this family are commonly known as  Slender lorises is secretive and has nocturnal
sweet leaf plants locally and the bark of a few habits.
species were used to treat skin diseases by
tribals and also for dye-making.  It spend most of their life in trees, traveling
along the tops of branches with slow and
8. d precise movements.

www.shankariasacademy.com | www.iasparliament.com
63

 The slender loris has been listed as d. In recent years, the Indian Peafowl have been
‗Endangered‘ by the International Union for found to be expanding in Kerala, a region falling
the Conservation of Nature. under humid tropics.
 It has been brought under Schedule I of the
Wild Life (Protection) Act, 1972 in order to 3) Consider the following statements with respect to
provide the highest level of legal protection. Financial Inclusion Index
 The wildlife census conducted during 2016-17 1. It is being released once in a year by the
showed an appreciable population of 3,500 Ministry of Finance.
slender loris in the Kadavur Reserve Forest.
2. It has no base year.
 The loris species is also found in the adjoining
forest areas on the eastern, southern and 3. It covers banking, investments, insurance,
western slopes of the Kadavur hills, which are postal and pension sectors.
managed by the Tiruchirappalli and Dindigul Which of the statement(s) given above is/are correct?
forest divisions respectively.
a. 1 only
b. 3 only
18-08-2021
c. 2 and 3 only
1) With reference to Ramsar Sites in India, consider the
following statements: d. 1, 2 and 3

1. India currently has 46 Ramsar sites designated


as Wetlands of International Importance. 4) Consider the following statements with respect to
2. Uttar Pradesh has the most number of Ramsar Land Use Statistics data
Sites in India. 1. Maharashtra has the highest agricultural
3. Currently, no sites in India are listed under landholding in terms percentage of its total
Montreux record, a register of wetland sites geographical area.
where changes in ecological character have 2. The national average of geographical area
occurred, are occurring, or are likely to occur. available for agriculture is around 55%.
Which of the statement(s) given above is/are correct? 3. Between 2001 and 2011, there is a trend of
a. 2 only decreasing cultivators and agricultural
labourers in the country.
b. 1 and 2 only
Which of the statement(s) given above are not correct?
c. 2 and 3 only
a. 1 and 2 only
d. None of the above
b. 2 and 3 only
c. 1 and 3 only
2) With reference to Indian Peafowl, which of the
statements given above is incorrect? d. 1, 2 and 3

a. It is an arid-land species which prefers semi-


arid biomes and dry deciduous forests. 5) Consider the following statements with respect to
b. It is native to Mediterranean region and National Monetisation Pipeline
travelled from Iran and Turkey to India. 1. It is a roadmap of or the asset monetization of
c. They are now listed under the ‗Least Concern‘ various brownfield infrastructure assets.
category of International Union for Conservation 2. It has been created for a 4-year period from
of Nature (IUCN) Red Data list. financial year 2021-22 to 2024-25.
3. Department of Investment and Public Asset
Management under Ministry of Finance

www.shankariasacademy.com | www.iasparliament.com
64

finalised the framework for the National 2. It causes respiratory disease among elephants,
Monetisation Pipeline. so far there is no effective cure or treatment for
the disease.
Which of the statement(s) given above is/are correct?
Select the correct answer using the codes given below:
a. 1 only
a. 1 only
b. 3 only
b. 2 only
c. 1 and 2 only
c. Both 1 and 2
d. 2 and 3 only
d. Neither 1 nor 2

6) Which of the following statement(s) is/are correct


with respective to Congressional Gold Medal 9) Both parole and furlough are considered as
reformative processes and are covered under the
1. It is one of the highest civilian awards in the
Prisons Act of 1894. Which of the following statements
United States.
are correct with respective to Parole and Furlough?
2. Mahatma Gandhi was the first Indian to
1. Parole is not a right, and is given to a prisoner
receive this award in 1940.
for a specific reason, such as a death in the
Select the correct answer using the codes given below: family or a wedding of a blood relative.
a. 1 only 2. The period of furlough granted to a prisoner is
treated as remission of his sentence.
b. 2 only
Which of the statement(s) given above is/are correct?
c. Both 1 and 2
a. 1 only
d. Neither 1 nor 2
b. 2 only
c. Both 1 and 2
7) Which of the following statement(s) is/are correct
with respective to Receptor Binding Domain (RBD)? d. Neither 1 nor 2
1. It is a short immunogenic fragment from a
virus that binds to a specific endogenous
10) Consider the following statements with respective
receptor sequence to gain entry into host cells.
to National Automobile Scrappage Policy
2. It is a critical component of the viral spike
1. All official vehicles more than 25 years‘ old will
glycoprotein that is found on coronaviruses
be scrapped, irrespective of their working
including SARS-CoV-2
conditions.
Select the correct answer using the codes given below:
2. The state governments may offer a road-tax
a. 1 only rebate of up to 25% for personal vehicles to
owners to scrap old and unfit vehicles.
b. 2 only
Which of the statement(s) given above is/are correct?
c. Both 1 and 2
a. 1 only
d. Neither 1 nor 2
b. 2 only
c. Both 1 and 2
8) Which of the following statement(s) is/are correct
with respective to EEHV-1 Herpesvirus? d. Neither 1 nor 2
1. It highly affects the African elephant
population and causes very less fatality on
Asian elephants. Answers
1. b

www.shankariasacademy.com | www.iasparliament.com
65

Wetlands of International Importance (Ramsar  It comes under Section 51 (1-A) of Schedule I


Sites) of the Wild (Life) (Protection) Act, 1972, with
imprisonment that may be extended up to
 Four more wetlands from India get recognition
seven years, along with a fine that shall not be
from the Ramsar Secretariat as Ramsar sites
less than Rs 10,000.
recently.
 Since 2014, Indian Peafowl have been
 With this, the number of Ramsar sites in India
protected under Appendix III of the
is increased to 46.
Convention on International Trade in
 Uttar Pradesh, with 8 Ramsar Sites, has the Endangered Species of Wild Fauna and Flora
most number of Ramsar Sites in India. (CITES).
Montreux Record  They are now listed under the ‗Least Concern‘
(LC) category of the International Union for
 It is a register of wetland sites on the List of the Conservation of Nature Red Data list.
Wetlands of International Importance where
changes in ecological character have occurred, 3. c
are occurring, or are likely to occur as a result Financial Inclusion Index
of technological developments, pollution or
other human interference.  Financial Inclusion Index is released by RBI
once in a year in the month of July.
 It is maintained as part of the Ramsar List.
 It is to capture the extent of financial inclusion
 At present, there are two Indian sites Loktak across the country.
Lake, Manipur and Keoladeo National Park,
Rajasthan are on the Montreux Record.  It comprises of 3 parameters - Access (35%),
Usage (45%), and Quality (20%) .
2. b
 It incorporates details of banking, investments,
Increase of Peacock in Kerala
insurance, postal as well as the pension sector
 During recent years, the Indian Peafowl, an in consultation with the government and
arid-land species, have been found to be respective sectoral regulators.
expanding in Kerala, a region falling under
 It has no base year as it reflects the cumulative
humid tropics.
efforts of various sectors towards financial
 Birds can be regarded as a bioindicator of inclusion in the country.
environmental change as they are very
 Its value ranges from 0 to 100 in which 0
sensitive to vagaries of climate.
represents financial exclusion and 100 reflects
 The increasing sightings of the Indian Peafowl complete financial inclusion.
(Pavo cristatus) are an indicator of the
4. c
changing climate in Kerala.
Land Use Statistics
Indian Peafowl (Pavo cristatus)
 Union Agricultural Minister in his recent
 It is commonly seen in scrub jungles and forest
speech in Lok Sabha revealed the land use
edges, with an affinity towards dry deciduous
statistics data for the year 2017-18.
forests, semi-desert areas and semi-arid
biomes.  It highlights the following facts about the
agricultural landholdings in the country.
 The Indian peafowl is a native of India and
some parts of Pakistan and Sri Lanka. 1. The national average of geographical area
available for agriculture was 55.03 %.
 The Arakan hills prevented their spread
further east while the Himalayas and the 2. The States of Haryana (85.03%) and Punjab
Karakoram did so northwards. (84.09%) have a maximum amount of
available geographical area under agriculture.
 As our national bird, the peacock has the
utmost level of legal protection.

www.shankariasacademy.com | www.iasparliament.com
66

3. It was as low as 4.86% in Jammu and Kashmir, 6. a


5.06% in Arunachal Pradesh, 39% in Andaman
Congressional Gold Medal
and Nicobar Islands, and 9.09% in
Chandigarh.  The Congressional Gold Medal is an award
4. Total agricultural land consists of net area bestowed by the United States Congress.
sown, current fallows, culturable waste and  It is Congress's highest expression of national
land under miscellaneous tree crops. appreciation for distinguished achievements
 2011 census data revealed that there was a and contributions by individuals or
decline (7%) in the number of cultivators and institutions.
an increase (26%) in the number of  Recently, a resolution has been reintroduced in
agricultural labourers between 2001 and 2011. the US House of Representatives to
5. c posthumously award the Congressional Gold
Medal to Mahatma Gandhi for his
National Monetisation Pipeline contributions made through his methods of
non-violence.
 NITI Aayog has finalised the National
Monetisation Pipeline framework for financial  If given the award, Mahatma Gandhi would
year 2021-22 to 2024-25. become the first Indian to receive the
Congressional Gold Medal, which is the
 It serve as a roadmap for the asset
highest civilian award in the US.
monetization of various brownfield
infrastructure assets across sectors including 7. c
roads, railways, aviation, power, oil and gas,
and warehousing. Receptor Binding Domain (RBD)

 Central ministries were given a target of Rs 2.5  It is a short immunogenic fragment from a
trillion for monetising their assets over three virus that binds to a specific endogenous
years starting 2021-22 (FY22). receptor sequence to gain entry into host cells.
 RBD is a critical component of the viral spike
glycoprotein that is found on coronaviruses
including SARS-CoV-2, the virus that causes
COVID-19.
 The binding of the RBD on the spike domain is
a critical step that allows coronaviruses to bind
to target body receptors (such as ACE2 on
respiratory epithelial cells) and enter cells to
cause infection.
 The RBD is therefore an important target for
neutralizing antibodies, either through
engineered vaccination or convalescent plasma
of recovered patients.
8. d
EEHV-1 Herpesvirus
 (ElHV-1) is a type of herpesvirus, which can
cause a highly fatal haemorrhagic disease
when transmitted to young Asian elephants.
 It has a very high mortality rate in Asian
elephants, which kills up to 80% of severely
affected individuals.

www.shankariasacademy.com | www.iasparliament.com
67

 The disease can be treated with the rapid  It aims to reduce the population of old and
application of antiviral drugs, but this has only defective vehicles, bringing down vehicular air
been effective in around a third of cases. pollutants, improving road and vehicular
safety.
 It is one of the most deadly viral infections in
elephants worldwide but is most commonly  Highlights of the policy
found in Asian elephants. EEHV can strike
1. Old vehicles will have to pass a fitness test
without warning.
before re-registration and as per the policy
 When infected with the virus, elephants government commercial vehicles more than 15
usually show symptoms of sudden illness years old and private vehicles which are over
which include reduced appetite, swollen glands 20 years old will be scrapped.
on both sides of the face and nasal discharge.
2. Old vehicles will be tested at authorized
 Unlike the common herpes-simplex viruses, Automated Fitness Center and will not be
which attacks the skin and lies latent in nerve scrapped merely on the basis of age.
cells, the elephant virus attacks endothelial 3. Emission test, braking system, safety
cells the cells that line blood vessels, the heart components will be tested and the vehicles
and other organs. which fail in the fitness test will be scrapped.
9. c 4. If the old vehicle passes the test, the owner can
Parole continue to use it, but the charges for
reregistration will be much steeper.
 It is a system of releasing a prisoner with
suspension of the sentence. 5. The state governments may be advised to offer
a road-tax rebate of up to 25% for personal
 The release is conditional, usually subject to vehicles and up to 15% for commercial vehicles
behaviour, and requires periodic reporting to to provide incentive to owners of old vehicles
the authorities for a set period of time. to scrap old and unfit vehicles.
 Parole is not a right, and is given to a prisoner 6. Vehicle manufacturers will also give a discount
for a specific reason, such as a death in the of 5% to people who will produce the
family or a wedding of a blood relative. 'Scrapping Certificate' and registration fees
will be waived off on the purchase of a new
 It may be denied to a prisoner even when he
vehicle.
makes out a sufficient case, if the competent
authority is satisfied that releasing the convict
would not be in the interest of society.
19-08-2021
Furlough
1) Which of the following statements is correct about
 It is similar to parole, but with some UNITE Aware?
significant differences. It is given in cases of
long-term imprisonment. a. It is a peacekeeping mission by UN to restore
democratic government in Afghanistan.
 The period of furlough granted to a prisoner is
treated as remission of his sentence. b. It is a resolution in UNSC put forward by India
to create awareness about Rohingya refugee crisis
 Unlike parole, furlough is seen as a matter of in the country.
right for a prisoner, to be granted periodically
c. It is a technological platform to enhance the
irrespective of any reason, and merely to
security of UN Peacekeeping forces.
enable the prisoner to retain family and social
ties, and to counter the ill-effects of prolonged d. It is a mutual agreement among G7 nations to
time spent in prison. combat climate change through strengthening
Nationally Determined Contributions (NDC).
10. b
National Automobile Scrappage Policy

www.shankariasacademy.com | www.iasparliament.com
68

2) Consider the following statements about Kigali d. Prime Ministers‘ Office (PMO)
Amendment to the Montreal Protocol
1. It aims to phase down all the ozone depleting
5) Consider the following statements about
substances by cutting both their production
Enumeration of National Minimum Wages
and consumption.
1. The Expert Committee on determining the
2. It is legally binding and came into force in
Methodology for Fixing the National Minimum
2019.
Wages fixed the minimum wages equivalent to
3. Under the amendment, India‘s reduction of National Commission for Rural Labour
Hydroflurocarbon (HFC) will begin from 2028 Recommendations.
and target is to reduce it by 80% by the year
2. The committee expanded the units of
2047.
consumption per household and included ―City
Which of the statement(s) given above is/are correct? Compensation allowance‖ for urban workers.
a. 2 only Which of the statement(s) given above is/are correct?
b. 3 only a. 1 only
c. 1 and 3 only b. 2 only
d. 2 and 3 only c. Both 1 and 2
d. Neither 1 nor 2
3) Consider the following statements about Mineral
Prospecting Operations in the country
6) Wool consumption by processing units increased by
1. Under the Mines and Minerals (Development 50 per cent between 2010-2020. Which of the
and Regulation) Amendment Act, 2021, any following statement(s) is/are correct with respect to
entities including private entities are allowed Wool production in India?
to undertake prospecting operations.
1. Rambouillet sheep is indigenous to India and
2. The act empowers central government to wool from this species alone accounts in 35%
reserve any mine to be leased through an of wool production.
auction for a particular end use.
2. Rajasthan is India‘s largest wool producer
Which of the statement(s) given above is/are correct? known for its superior carpet grade Chokla and
Magra wool.
a. 1 only
Select the correct answer using the codes given below:
b. 2 only
a. 1 only
c. Both 1 and 2
b. 2 only
d. Neither 1 nor 2
c. Both 1 and 2
d. Neither 1 nor 2
4) In the light of recent controversy over telephonic
interceptions in Pegasus issue, the government has
recently informed the parliament about the competent
7) Consider the following statements with respective
authority to approve legal telephonic interception
to Nuclear Fusion
under Indian Telegraph Act, 1885 and IT Act, 2000.
Which of the following authorities is entrusted with 1. It is defined as the combining of two lighter
such powers in case of Central Government? nuclei into a heavier one by releasing positively
charged alpha particles.
a. Secretary to Department of Telecommunication
2. Currently all commercial nuclear reactors are
b. Minister of State (Electronics and IT)
based on nuclear fusion technology.
c. Union Home Secretary
Which of the statement(s) given above is/are correct?

www.shankariasacademy.com | www.iasparliament.com
69

a. 1 only a. 1 only
b. 2 only b. 2 only
c. Both 1 and 2 c. Both 1 and 2
d. Neither 1 nor 2 d. Neither 1 nor 2

8) Consider the following statements with respective to Answers


Water plus cities
1. c
1. Water Plus certificate is awarded to cities that
have met all the ODF Double Plus standards in UNITE Aware
the SwachhSurvekshan Survey.  India is currently presiding over UNSC. In its
2. Kochi, Kerala has been declared as the capacity, it has rolled out an initiative in
country's first 'water plus city'. partnership with UN called ―UNITE Aware‖.
Which of the statement(s) given above is/are correct?  It is a technological platform to enhance the
security of UN Peacekeeping forces.
a. 1 only
 It is to ensure that entire peacekeeping
b. 2 only operation can be visualized, coordinated and
c. Both 1 and 2 monitored on a real time basis in order to
prevent any attack or respond to an attack
d. Neither 1 nor 2 immediately.
 It is rolled out in four UN Peacekeeping
9) Which of the following statement(s) is/are correct Missions – UNMISS (Sudan), UNFICYP
with respective to Phagocytosis (Cyprus), MINUSMA (Mali) and AMISOM
(Somalia)
1. It is the process by which a cell uses its plasma
membrane to give rise to an internal 2. d
compartment called the phagosome.
Kigali Amendment to Montreal Protocol
2. It is a major mechanism in a multicellular
 Union Cabinet has recently approved the
organism's immune system used to remove
ratification of the Kigali Amendment to the
pathogens and cell debris.
Montreal Protocol on Substances that deplete
Select the correct answer using the codes given below: the ozone layer.
a. 1 only  In 2016, Kigali amendment was agreed by
more than 170 countries and came into force in
b. 2 only
2019.
c. Both 1 and 2
 It is legally binding.
d. Neither 1 nor 2
 The amendment aims to phase down
Hydroflurocarbon (HFC) i.e to achieve over
10) Which of the following statement(s) 80% reduction in HFC by 2047.
is/are incorrect with respective to Drone Forensic Lab  HFCs were introduced as a replacement to
1. India‘s first Drone Forensic Lab and Research ozone depleting substances such as
Centre has setup in Gujarat by Ministry of Civil chlorofluorocarbons and hydro
Aviation chlorofluorocarbons.

2. It will look into both threat aspects of a drone,  Thus, HFCs have zero ozone depleting
and production of drones for the use of the potential but known to be hundreds, even
police force. thousands, of times more potent than carbon
Select the correct answer using the codes given below:

www.shankariasacademy.com | www.iasparliament.com
70

dioxide in their ability to cause global Monitoring and Decryption of Information)


warming. Rules, 2009.
 With the Kigali Amendment, the Montreal 5. b
Protocol has become an even more powerful
The Expert Committee on determining the
instrument against global warming.
Methodology for Fixing the National Minimum
 Under the amendment, India‘s reduction of Wages
Hydroflurocarbon (HFC) will begin from 2028
 It outlined a methodology for enumeration of
and target is to reduce it by 80% by the year
minimum wages.
2047.
 It used consumption expenditure and
3. a
employment data to arrive at a figure that
Mines and Minerals (Development and allowed for a balanced diet, other non-food
Regulation) Amendment Act, 2021 essential items and expanded the units of
consumption per household.
 At present, only government agencies are
involved in exploration and the pace of  Based on this, the committee stated a figure of
exploration is limited by their capacity. Rs.375/day as a national floor-level minimum
wage, with regional variations, and a ―city
 The amendment acts empower the Central compensatory allowance‖ for workers in urban
government to notify entities, including private areas.
entities, to undertake prospecting operations.
 This figure is much higher than the non-
 The Mines and Minerals (Development and binding national floor wage of Rs.176 (based
Regulation) Act, 1957 empowers the central on National Commission of Rural Labour
government to reserve any mine other than Recommendations), it is much lower than the
coal, lignite, and atomic minerals to be leased `600 demanded by trade union federations,
through an auction for a particular end-use. based on the 7th Pay Commission
 Particular end-use involves reserving mined recommendations.
ore for a purpose such as iron ore mine for a  Minimum wage enumeration is based on two
steel plant. Such mines are called as captive key features
mines.
o Recommendations of the 15th Indian
 The amendment act provides that no mine will Labour Conference (1957)
be reserved for particular end-use.
o Supreme Court judgment in Workmen
4. c v Reptakos Brett (1992).
Telephonic Interceptions 6. b
 In a recent reply by the government in the Wool production in India
Parliament, it is made clear that that the Home
Secretary is the competent authority to deal  India has the 3rd largest sheep population
with legal interceptions in case of Central country in the world having 07 million
government and Secretary in charge of Home sheep producing 43.50 million kg of raw wool
Department in case of States/Union in 2017-18.
Territories.
 Carpet grade is rougher than apparel grade
 Home Secretary‘s power to approve legal and accounts for 85 per cent of India‘s
interceptions is mandated under production.
1. Section 5(2) of the Indian Telegraph Act, 1885  Apparel grade wool accounts for less than five
read with Rule 419A of Indian Telegraph (1st per cent of production and coarse grade fit for
Amendment of 2014) Rules, 2014 making rough blankets accounts for the rest.
2. Section 69 of the Information Technology Act,  Largest wool producing states in India are in
2000 read with the Information Technology) the order of Rajasthan, Telangana, Karnataka,
Procedure and Safeguards for Interception, Gujarat, and Himachal Pradesh.

www.shankariasacademy.com | www.iasparliament.com
71

 Rajasthan is the largest wool producer known  Under the Swachh Bharat Mission (Urban),
for its superior carpet grade Chokla and Magra the cities of the country are tested on the basis
wool. of various cleanliness parameters.
 Australian Merino sheep, known to have the  It has categories of ODF+, ODF++ and
softest and finest wool used for apparels, India Water+.
is planning to import these sheeps.
 The Water Plus certificate is awarded to cities
 The last import was of Rambouillet sheep from that have met all the ODF Double Plus
the US in 1993, its purpose was crossbreeding, standards.
which had a low survival rate.
 Also, the residual sewage from residential and
7. a commercial establishments is released into the
environment only after treatment. Reuse of
Nuclear Fusion
treated wastewater should also be ensured.
 Nuclear Fusion is defined as the combining of
 Indore, which has been ranked number one in
two lighter nuclei into a heavier one.
cleanliness four times in the country, has been
 Such nuclear fusion reactions are the source of declared as the country's first 'water plus city'
energy in the Sun and other stars. in the results of Swachh Survekshan, 2021.
 Recently lasers were used to heat pellets 9. c
containing deuterium and tritium fused and Phagocytic cells
produced more energy (a yield of more than
1.3 megajoules)  Phagocytosis is the process by which a cell uses
its plasma membrane to engulf a large particle,
 It released positively charged particles called giving rise to an internal compartment called
alpha particles, which in turn heated the the phagosome.
surrounding plasma.
 It is one type of endocytosis, a cell that
 The heated plasma also released alpha performs phagocytosis is called a phagocyte.
particles and a self-sustaining reaction called
ignition took place.  Newly discovered phagocytic cells were
identified in cauliflower coral and starlet sea
 Nuclear fusion energy is a good choice as the anemone.
baseload energy in the future with many
advantages, such as inexhaustibility of  In a multicellular organism's immune system,
resources, inherent safety, no long-lived phagocytosis is a major mechanism used to
radioactive wastes, and almost no CO2 remove pathogens and cell debris. The
emissions. ingested material is then digested in the
phagosome.
Nuclear Fission
 Bacteria, dead tissue cells, and small mineral
 In this reaction the nucleus of an atom splits
particles are all examples of objects that may
into two daughter nuclei.
be phagocytized.
 The resulting fragments tend to have a
 Some protozoa use phagocytosis as means to
combined mass which is less than the original.
obtain nutrients.
 The missing mass is usually converted into 10. a
nuclear energy.
Drone Forensic Lab
 Currently all commercial nuclear reactors are
based on nuclear fission.  The Kerala Police department will launch a
first-of-its-kind Drone Forensic Lab and
8. a Research Centre in the state.
Water plus cities
 It will help in keeping a watch over
unauthorized drones and also for the

www.shankariasacademy.com | www.iasparliament.com
72

production of drones for the use of the police


force.
3) With respect to Methane Hydrates, consider the
 This lab-cum-research centre will look into following statements:
both utility and threat aspects of a drone.
1. It is a solid compound in which a large amount
 It had recently analysed a drone brought down of methane gas molecules (CH4) are caged
by the Border Security Force (BSF) near the within a crystalline structure of water.
international border in Kathua district in 2. The total amount of carbon in permafrost
Jammu and Kashmir and helped the local associated methane hydrates is much less than
enforcement zero in on its illegal operator. the carbon in permafrost soils.
Which of the statement(s) given above is/are correct?
19-08-2021 (IPCC Special) a. 1 only
1) With respect to the Intergovernmental Panel on b. 2 only
Climate Change (IPCC), consider the following
statements: c. Both 1 and 2

1. It is an intergovernmental body of the United d. Neither 1 nor 2


Nation (UN) that publishes Assessment
Reports once in every five years.
4) With reference to Radiative Forcing, consider the
2. It was established by the United Nations following statements:
General Assembly (UNGA) and the World
Meteorological Organization (WMO) in 1988. 1. It is the difference between incoming and
outgoing energy in the Earth‘s climate.
3. Participation in the IPCC is open to all
member countries of the WMO and UN. 2. Positive Forcing refers to cooling where the
outgoing energy is greater than the incoming
Which of the statement(s) given above is/are correct? energy.
a. 3 only 3. Negative forcing refers to warming where the
b. 1 and 2 only incoming energy being greater than the
outgoing energy.
c. 2 and 3 only
4. Forcings from volcanoes and human-emitted
d. 1, 2 and 3 aerosols are negative forcings.
Which of the statement(s) given above is/are correct?
2) With reference to the findings of 6th Assessment a. 1 only
Report of the Intergovernmental Panel on Climate
Change (IPCC), consider the following statements: b. 1 and 4 only

1. The ocean surface will continue to warm more c. 2, 3 and 4 only


than the land surface. d. 1, 2, 3 and 4 only
2. The Arctic will continue to warm more than
the global surface temperature.
5) With reference to Net Zero Emissions, consider the
3. Human-induced global warming has been following statements:
more rapid in Africa than the rest of the world.
1. It is a condition in which a country would
Which of the statement(s) given above is/are correct? bring down its emissions to zero, taking pre-
a. 2 only industrial levels as base range.

b. 1 and 3 only 2. India has not yet committed to a net zero


timeline.
c. 2 and 3 only
Which of the statement(s) given above is/are correct?
d. 1, 2 and 3

www.shankariasacademy.com | www.iasparliament.com
73

a. 1 only 2. The Antarctic is the biggest carbon pool on


Earth, storing twice as much carbon in its
b. 2 only
permafrost, than is currently stored in the
c. Both 1 and 2 atmosphere.
d. Neither 1 nor 2 Which of the statement(s) given above is/are correct?
a. 1 only
6) Consider the following statements with respect to b. 2 only
Natural variability
c. Both 1 and 2
1. It refers to variations in climate that are
d. Neither 1 nor 2
caused by processes other than human
influence.
2. External natural variability corresponds to the 9) Consider the following statements:
redistribution of energy within the climate
1. Warming will be stronger in the Arctic, on land
system whereas the Internal Variability
and in the Northern Hemisphere.
corresponds to changes in Earth‘s orbit or
from volcanic eruptions. 2. Precipitation will increase in high latitudes, the
tropics and monsoon regions.
Which of the statement(s) given above is/are correct?
Which of the statement(s) given above is/are correct?
a. 1 only
a. 1 only
b. 2 only
b. 2 only
c. Both 1 and 2
c. Both 1 and 2
d. Neither 1 nor 2
d. Neither 1 nor 2

7) With respect to Short Lived Climate Forcers


(SLCFs), consider the following statements: 10) With reference to Permafrost, consider the
following statements:
1. These are compounds that warm or cool the
Earth‘s climate over shorter time scales. 1. It is any ground that remains completely
frozen or colder for at least two years straight.
2. It includes gases as well as tiny particles called
aerosols. 2. It can be found in cold places like Tibetan
Plateau, high-altitude regions like Rocky
3. They are emitted both naturally and as a result
Mountains, and on the floor of the Arctic
of human activities.
Ocean.
Which of the statements given above are correct?
3. Although the ground is frozen in permafrost
a. 1 and 2 only regions, they are not always covered in snow.
b. 1 and 3 only Which of the statements given above are correct?
c. 2 and 3 only a. 1 and 2 only
d. 1, 2 and 3 b. 1 and 3 only
c. 2 and 3 only
8) Consider the following statements: d. 1, 2 and 3
1. Climate warming induced permafrost thawing
could release greenhouse gases in to the
atmosphere and further amplifies climate Answers
change. 1. a

www.shankariasacademy.com | www.iasparliament.com
74

Intergovernmental Panel on Climate Change  As a result, the frequency of marine heatwaves


(IPCC) will continue to increase in the tropical ocean
and the Arctic.
 It is an intergovernmental body of the United
Nations (UN) for assessing the science related  This will amplify permafrost thawing and loss
to climate change. of seasonal snow cover of land and sea ice.
 It was created in 1988 by the World  The Arctic is likely to be practically sea ice-free
Meteorological Organization (WMO) and the at least once before 2050.
United Nations Environment Programme
(UNEP).  Highest CO2 Levels - The levels of CO2, the
primary driver of global heating, were higher
 The objective of the IPCC is to provide in 2019 than at any time in ―at least 2 million
governments at all levels with scientific years‖.
information that they can use to develop
climate policies.  Methane Cuts - In addition to slashing CO2
emissions, the world must also deliver ―strong,
 IPCC reports are also a key input into rapid and sustained reductions‖ in methane in
international climate change negotiations. order to get to grips with the climate crisis.
 The IPCC is an organization of governments  Sea Level Rise - A warmer climate will
that are members of the United Nations or intensify very wet and very dry weather and
WMO. climate events and seasons leading to flooding
or drought.
 The IPCC currently has 195 members.
 The IPCC published its First AR in 1990,
second in 1995, third in 2001, fourth in 2007
and the Fifth Assessment Report in 2014.
2. c
Highlights of the IPCC 6th Assessment Report
 Human’s role in climate crisis - Human
activity is the cause of climate change and this
is an unequivocal fact.
 Global temperatures have already risen by 1.1
degrees Celsius since the 19th century.
 Average global temperatures will continue to
rise and could increase by 5.7°C by the end of
this century as compared to 1850-1900.
 No region on Earth has escaped the impacts of
the climate crisis.
 Extreme Weather Events - Every
additional 0.5°C rise in temperature amplifies
the intensity and frequency of heatwaves,
heavy precipitation and droughts.
 The land surface will continue to warm more
than the ocean surface.
 Ice free Arctic - The Arctic will continue to
warm more than global surface temperature.
 The temperature on the coldest days will
increase by three times in the Arctic.

www.shankariasacademy.com | www.iasparliament.com
75

 Land and ocean‘s capacity to absorb carbon  The IPCC report noted that the human-caused
dioxide (CO2) will decrease resulted in the radiative forcing of 2.72 Wm–2 in 2019
emitted CO2 remain in the atmosphere. relative to 1750 has warmed the climate
system.
 Greenland ice sheet and Antarctic ice sheet will
continuously lose ice over the 21st century.  This warming is mainly due to increased GHG
concentrations, partly reduced by cooling due
 This ice loss from the Greenland ice sheet will to increased aerosol concentrations.
increase with cumulative emissions.
Radiative Forcing
 Deep ocean warming and ice sheet melt will
drive sea level rise for centuries and millennia.  Earth is continually bathed in energy from the
sun.
 Africa - Human-induced global warming has
been more rapid in Africa than the rest of the  Sunlight energy heats land and water at the
world. surface, and in turn, they emit heat.
 At 1.5°C global warming, heavy precipitation  This heat provides further warming of the
and associated flooding are projected to atmosphere.
intensify and be more frequent in most regions
 The mix of gases in our atmosphere keeps
in Africa.
some of the heat energy from escaping directly
3. c to space, similar to the way a blanket keeps
warmth near your body.
Methane Hydrates
 This process is the naturally occurring
 Methane Hydrate (MH) is a solid compound in
greenhouse effect, and it keeps Earth warm
which a large amount of methane gas
enough to support life.
molecules (CH4) are caged within a crystalline
structure of water under low temperature and  In accordance with the basic laws of
high pressure, forming a solid similar to ice. thermodynamics, as Earth absorbs energy
from the sun, it must eventually emit an equal
 It looks like ice, but starts burning when an
amount of energy to space.
open flame is brought close to it; methane
hydrate is often called ―fiery ice.‖  Radiative Forcing - The difference between
incoming and outgoing radiation is known as a
 They formed as frozen soils that were flooded
planet‘s Radiative Forcing (RF).
when sea levels rose after the last ice age.
 Positive Forcing - When forcings result in
 If these hydrates thaw, they may release
incoming energy being greater than outgoing
methane that can bubble up to the surface.
energy, the planet will warm (positive RF).
 The total amount of carbon in permafrost
 Negative Forcing - Conversely, if outgoing
associated methane hydrates is much less than
energy is greater than incoming energy, the
the carbon in permafrost soils.
planet will cool.
 Global warming takes millennia to penetrate
 Some forcings are positive while others, such
into the sediments beneath the ocean.
as those from volcanoes or human-emitted
 As a result, only a small fraction of the existing aerosols, are negative.
hydrates could be destabilised during the 5. b
coming century.
Net Zero Emissions
 Even when methane is released from hydrates,
most of it is expected to be consumed and  It is also referred to as carbon-neutrality.
oxidised into carbon dioxide in the ocean
 It does not mean that a country would bring
before reaching the atmosphere.
down its emissions to zero.
4. b
 Rather, net-zero is a state in which a country‘s
emissions are compensated by absorption and

www.shankariasacademy.com | www.iasparliament.com
76

removal of greenhouse gases from the  Internal Natural Variability - It


atmosphere. corresponds to a redistribution of energy
within the climate system and is most clearly
 Absorption of the emissions can be increased
observed as regional, rather than global.
by creating more carbon sinks such as forests,
while removal of gases from the atmosphere  For example, via atmospheric circulation
requires futuristic technologies such as carbon changes similar to those that drive the daily
capture and storage. weather, fluctuations in surface temperature.
 This way, it is even possible for a country to  External Natural Variability - It can result
have negative emissions, if the absorption and from changes in the Earth‘s orbit, small
removal exceed the actual emissions. variations in energy received from the sun, or
from major volcanic eruptions.
 A good example is Bhutan which is often
described as carbon-negative because it
absorbs more than it emits.
 India is currently the World‘s third biggest
emitter of greenhouse gases, after the US and
China.
 India has not yet committed to a net zero
timeline.
 Several other countries, including the UK and
France, have already enacted laws promising
to achieve a net-zero emission scenario by the
middle of the century.
 The European Union is working a similar
Europe-wide law, while many other countries
including Canada, South Korea, Japan and
Germany have expressed their intention to
commit themselves to a net-zero future.
 Even China has promised to go net-zero by
2060.
6. a 7. d
Natural variability Short Lived Climate Forcers (SLCFs)
 It refers to variations in climate that are  These are compounds that warm or cool the
caused by processes other than human Earth‘s climate over shorter time scales – from
influence. days to years. They do not remain in the
atmosphere for very long.
 It includes variability that is internally
generated within the climate system and  Climatic effects of Carbon dioxide, a
variability that is driven by natural external greenhouse gas, lasts for decades, centuries or
factors. even more.
 Natural variability is a major cause of year-to-  The SLCFs include gases as well as tiny
year changes in global surface climate and can particles called aerosols, and they can have a
play a prominent role in trends over multiple warming or cooling effect on the climate.
years or even decades.
 Warming SLCFs are either greenhouse gases
 But the influence of natural variability is (e.g., ozone or methane) or particles like black
typically small when considering trends over carbon (also known as soot), which warm the
periods of multiple decades or longer. climate by absorbing energy and are

www.shankariasacademy.com | www.iasparliament.com
77

sometimes referred to as short-lived climate 8. a


pollutants.
 Cooling SLCFs, on the other hand, are
mostly made of aerosol particles (e.g.,
sulphates, nitrates and organic aerosols) that
cool down the climate by reflecting away more
incoming sunlight.
 Some SLCFs do not directly affect the climate
but produce climate-active compounds and are
referred to as Precursors.
 Some of the SLCF Precursors includes sulphur
dioxide (which produces sulphates) and
nitrogen oxides (which produce nitrates and
ozone).
 SLCFs are emitted both naturally and as a
result of human activities, such as agriculture
or extraction of fossil fuels.
 Emissions have increased since the start of Arctic Warming
industrialization, and humans are now the
dominant source for several SLCFs and SLCF  In the Arctic, large amounts of organic carbon
precursors. are stored in permafrost.
 Permafrost is the ground that remains frozen
throughout the year.
 If significant areas of permafrost thaw as the
climate warm, some of that carbon may be
released into the atmosphere in the form of
carbon dioxide or methane, resulting in
additional warming.
 The Arctic is the biggest climate-sensitive
carbon pool on Earth, storing twice as much
carbon in its frozen soils, or permafrost, than
is currently stored in the atmosphere.
 As the Arctic region warms faster than
anywhere else on earth, there are concerns that
this warming could release greenhouse gases
to the atmosphere and therefore significantly
amplify climate change.
Permafrost
 The carbon in the permafrost has built up over
thousands of years, as dead plants have been
buried and accumulated within layers of frozen
soil, where the cold prevents the organic
material from decomposing.
 As the Arctic warms and soils thaw, the
organic matter in these soils begins to
decompose rapidly and return to the
atmosphere as either carbon dioxide or

www.shankariasacademy.com | www.iasparliament.com
78

methane, which are both important  In the southern hemisphere, where there‘s far
greenhouse gases. less ground to freeze, permafrost is found in
mountainous regions such as the South
 Permafrost can also thaw abruptly in a given
American Andes and New Zealand‘s Southern
place, due to melting ice in the ground
Alps, as well as below Antarctica.
reshaping Arctic landscapes, lakes growing and
draining, and fires burning away insulating
surface soil layers.
20-08-2021
9. c
1) With respect to Gulf Stream, consider the following
statements:
1. It is the biggest current in the North Atlantic
Ocean.
2. It is a warm current composed of both vertical
– Atlantic Meridional Overturning Circulation
(AMOC) and horizontal - North Atlantic
Subtropical Gyre circulations.
Which of the statement(s) given above is/are correct?
a. 1 only
b. 2 only
c. Both 1 and 2
d. Neither 1 nor 2

2) Consider the following statements with respect to


Nagarjunasagar Srisailam Tiger Reserve (NSTR)
1. The reserve spreads over the Nallamala ranges
10. d
of the southern Eastern Ghats.
Permafrost
2. It supports the tropical mixed dry deciduous
 It is any ground that remains completely and moist deciduous forests.
frozen—32°F (0°C) or colder—for at least two 3. Rajiv Gandhi Wildlife Sanctuary and Gundla
years straight. Brahmeswaram Wildlife Sanctuary constitute
 These permanently frozen grounds are most the core area of the tiger reserve.
common in regions with high mountains and Which of the statement(s) given above is/are correct?
in Earth‘s higher latitudes—near the North and
South Poles. a. 1 and 2 only

 Permafrost covers large regions of the Earth. b. 1 and 3 only

 Almost a quarter of the land area in the c. 2 and 3 only


Northern Hemisphere has permafrost d. 1, 2 and 3
underneath.
 Although the ground is frozen, permafrost
regions are not always covered in snow. 3) DRDO has recently developed an Advanced Chaff
Technology for Indian Air Force. Chaff Technology is
 Permafrost can be found in colder places like associated with?
the Tibetan plateau, high-altitude regions like
a. Search and Rescue
the Rocky Mountains, and on the floor of the
Arctic Ocean as undersea permafrost. b. Intelligence gathering

www.shankariasacademy.com | www.iasparliament.com
79

c. Underwater Detection Capabilities a. 1 only


d. Protection from hostile radar threats b. 2 only
c. Both 1 and 2
4) Consider the following pairs: d. Neither 1 nor 2
Peaks in News – Countries
1. Mt Manirang – India 7) Consider the following statements about Mahatma
Gandhi‘s favourite Hymns
2. Mt Elbrus – Russia
1. His favourite bhajan ―Vaishnav jan to‖ was
3. Mt Merapi – New Zealand
written by 15th century Gujarati poet-
Which of the pair(s) given above is/are correctly philosopher Narasinh Mehta.
matched?
2. Gandhi first consciously introduced the bhajan
a. 3 only around 1915 to fellow residents of Sabarmati
Ashram.
b. 1 and 2 only
Which of the statement(s) given above is/are correct?
c. 2 and 3 only
a. 1 only
d. 1, 2 and 3
b. 2 only
c. Both 1 and 2
5) Arrange the following countries in descending order
based on the amount of coal they exported to India, d. Neither 1 nor 2
during 2020-21:
1. Australia
8) Consider the following statements about Official
2. USA Language Act, 1963
3. Russia 1. It provides for the use of English language for
all official purposes of the Union and
4. Indonesia transaction of business in the parliament.
5. South Africa 2. Parliament can authorize the use of Hindi or
Which of the statement(s) given above is/are correct? any other official language of State for
judgments, decrees and orders passed by the
a. 1-2-3-4-5 High Court of the State.
b. 4-1-5-2-3 3. It lay down that English should be used for
c. 5-4-3-2-1 purposes of communication between the
Union and the Non-Hindi states.
d. 4-2-1-5-3
Which of the statements given above are correct?
a. 1 and 2 only
6) Consider the following statements about The
Deposit Insurance and Credit Guarantee Corporation b. 1 and 3 only
(DICGC) Amendment Bill, 2021 c. 2 and 3 only
1. DICGC cannot increase the flat rate premium d. 1, 2 and 3
of 0.15% for a bank from its total outstanding
deposits.
2. The maximum limit of insurance cover for 9) Consider the following statements about Rice
bank deposits is Rs. 5 lakh per depositor. Fortification
Which of the statement(s) given above is/are correct? 1. It is a process of deliberately adding essential
macronutrients such as iron, folic acid, vitamin

www.shankariasacademy.com | www.iasparliament.com
80

B-12 to regular rice to improve its nutritional 1. Vertical - Atlantic Meridional Overturning
quality. Circulation (AMOC)
2. In India, fortified rice kernels are produced to 2. Horizontal - North Atlantic Subtropical Gyre
extruder technology.
 Based on models and theory, scientific studies
3. The cooking of fortified rice requires special indicate that, while the AMOC is expected to
procedure and shelf life usually lasts for 3 slow in a warming climate, the Gulf Stream
months. will not change much and would not shut
down totally, even if the AMOC did.
Which of the statements given above are not correct?
a. 1 and 2 only  Most climate models project that the AMOC
slows in the later 21st century.
b. 1 and 3 only
 One reason why is freshening of the ocean
c. 2 and 3 only waters:
d. 1, 2 and 3 1. by meltwater from Greenland
2. changing Arctic sea ice
10) Consider the following statements about Global 3. increased precipitation over warmer northern
Youth Development Index 2020 seas.
1. It ranks States according to the developments  The Gulf Stream affects the weather and sea
in youth education, employment, health, level, so if it slows, North America will see
equality, political and civic participation higher sea levels and Europe‘s weather and
between the age of 15 and 29. rate of relative warming will be affected.
2. India is ranked among top 5 risers alongside
Afghanistan, Russia.
Which of the statement(s) given above is/are correct?
a. 1 only
b. 2 only
c. Both 1 and 2
d. Neither 1 nor 2

Answers
1. c
Gulf Stream
 The Gulf Stream is the biggest current in the
North Atlantic Ocean.
 It transports about 30 billion kilograms of
water per second northward past points on the
east coast of North America.
 It is a warm current, with temperatures 5°C to
15°C warmer than surrounding waters, so it
carries warmer water (thermal energy) from its 2. d
southern origins and releases warmth to the
 The Government of India has recognised the
atmosphere and surrounding water.
reserve forest area on the fringes of
 It is a part of two circulation patterns: Nagarjunasagar Srisailam Tiger Reserve

www.shankariasacademy.com | www.iasparliament.com
81

(NSTR) as an Eco-Sensitive Zone under the  The importance of this technology lies in the
Environment Protection Act, 1986, ratifying fact that very less quantity of chaff material
the proposals sent by the Government of deployed in the air acts as decoy to deflect
Andhra Pradesh. enemy‘s missiles for ensuring safety of the
fighter aircraft.
 The move would fillip the efforts to conserve
the big cat in the country‘s biggest reserve,  The technology has been given to the industry
NSTR. for production in large quantities to meet the
annual rolling requirement of the Indian Air
Nagarjunasagar Srisailam Tiger Reserve
Force.
(NSTR)
4. b
 The NSTR is spread over an area of 3,727.82
sq. km in the Nallamala ranges of the southern Mount Elbrus
Eastern Ghats.
 It is the highest and most prominent peak in
 Most of the area is hilly terrain with plateaus, Russia and Europe.
ridges, gorges and deep valleys, which support
the tropical mixed dry deciduous and moist  It is situated in the western part of the
deciduous forests with an under growth of Caucasus and is the highest peak of the
bamboo and varieties of grass species. Caucasus Mountains.

 At present, two wildlife sanctuaries – Rajiv  It is situated on a dormant volcano rises 5,642
Gandhi Wildlife Sanctuary and Gundla m above sea level.
Brahmeswaram Wildlife Sanctuary (GBM) – Mount Merapi
constitute the core area of the tiger reserve.
 It is an active stratovolcano located in
 The NSTR attained the status of a tiger reserve Indonesia.
in 1983.
 It is the most active of more than 120 active
3. d volcanoes in Indonesia and has repeatedly
Chaff Technology erupted with lava and gas clouds recently.

 Defence Laboratory Jodhpur, a DRDO  Indonesia, an archipelago, is prone to


laboratory has recently developed an advanced earthquakes and volcanic activity because it
Chaff material and chaff cartridge-118/I in sits along the Pacific ―Ring of Fire,‖ a
collaboration with High Energy Materials horseshoe-shaped series of seismic fault lines
Research Laboratory (HEMRL), a Pune based around the ocean.
laboratory of DRDO. Mt Manirang
 The Indian Air Force has started the process of  It is one of the highest peaks of Himachal
induction of this technology after completion Pradesh, nestled at the border of Kinnaur &
of successful user trials. Spiti districts.
Working of Chaff Technology  Close to the peak is the Manirang pass, which
 In today‘s electronic warfare, survivability of was one of the early trade routes between Spiti
fighter aircraft is of prime concern because of and Kinnaur, before the motorable road was
advancement in modern radar threats. built.

 To ensure survivability of aircraft, Counter 5. b


Measure Dispensing System (CMDS) is used In the year 2020-21, coal was mainly imported from:
which provides passive jamming against Infra-
Red and radar threats. 1. Indonesia (92.535 MT)

 Chaff is a critical defence technology used to 2. Australia (54.953 MT)


protect fighter aircraft from hostile radar 3. South Africa (31.143 MT)
threats.
4. USA (12.204 MT)

www.shankariasacademy.com | www.iasparliament.com
82

5. Russia (6.749 MT) especially Section 3 – both Hindi and English


should be used for the official documents.
6. Singapore (4.486 MT)
 The Act laid down that English should be used
7. Mozambique (3.570 MT)
for purposes of communication between the
6. b Union and the Non-Hindi states.
DICGC  The Governor of a State with the previous
consent of the President can authorize the use
 It is a wholly owned subsidiary of RBI.
of Hindi or any other official language of State
 It is liable to pay the insured amount to the for judgments, decrees and orders passed by
depositors when the insured bank undergoes the High Court of the State.
liquidation, reconstruction under a scheme 9. b
and merger/acquisition.
Rice Fortification
 At present, it charges a flat premium rate of
0.15% from the bank out of its total  Rice fortification is a process of
outstanding deposits to meet its liability to pay adding micronutrients to regular rice.
the insured amount to the depositor.
 For rice fortification in India, ‗extrusion‘ is
 As per the amendment bill, with the approval considered to be the best technology, which
of RBI, DICGC can increase the deposit involves the production of fortified rice kernels
insurance premium for a bank based on its risk (FRKs) from a mixture using an extruder
profile. machine.
 The insurance cover for bank deposits  These kernels have a shelf life of at least 12
increased fivefold to Rs. 5 lakh per depositor months.
with effect from February 2020.
 The cooking of fortified rice does not require
7. a any special procedure.
Favourite Hymns of Mahatma Gandhi  The rice needs to be cleaned and washed in the
normal way before cooking. After cooking,
 It includes Vaishnav jan to, verses from
fortified rice retains the same physical
Ramacharitamanas of Tulsidas, Rabindranath
properties and micronutrient levels as it had
Tagore‘s ―When the Heart is hard‖, ―Here is
before cooking.
thy Footstool‖.
 Fortified rice will be packed in jute bags with
 His favourite bhajan ―Vaishnav jan to‖ was
the logo (‗+F‘) and the line ―Fortified with
written by Narashinh Mehta.
Iron, Folic Acid, and Vitamin B12‖ mandatorily
o He was born in Talaja village, printed on the pack.
Junagadh in Gujarat.
10. c
o He was a 15th century poet-philospher
Global Youth Development Index 2020
of the vaishnava sect from the Bhakti
era.  It is released by the Commonwealth
Secretariat in London.
 Research Suggest that Gandhi first consciously
introduced the bhajan around 1907 to fellow  It measures the condition of young people
residents of the Phoenix settlement and across 181 countries.
Tolstoy farm in South Africa.
 India is ranked at 122 and it is among the top 5
8. b risers alongside Afghanistan, Russia, Ethiopia
Official Language Act, 1963 and Burkina Faso.

 Recently, Madras High Court has directed the  Singapore ranked topmost followed by
Union government and all its instrumentalities Slovenia, Norway, Malta and Denmark.
to follow the Official Languages Act, 1963,

www.shankariasacademy.com | www.iasparliament.com
83

 It ranks States according to the developments Select the correct answer using the codes given above:
in youth education, employment, health,
a. 1-c; 2-d; 3-b; 4-a
equality and inclusion, peace and security, and
political and civic participation. b. 1-a; 2-d; 3-b; 4-c
c. 1-c; 2-a; 3-b; 4-d
21-08-2021 d. 1-a; 2-c; 3-b; 4-d
1) Sunil Sethi Commission, sometimes seen in the news
recently, is constituted for which of the following 4) Binance, often seen in the news recently, refers to?
purposes?
a. A Cryptocurrency Exchange
a. To study the impact of pandemic lockdowns in
public behavior b. A notice issued to a fugitive economic offender
b. To double the production and quadruple the c. A microprocessor to carry out cryptographic
exports of handlooms operations
c. To suggest measures to improve management of d. A stage of a market cycle where asset prices rise
public funds and reduce fiscal deficit to irrationally high levels
d. To examine the feasibility of integrating the
agriculture year with the financial year 5) With reference to Greenland, consider the following
statements:
2) The Ministry of Power has recently said that 17 new 1. It is the world‘s largest island between the
Islanding schemes have been planned for the major Arctic and Atlantic Oceans.
cities besides already existing 26 schemes. In this 2. It has three-quarters of its surface covered
context, consider the following statements: with a permanent ice sheets.
1. Islanding is the intentional isolation of a part 3. Recently, for the 1st time on record, the
of power system during external widespread summit of Greenland received rainfall and not
grid disturbance. snow.
2. Islanding Scheme ensures the maintenance of Which of the statement(s) given above is/are correct?
essential services and also faster restoration of
supply to the electricity consumers in the event a. 1 and 2 only
of any outage. b. 1 and 3 only
Which of the statement(s) given above is/are correct? c. 2 and 3 only
a. 1 only d. 1, 2 and 3
b. 2 only
c. Both 1 and 2
Answers
d. Neither 1 nor 2
1. b
 The Government has recently constituted a
3) Match the following: Committee headed by
Geo-Heritage Sites – States Sunil Sethi, Chairman, Fashion Design Council
of India (FDCI) New Delhi.
1. Chabimura – a. Tripura
 The main objective of the committee is to
2. Sangetsar Tso – b. Manipur suggest the strategy and policy framework for
3. Loktak Lake – c. Meghalaya doubling the production and quadrupling the
exports of handlooms in a span of 3 years.
4. Mawmluh Cave – d. Arunachal Pradesh

www.shankariasacademy.com | www.iasparliament.com
84

 It will also suggest measures for improving the  It is located near Cherrapunjee in the East
quality of handloom products with the aim of Khasi Hills district.
enhancing income of the weavers.
 This cave led scientists to the Meghalayan Age.
2. c
 Meghalayan Age - According to the
Islanding scientists, when the Meghalayan Age began i.e.
4200 years ago, there was a sudden drought
 Islanding is the intentional isolation of a part
worldwide and the temperature had declined
of power system during external widespread
grid disturbance.  A stage of the Meghalayan Age is defined from
a specific level in a stalagmite from this cave.
 This isolated part of Grid is called Island.
Chabimura – Tripura
 Such a disturbance may lead to black out.
 In Gomati district, this site is known for its
Islanding Scheme
panels of rock carving on a steep hill wall on
 Resilience of the electricity grid is very the bank of river Gomati.
important particularly in restoration of supply
 The huge images of Shiva, Vishnu, Karthikeya,
in the event of the major power outage.
Durga and other gods and goddesses date back
 Therefore, Islanding scheme provides a mean to the 15th-16th century and the biggest carved
to continue to supply power to the essential deity is about 20 ft.
services in a zone or area.
 The hill range is covered with thick jungles and
 Islanding Scheme is a defense mechanism for one can reach this abode of gods after trekking
the power system in which a part of the system through the foliage but rafting or boating on
is islanded from a disturbed grid so that this the river is the only option for a view of the
subpart could survive in isolation from rest of rock-face carvings.
grid and continuity of supply to the essential
Sangetsar Tso – Arunachal Pradesh
load in this area is maintained.
 It was popularly known as Madhuri Lake.
 In such electrical Islanded system, the
essential loads should be identified and  This water body in Tawang district is close to
covered so that these load continue to be the border with Tibet and was formed due to
served even during any major outage. the damming of a river during a major
earthquake in 1950.
 As informed by Central Electricity Authority
(CEA), 17 numbers of new Islanding schemes  The lake is surrounded by a lush valley and
have been planned for the major cities besides snow-capped mountains.
already 26 existing/under implementation
scheme in India. Loktak Lake – Manipur

3. b  This lake in the Bishnupur district is the


largest freshwater lake in the Northeast.
 The Geological Survey of India (GSI) has
identified certain geological sites across the  The attractions of this lake are the ‗ phumdis‘
Northeast for promotion of geo-tourism. or floating biomass and the ‗phumsangs‘ or
huts of fishermen on them.
 Twelve locations in the Northeast are included
in the 32 approved geo-tourism or geo-  The Keibul Lamjao National Park, the only
heritage sites in the country. floating wildlife habitat on earth, is on the
south-western part of the lake and is the last
 Of the 12 sites in the Northeast, three are in natural habitat of the sangai or brow-antlered
Meghalaya, two each in Assam and Tripura, dancing deer.
and one each in Arunachal Pradesh, Manipur,
Mizoram, Nagaland and Sikkim. 4. a

Mawmluh Cave – Meghalaya Binance

www.shankariasacademy.com | www.iasparliament.com
85

 It is a cryptocurrency exchange that provides a  Rain & high temperatures triggered extensive
platform for trading various cryptocurrencies. melting here, which is 7 times more than daily
average observed at this time (August) of the
 The exchange offers a wide range of services to year.
users across the globe, from cryptocurrency
spot and derivatives trading to loans and non-  Rapid ice melting will be running off into the
fungible tokens. ocean in volumes, thus accelerating global sea
level rise.
 It also runs a "decentralised" exchange that
allows users to trade directly with each other.  Concern - The UN‘s ―code red‖ climate report
concluded that the burning of fossil fuels led to
 Its own cryptocurrency, Binance Coin, is the Greenland melting in the last 20 years.
third-biggest in the world, with some $68
billion-worth in circulation.  Arctic Ocean may witness ice-free summers by
2050 due to extreme climate interventions.
 The global financial regulators were concerned
over the use of crypto in money laundering.  If that happens, sea levels could rise by 20 feet,
threatening low-lying cities around the world
 They have targeted Binance and some have such as Mumbai, New York, etc.
banned the platform from certain activities,
while others have warned consumers that it  Rapid melting is also threatening polar bears,
was unlicensed to operate. which have to make their way towards
Greenland‘s interior from the coasts, where
 The platform has said that it would demand they usually find enough food.
stricter background checks on customers to
bolster efforts against money laundering, with
immediate effect.
23-08-2021
5. d
1) With respect to Eutrophication, consider the
Rain at Greenland Summit following statements:
 For the 1st time on record, the summit of 1. It is defined as the population explosion of
Greenland received rain and not snow, just as algae and blue-green bacteria that covers
temperatures at the spot went above freezing almost entire surface layer of a lake or river.
for the 3rd time in less than 10 years.
2. It is primarily caused by the leaching of
 This was the heaviest rainfall that the phosphate and or nitrate containing fertilisers
Greenland received since record keeping began from agricultural lands into lakes or rivers.
in 1950.
Which of the statement(s) given above is/are correct?
 Greenland is the world‘s largest island between a. 1 only
the Arctic and Atlantic Oceans.
b. 2 only
 Three-quarters of its surface is covered with a
permanent ice sheet. c. Both 1 and 2
 At the highest point on Greenland‘s ice sheet, d. Neither 1 nor 2
the US‘s National Science Foundation
maintains a Summit Station, a research facility
that observes changes occurring over the 2) NTPC Ltd has recently commissioned the India‘s
island as well as in Arctic weather. largest floating Solar Photo Voltaic power project in
which of the following regions?
 The research facility observed rain at the
normally frigid summit, with the precipitation a. Rann of Kutch, Gujarat
extending up to Greeland‘s southeast coast. b. Banasura Sagar reservoir, Kerala
 Not only is water warmer than the usual snow, c. Basava Sagar Reservoir, Karnataka
it's also darker - so it absorbs more sunlight
rather than reflecting it away. d. Simhadri Reservoir, Andhra Pradesh

www.shankariasacademy.com | www.iasparliament.com
86

b. 1 and 3 only
3) Consider the following statements with respect to c. 2 and 3 only
Ubharte Sitaare Fund
d. 1, 2 and 3
1. Under the scheme, an identified company is
supported even if it is currently
underperforming or may be unable to tap its 6) Under the new Tribunals Reform Act, 2021, which
latent potential to grow. of the following is/are part of the Search cum Selection
Committee with voting rights to appoint the
2. The fund has been set up by the EXIM Bank
chairperson/members of the tribunals?
and Small Industries Development Bank of
India (SIDBI). 1. Chief Justice of India
3. The fund is a mix of structured support 2. Retired SC/HC Judge
comprises both financial and advisory
services. 3. Sitting/Outgoing Chairperson

Which of the statements given above are correct? 4. Concerned Ministry‘s Secretary

a. 1 and 2 only 5. Minister of State for Home Affairs

b. 1 and 3 only Choose the correct option

c. 2 and 3 only a. 1, 2 and 3 only

d. 1, 2 and 3 b. 1, 3 and 4 only


c. 1, 2, 4 and 5 only

4) Jalakanyaka, sometimes seen in the news recently, d. 1, 2, 3, 4 and 5


is?
a. Marine algae species discovererd in Andaman & 7) Consider the following statements about
Nicobar Islands Desertification and Land Degradation Atlas of India
b. Jain stories related to previous births of 1. It is prepared by the Geological Survey of
Mahavira in both human & animal form India.
c. Ancient system of astronomy used during the 2. According to it, about 50% of India‘s Total
period of Vijayanagar Empire Geographical Area (TGA) became degraded
d. A form of water sport played by the Irula tribes during 2018-19.
of Kerala along the Arabian Coast 3. India witnessed an increase in the level of
desertification in 28 of 31 states and Union
territories between 2011-13 and 2018-19.
5) Consider the following statements with respect to
Malabar Rebellion of 1921 Which of the statements given above are correct?

1. It is an uprising of Muslim tenants (Moplahs) a. 1 only


against local Hindu landlords (Jenmis) and b. 3 only
British rulers.
c. 1 and 2 only
2. It is an armed revolt led by Mannath
Padmanabhan, A.K. Gopalan and N.P. d. 2 and 3 only
Damodaran.
3. The revolt, began as a part of Khilafat 8) In India, the impacts of glyphosate and imidacloprid
Movement, took place in the regions under the on bacterioplanktons are examined widely. These
Malappuram district of Kerala. chemicals are used as?
Which of the statements given above are correct? a. Fertilizers in agriculture
a. 1 and 2 only

www.shankariasacademy.com | www.iasparliament.com
87

b. Herbicides and Insecticides in agriculture could be fatal to fishes, humans, birds and
other aquatic organisms.
c. Microplastics used in cosmetics
 Muttukadu Estuary is located near Chennai,
d. Fruit-ripening agents
Tamil Nadu and runs parallel to the east coast,
the Bay of Bengal.
9) India has signed an Inter-Governmental Agreement Eutrophication
(IGA) with which of the following countries for the
import of AK-203 assault rifles and Kamov-226 utility  Eutrophia is a greek word which means
helicopters? adequate & healthy nutrition.

a. United States of America  Eutrophication is a syndrome of ecosystem,


response to the addition of artificial or
b. United Kingdom natural nutrients such as nitrates and
c. Russia phosphates through fertilizer, sewage, etc that
fertilize the aquatic ecosystem.
d. Israel
 It is primarily caused by the leaching of
phosphate and or nitrate containing fertilisers
10) Consider the following statements about FDI in from agricultural lands into lakes or rivers.
Insurance Sector Algal Bloom
1. The Department for Promotion of Industry  Some algae and blue-green bacteria thrive on
and Internal Trade (DPIIT) has revised the the excess ions and a population explosion
FDI limit in insurance sector from 49% to 74% covers almost entire surface layer is known as
under automatic route. algal bloom.
2. Such overseas investments in insurance  As Algal Bloom covers the surface layer, it
company promoted by private bank would restricts the penetration of sunlight and
need to be cleared by RBI in consultation with reduces the oxygen level.
Insurance Regulatory and Development
Authority of India (IRDAI).  Due to reduced oxygen level, fishes and other
aquatic organism suffocate and they die.
Which of the statements given above is/are correct?
2. d
a. 1 only
 The National Thermal Power Corporation
b. 2 only
(NTPC) Ltd, has recently commissioned the
c. Both 1 and 2 largest floating solar PV project of 25MW on
the reservoir of its Simhadri thermal station in
d. Neither 1 nor 2
Visakhapatnam, Andhra Pradesh.
 This is also the first solar project to be set up
Answers under the Flexibilisation Scheme, notified by
the Government of India in 2018.
1. b
 The 2000MW coal-based Simhadri Station is
 An excessive algal and plant growth in the the first power project to implement an open
Muttukadu Estuary is posing a serious sea intake from the Bay of Bengal which has
problem to the environment and fish species. been functional for more than 20 years.
 Microcystis aeruginosa, a species of  NTPC is also planning to set up a hydrogen-
cyanobacteria that can form harmful algal based micro-grid system on a pilot basis at
blooms, has affected the water quality of the Simhadri.
Muttukadu backwaters.
 NTPC is India's first energy company to
 These algae produce hepatotoxin, called declare its energy compact goals as part of the
‗microcystin‘, which in high concentrations UN High-level Dialogue on Energy (HLDE).

www.shankariasacademy.com | www.iasparliament.com
88

3. d depend on oxygen to live, including this


species.
 Union Finance Minister Nirmala Sitharaman
has recently launched the Ubharte Sitaare 5. b
Fund
 August 20, 2021 marks the centenary of the
Ubharte Sitaare Fund Malabar rebellion or the Moplah (Muslim)
riots of 1921.
 The fund will aid the micro, small and medium
enterprises (MSMEs) which could be future  Malabar rebellion had been an uprising of
champions with good export potential. Muslim tenants (Moplahs) against local Hindu
landlords (Jenmis) and British rulers.
 The fund has been set up by EXIM Bank and
SIDBI.  The uprising, which began as part of the
Khilafat Movement, took place in regions
 The fund is expected to identify Indian which are under the Malappuram district,
enterprises with potential advantages by way Kerala.
of technology, products or processes along
with export potential, but which are currently  It was an armed revolt led by Variyamkunnath
underperforming or unable to tap their latent Kunjahammed Haji.
potential to grow.
 It has been perceived as one of the 1st
 The fund is a mix of structured support, both nationalist uprisings in southern India. It has
financial and advisory services through even been described as a peasant revolt.
investments in equity or equity like
instruments, debt (funded and non-funded)  In 1971, the Kerala government had included
and technical assistance (advisory services, the participants of the rebellion in the category
grants and soft loans) to the Indian companies. of freedom fighters.

 The fund will go a long way in making India a  Impacts - The rebellion of Mappilas inspired
major exporting hub. by religious ideology and a conception of an
alternative system of administration - Khilafat
4. a government - dealt a blow to the nationalist
movement in Malabar.
Jalakanyaka
 Fanaticism of rebels, foregrounded by the
 A new marine algae species with an umbrella-
British, fostered communal rift and enmity
like cap, which has been named Jalakanyaka,
towards the Congress.
has been discovered on Andaman & Nicobar
Islands. 6. a
 The new algae species has been named Tribunal Reforms Act, 2021
‗Acetabularia jalakanyakae‘, after the Sanskrit
word ‗Jalakanyaka‘ meaning the ‗goddess of  It seeks to dissolve certain existing appellate
oceans‘ or ‗mermaid‘. bodies and transfer their functions (such as
adjudication of appeals) to other existing
 This species is the 1st of the genus Acetabularia judicial bodies.
to be discovered in India.
 The Chairperson and Members of the
 It is also the first of its kind on the Island. Tribunals will be appointed by the central
government on the recommendation of a
 Algal diversity is one of the highest in the Search-cum-Selection Committee.
Andaman & Nicobar Islands.
 The Committee will consist of
 The whole plant is made up of just one cell
with only one nucleus. (i) The Chief Justice of India, or a Supreme Court
Judge nominated by him, as the Chairperson (with
 Threats - Rising seawater temperature casting vote),
decreases oxygen concentration in water,
dangerously affecting all organisms that (ii) Two Secretaries nominated by the central
government,

www.shankariasacademy.com | www.iasparliament.com
89

(iii) The sitting or outgoing Chairperson, or a retired  These services can be impaired if microbiomes
Supreme Court Judge, or a retired Chief Justice of a are severely affected by excessive pesticide
High Court, and usage.
(iv) The Secretary of the Ministry under which the  Study observations - Bacterial cell density
Tribunal is constituted (with no voting right). increased, albeit slightly, due to glyphosate
7. b and carbon substrate utilisation witnessed
negligible effect with the introduction of both
Desertification and Land Degradation Atlas of glyphosate and imidacloprid.
India
9. c
 It was published by the Space Applications
AK-203 Assault Rifles
Centre under ISRO in June 2021.
 In February 2019, India and Russia has signed
 Report Highlights – In 2018-19, 29.7% of the
an Inter-Governmental Agreement (IGA) to set
country‘s Total Geographical area (97.85
up a joint enture — Indo-Russian Rifles Private
million hectares) became degraded.
Limited (IRRPL) at Korwa in Uttar Pradesh for
 Land degradation within dry land regions manufacturing AK-203 rifles.
(arid, semi-arid and dry sub-humid regions) is
 In 2015, India and Russia had concluded an
termed as ‗desertification‘.
Inter-Governmental Agreement (IGA) for at
 Most of the degraded area is contributed by least 200 Ka-226T twin-engine utility
Rajasthan, Maharashtra, Gujarat, Karnataka, helicopters.
Ladakh, Jharkhand, Odisha, Madhya Pradesh
10. c
and Telangana.
FDI in Insurance Sector
 India witnessed an increase in the level of
desertification in 28 of 31 states and Union  The Insurance (Amendment) Bill, 2021, was
territories between 2011-13 and 2018-19. passed by Parliament in March which
increased the FDI limit in the sector from 49%
 In Goa and Odisha, desertification declined to 74% under automatic route.
between 2003-05 and 2011-13.
 According to the new rules Foreign Exchange
 Land degradation and desertification was Management (Non-debt Instruments) (Second
declining in Uttar Pradesh, Rajasthan and Amendment) Rules, 2021, applications for FDI
Telangana in 2018-2019. in an insurance company promoted by a
8. b private bank would be cleared by the RBI and
IRDAI.
Impact of Insecticides and Herbicides
 This is to ensure that the 74% limit of overseas
 Planktons are tiny organisms found in water investment is not breached.
and as they can‘t propel themselves are
transported by the currents.
 The major groups of planktons are 24-08-2021
phytoplankton, zooplankton, bacterioplankton,
1) With reference to Yuktdhara, sometimes seen in the
microplankton and virioplankton.
news recently, consider the following statements:
 A recent study has examined the impact of 1. It is a geospatial portal developed under the
herbicide glyphosate and the insecticide called existing Bhuvan portal of ISRO.
imidacloprid on the bacterioplankton
communities. 2. The portal is dedicated for educational and
associated services and helps in taking
 Bacterial communities provide useful education to every nook and corner of India.
ecosystem services like the decomposition of
organic matter and the cycling of nutrients. 3. This platform will also serve as a repository of
assets (geotags) created under various national
rural development programmes.

www.shankariasacademy.com | www.iasparliament.com
90

Which of the statement(s) given above is/are correct? Programme - Objectives


a. 2 only 1. PRIYA - a. Accessibility booklet
b. 1 and 2 only 2. NISHTHA - b. Foundational Literacy
and Numeracy
c. 1 and 3 only
3. SARTHAQ - c. NEP implementation plan
d. 1, 2 and 3
4. NIPUN Bharat - d. Capacity building of
Secondary teachers
2) With reference to Sree Narayana Guru, consider the
Select the correct answer using the codes given below:
following statements:
a. 1-a; 2-d; 3-c; 4-b
1. He himself consecrated a Vishnu idol in
Aruvippuram Temple against the then b. 1-c; 2-d; 3-a; 4-b
tradition of Brahmins holding the right to
c. 1-a; 2-b; 3-c; 4-d
consecrate.
d. 1-d; 2-c; 3-a; 4-b
2. He started a Sanskrit school in Varkala and
poor boys and orphans were given free
education regardless of their caste.
5) Consider the following statements with respect to
3. In 1913, he founded the Advaita Ashram at the BRICS STI Framework Programme
Aluva, which was dedicated to a great principle
– all human beings are equal in the eyes of 1. It was endorsed in 2015 to support excellent
God. research on priority areas which can best be
addressed by a multinational approach.
Which of the statement(s) given above is/are correct?
2. The initiative should facilitate cooperation
a. 3 only among the researchers in the consortia, which
consist of partners from at least 3 of the BRICS
b. 2 and 3 only
countries.
c. 1, 2 and 3
Which of the statement(s) given above is/are
d. None of the above incorrect?
a. 1 only
3) With respect to Nuclear Fusion process, consider the b. 2 only
following statements:
c. Both 1 and 2
1. It is a clean and green route to producing
d. Neither 1 nor 2
energy than nuclear fission.
2. Sun generates energy using this process by
converting hydrogen into helium. 6) Consider the following statements about India‘s
highest herbal park
3. The energy released during nuclear fission is
lower than the energy released during nuclear 1. It is located in Leh district in the UT of
fusion. Ladakh.
Which of the statement(s) given above is/are correct? 2. It is inaugurated to conserve various alpine
species and study their ecology.
a. 1 and 2 only
3. It contains a dedicated section on Ashtavarga
b. 1 and 3 only
species, a group of herbs found in Himalayas.
c. 2 and 3 only
Which of the statements given above are correct?
d. 1, 2 and 3
a. 1 and 2 only
b. 2 and 3 only
4) Match the following:

www.shankariasacademy.com | www.iasparliament.com
91

c. 1 and 3 only 3. Kappa is a variant of Interest and is a sibling of


Delta variant.
d. 1,2 and 3
Which of the statements given above are not correct?
a. 1 and 2 only
7) Consider the following statements about Delhi‘s
Smog Tower b. 2 and 3 only
1. It uses updraft air cleaning system that purifies c. 1 and 3 only
air by sucking it through the fans present at
d. 1,2 and 3
the bottom and filtered air is released at the
top.
2. It cannot filter particles less than 0.5 microns. 10) Adi Tribes, recently in news for naming a frog
species after Adi Hills, are seen in which of the
Which of the statement(s) given above is/are correct?
following States?
a. 1 only
a. Arunachal Pradesh
b. 2 only
b. Madhya Pradesh
c. Both 1 and 2
c. Mizoram
d. Neither 1 nor 2
d. Andhra Pradesh

8) Consider the following statements about Children's


Climate Risk Index Answers
1. It is released by German Watch and UNEP. 1. c

2. According to it, India is placed at extremely The government has recently launched Yuktdhara
high risk countries with flooding and drought portal.
being the repeated environment shocks. Yuktdhara
3. Nepal, Bhutan and Sri Lanka ranked better  It is a geospatial planning portal.
than India.
 Yukt is derived from Yojanam, the planning
Which of the statements given above are correct? and Dhara indicates the flow.
a. 1 only
 It will facilitate planning of new MGNREGA
b. 2 only assets using Remote Sensing and GIS based
information.
c. 3 only
 The portal will serve as a repository of geotags
d. 1,2 and 3
created under various national rural
development programmes i.e. MGNREGA,
Integrated Watershed Management
9) Consider the following statements about classes of
Programme, Per Drop More Crop programme
SARS-CoV-2 variants
and Rashtriya Krishi Vikas Yojana, along with
1. Variant of Interest is known for its increase in field photographs
transmissibility, significant reduction in
Highlights of the portal
neutralization by antibodies and severe
diseases.  This portal integrates wide variety of thematic
layers, multi-temporal high resolution earth
2. Variant of Concern has clear evidence that
observation data with analysis tools.
prevention measures or medical
countermeasures (MCMs) have significantly  Planners will analyse previous assets under
reduced effectiveness relative to previously various schemes and facilitates identification
circulating variants. of new works using online tools.

www.shankariasacademy.com | www.iasparliament.com
92

 Plans prepared will be evaluated by  In this lab, using laser beams, tiny pellets of
appropriate authorities under State deuterium and tritium (heavier isotopes of
Departments. hydrogen) have been fused to form helium and
release energy that very nearly matches the
 Thus, Yuktdhara based plans will be prepared amount of energy input using the lasers.
by grassroot functionary and verified by
appropriate authorities for relevance and  To be functional, a reactor has to produce an
resource allocation. output that is at least tens of times the input
energy.
 This would ensure quality of plan and enable a
long term monitoring of the assets created over Nuclear Fission & Fusion
the years.
 Atomic energy can be released in two ways
2. b
1. by breaking up heavy nuclei, like uranium, into
Shree Narayana Guru (1856–1928) smaller fragments, releasing a whole lot of
energy in the process – Nuclear Fission
 He was also known as Shree Narayana Guru
Swami. 2. by fusing together light nuclei like hydrogen to
form heavier stable nuclei and high-energy
 He was a Hindu saint & social reformer of neutrons which carry a lot of energy that can
India. be harnessed – Nuclear Fusion
 The Guru was born into an Ezhava family, in  Nuclear Fission is happening in established
an era when people from backward nuclear reactors around the world.
communities like the Ezhavas faced social
injustice in the caste-ridden Kerala society.  Stars generate energu through Nuclear Fusion.
 Aruvippuram Movement - The Guru himself  In our Sun, for example, hydrogen is being
consecrate a Shiva linga (idol) in the converted into helium, releasing huge amounts
Aruvippuram temple. of energy.
 When asked about his right to consecrate an  Nuclear fusion is a clean and green route to
idol he replied with his famous quote: I producing energy, as it does not involve any
installed my Siva; not a Brahmin Siva. It remnant radioactive waste products.
later became a famous quote, used against
 The energy released by fission is a million
casteism.
times greater than that released in chemical
 He started a Sanskrit school in Varkala and reactions; but lower than the energy released
poor boys and orphans were given free by nuclear fusion.
education regardless of their caste
 Fusion reactions power hydrogen bombs.
 Advaita Ashram - In 1913, the Guru founded
4. a
an Ashram at Aluva. It was called the Advaita
Ashram. Priya
 The Ashram was dedicated to a great principle  It is an accessibility booklet developed by
– Om Sahodaryam Sarvatra (all human beings NCERT in collaboration with Department of
are equal in the eyes of God). Empowerment of Persons with Disability.
3. d  It aims for ingraining the concept and
significance of accessibility in children right
 So far, Nuclear Fusion devices that show a net
from their formative years itself as a move
energy gain have not been demonstrated in
towards inclusive education.
labs.
Other initiatives in education sector
 An experiment at the U.S. National Ignition
Facility (NIF), within the Lawrence Livermore  SARTHAQ – A flexible, interactive, indicative
National Laboratory, Livermore, California, and inclusive NEP implementation plan.
comes close to demonstrating this.

www.shankariasacademy.com | www.iasparliament.com
93

 NIPUN Bharat Mission – Mission on  It contains a dedicated section on ―Ashtavarga‖


Foundational Literacy and Numeracy species, which is a group of eight herbs found
in the Himalayas.
 Vidya Pravesh – A three months School
Preparation Module  Among the major herbs found in the park,
―Brahmakamal‖ (Saussurea Obvallata), state
 NISHTHA – Capacity building of Secondary flower of Uttarakhand is also present.
teachers
 Interestingly, this flower blooms only one
5. d
night in a year, and that too, after sunset. It is
 Department of Biotechnology, Ministry of also called as King of Himalayan Flowers.
Science and Technology in collaboration with 7. d
BRICS countries is implementing SARS-CoV-2
NGS-BRICS consortium and multi centric India’s First Smog Tower
programme to study the impact of severe
 It was recently inaugurated in Delhi to purify
COVID-19 conditions on TB patients.
air in a 1-km radius around the structure.
 This collaborative study is expected to provide
 The structure is 24 m high with 40 fans in its
valuable co-morbidity data pertaining to
base which can discharge up to 1,000 cubic
pulmonary TB patients with or without
metres per second of air from the tower.
COVID-19 co-infection that is expected to be
generated for better disease management.  The tower uses a ‗downdraft air cleaning
BRICS STI Framework Programme system‘ in which the polluted air is sucked in at
a height of 24 m, and filtered air is released at
 In 2015, the BRICS STI Framework the bottom of the tower, at a height of about 10
Programme (BRICS STI FP) has been m from the ground.
endorsed aiming to support excellent research
on priority areas which can best be addressed  Inside the tower in two layers are 5,000 filters.
by a multinational approach.  The ‗macro‘ layer in the filter traps particles of
 The initiative should facilitate cooperation 10 microns and larger, while the ‗micro‘ layer
among the researchers and institutions in the filters smaller particles of around 0.3 microns.
consortia which consist of partners from at  In China, the updraft system is deployed in
least three of the BRICS countries. which air is sucked in from near the ground
 Since 2016 coordinated calls for multilateral and filtered air is released at the top by
research projects are launched under BRICS propelling upwards by heating and convection.
STI FP inviting researchers from BRICS 8. c
member states to jointly carry out basic,
applied and innovation research projects on Children's Climate Risk Index
multilateral approach.  It is released by UNICEF for the first time.
6. b  India is among 33 ―extremely high risk‖
India’s highest herbal park countries where children are most at risk of
climate change affecting their health and
 The park is located in Uttarakhand's Chamoli education.
district.
 It ranks India at a vulnerable 26th rank and is
 It is built at a height of 11,000 feet and is among 4 South Asian Nations such as
adjacent to the famous Badrinath temple in Pakistan, Bangladesh and Afghanistan.
Mana, which is the last Indian village in
Chamoli bordering China.  In India, flooding and air pollution is
considered as the repeated environmental
 Its aims conserve various medicinally and shocks leading to socio-economic adverse
culturally important alpine species, and consequences for women and children.
facilitate a study on the propagation of these
species, as well as their ecology.

www.shankariasacademy.com | www.iasparliament.com
94

 India's neighbours Nepal is ranked 51st and Sri transport sector in Asia, in line with a well
Lanka 61st. Bhutan is ranked 111th, with below 2-degree pathway.
children at relatively lower risk.
2. It was jointly launched by the NITI Aayog and
9. a World Bank.
Classes of SARS-CoV-2 variants Which of the statement(s) given above is/are correct?
 Variant of Interest - A variant with specific a. 1 only
genetic markers that have been associated with b. 2 only
changes to receptor binding which affect its
diagnosis and are expected to cause unique c. Both 1 and 2
outbreak clusters. d. Neither 1 nor 2
 It is known for it predicted increase in
transmissibility.
2) With respect to NDC Transport Initiative for Asia
 Variant of Concern - A variant for which (TIA 2020-2023), which of the statement given below
there is evidence of an increase in is incorrect?
transmissibility, more severe disease (e.g.,
increased hospitalizations or deaths). a. It aims to facilitate a paradigm shift to zero-
emission transport across Asia.
 It is known for its significant reduction in
neutralization by antibodies generated during b. It is a joint programme of seven organisations
previous infection or vaccination, reduced that will engage China, India, and Vietnam in
effectiveness of treatments or vaccines, or promoting a comprehensive approach to
diagnostic detection failures. decarbonizing transport in their respective
countries.
 Variant of High Consequence – It has
clear evidence that prevention measures or c. The project is part of the Paris Agreement on
medical countermeasures (MCMs) have climate change adopted at COP 21, 2015.
significantly reduced effectiveness relative to d. NITI Aayog is the implementing partner for the
previously circulating variants. India component of the project.
 The Delta and Kappa variants are actually
siblings, the direct descendants of a variant
that earlier used to be referred to as the double 3) With reference to Nidhi Companies, which of the
mutant, or B.1.617. statement given below is incorrect?

10. a a. A Nidhi to be incorporated under the


Companies Act, 2013 shall be a public company
Adi Tribes and shall have a minimum paid up equity share
capital of 5 lakh rupees.
 Adi tribe of Adi-Pasi, Arunachal Pradesh is a
Scheduled Tribe under Article 366 of the b. Every Company incorporated as a ―Nidhi‖ shall
Constitution. have the last words ‗Nidhi Limited‘ as part of its
name.
 They came from southern China in the 16th
century and speak the Tibeto-Burman c. No Nidhi shall carry on the business of
language. borrowing or lending in its own name.
d. Every Nidhi shall ensure that its membership is
not reduced to less than two hundred members at
25-08-2021 any time.
1) Consider the following statements with respect to
Forum for Decarbonizing Transport in India
4) Consider the following statements with respect to
1. It aims at bringing down the peak level of India ASEAN Trade Relationship
Green House Gas (GHG) emissions in

www.shankariasacademy.com | www.iasparliament.com
95

1. ASEAN as a region is the third largest export  India has a massive and diverse transport
destination of Indian engineering products sector, which is also the third most CO2
after European Union (EU) and North emitting sector.
America.
 Data from IEA, 2020; Ministry of
2. Among the ASEAN member nations, Environment Forest and Climate Change, 2018
Singapore and Taiwan are major export suggests that within the transport sector, road
destinations for Indian engineering products. transport contributes to more than 90% of the
Which of the statement(s) given above is/are correct? total CO2 emissions.

a. 1 only 2. c

b. 2 only NDC Transport Initiative for Asia (TIA 2020-


2023)
c. Both 1 and 2
 The NDC Transport Initiative for Asia (NDC-
d. Neither 1 nor 2 TIA) aims to facilitate a paradigm shift to zero-
emission transport across Asia.

5) Consider the following statements:  It is a joint programme of seven organisations


that will engage China, India, and Vietnam in
1. The protection and management of forests is promoting a comprehensive approach to
primarily the responsibility of concerned decarbonizing transport in their respective
state/UT. countries.
2. The permission for mining cannot be granted  The project is part of the International Climate
inside any National Park and Wildlife Initiative (IKI).
Sanctuary.
 The Federal Ministry for the Environment,
3. The mining activities can be permitted in Nature Conservation and Nuclear Safety
Protected Forest (PF) areas by the concerned (BMU) support the initiative on the basis of a
state government subject to prior approval of decision adopted by the German Bundestag.
Central Government.
 Beyond the three countries, on the regional
Which of the statements given above are correct? and global level, the program will maximize
a. 2 only impact by reaching out to additional countries
in Southeast Asia.
b. 1 and 3 only
3. c
c. 2 and 3 only
Nidhi Companies
d. 1, 2 and 3
 They are also called as Mutual Benefit Society.
 Every nidhi company should be registered
Answers under section 406 of the Companies Act, 2013.
1. a  Their purpose is to cultivate the habit of thrift
 NITI Aayog and World Resources Institute and savings among its members, receiving
(WRI), India, jointly launched the ‗Forum for deposits from, and lending to, solely its
Decarbonizing Transport‘ in India as part of members, for mutual benefit.
the NDC-Transport Initiative for Asia (NDC-  A Nidhi to be incorporated under the
TIA) Project. Companies Act, 2013 shall be a public
 The project aims at bringing down the peak company and shall have a minimum paid up
level of GHG emissions in transport sector in equity share capital of five lakh rupees.
Asia in line with a well below 2-degree  Every Company incorporated as a ―Nidhi‖ shall
pathway, resulting in problems like congestion have the last words ‗Nidhi Limited‘ as part of
and air pollution. its name.

www.shankariasacademy.com | www.iasparliament.com
96

 No Nidhi shall carry on the business of chit  However, the mining activities can be
fund, hire purchase finance, leasing finance, permitted in Protected Forest (PF) areas by the
insurance or acquisition of securities issued by concerned state government subject to prior
any body corporate. approval of Central Government under the
provisions of Forest (Conservation) Act, 1980.
 A Nidhi shall not admit a body corporate or
trust as a member  Protection and management of forests is
primarily the responsibility of concerned
 Except as otherwise permitted under these state/UT.
rules, every Nidhi shall ensure that its
membership is not reduced to less than two
hundred members at any time.
26-08-2021
4. a
1) Consider the following statements about SAMRIDH
 Union Minister of State for Commerce and Scheme
Industry has recently inaugurated the ―India-
ASEAN Engineering Partnership Summit‖ 1. It is launched by the Ministry of Skill
organised by Engineering Exports Promotion Development and Entrepreneurship.
Council (EEPC) with the support from 2. Its aim is to create a conducive platform for the
Ministry of External Affairs and Department of growth of software product start-ups.
Commerce.
3. It will provide both funding and enhancing
India ASEAN Relationship their products through skill sets together to
enable startups to become successful.
 ASEAN, with over 15 percent share in India‘s
global engineering shipment, is likely to be a Which of the statements given above is/are correct?
key region to focus with a target of around
a. 1 and 2 only
USD 16 billion of exports for 2021-22.
b. 2 and 3 only
 ASEAN as a region is the third largest export
destination of Indian engineering products c. 1 and 3 only
after EU and North America.
d. 1,2 and 3
 Among the ASEAN member nations,
Singapore and Malaysia are major export
destinations for Indian engineering products. 2) Consider the following statements about Drone
Rules 2021
 This year marks the 25thanniversary of India-
ASEAN dialogue partnership and 10 years of 1. According to the new rules, the quantum of
the Strategic Partnership. fees to be paid for the approval is linked to the
size of the drones.
5. d
2. There are 3 zones in place to restrict the areas
 According to the Union Environment Ministry, where drones can fly.
around 38,846.7 ha of protected forest land
has been diverted for mining projects between 3. Foreign ownership of drones is allowed and
2011 and 2021. security clearance before registration has been
done away with.
 The highest diversion of protected forest took
place in Odisha followed by Chhattisgarh, Which of the statements given above is/are correct?
Madhya Pradesh and Telangana. a. 1 and 2 only
 As per the order of the Hon‘ble Supreme Court b. 2 and 3 only
in 2006 (TN Godavarman Thirumalpad vs
Union of India & Ors) the permission for c. 1 and 3 only
mining cannot be granted inside any National d. 1,2 and 3
Park and Wildlife Sanctuary.

www.shankariasacademy.com | www.iasparliament.com
97

3) Consider the following statements about Havana (SAMRIDH) programme was launched by the
Syndrome Ministry of Electronics & Information
Technology (MeitY).
1. It was first reported in 2016 and has also been
reported in other places such as China and  SAMRIDH will create a conducive platform to
Austria. growing Indian Software Product start-ups
that are ready for acceleration stage to
2. Acute symptoms include hearing loud sounds
enhance their products and secure
and may felt the sensation from a particular
investments for scaling their business.
direction or in a specific location.
Which of the statements given above is/are correct?  It will provide them with funding support,
mentorship and other supports that are
a. 1 only required by startups at this stage.
b. 2 only  The programme is being implemented by
c. Both 1 and 2 MeitY Start-up Hub (MSH).

d. Neither 1 nor 2  The government has invited existing startup


accelerators to partner with and will provide
funding of up to Rs.40 lakh to them.
4) Kadavur Forest Reserve, a sanctuary known for an  Only accelerators that have been in the
endangered Slender Loris is in which of the following incubation business for at least 3 years and
States? supported more than 50 startups, with at least
a. Kerala 10 having a non-public business will be able to
apply for the scheme.
b. Andhra Pradesh
 They must also have operations in India and
c. Tamil Nadu the necessary space and infrastructure to be
d. Karnataka eligible.
2. b

5) Consider the following statements about Nidhi  The central government has notified the Drone
Company Rules 2021, which is much more liberalised
than previous rules.
1. The company should have the minimum paid
up equity share capital of Rs. 5 crore.  The quantum of fees, which was earlier linked
to the size of drone, has been reduced and
2. Sabanayagam committee was constituted to delinked from the size.
suggest changes on policies and regulatory
framework of nidhi companies.  For example, the remote pilot license fee,
which was Rs 3,000 for a large size drone, has
Which of the statements given above is/are correct?
been reduced to Rs 100, which is the fee for all
a. 1 only categories of drones.
b. 2 only  The three zones - yellow, green and red have
been demarcated to tell drone operators where
c. Both 1 and 2
they can and cannot fly their aircraft systems.
d. Neither 1 nor 2
 The government has done away with the need
for security clearance prior to the issuance of a
registration or license.
Answers
 Also, on the part of companies operating
1. b drones, foreign ownership has been allowed.
SAMRIDH Programme
 However, the import of drones will continue to
 Start-up Accelerators of MeitY for pRoduct be regulated by the Directorate General of
Innovation, Development and growth Foreign Trade.

www.shankariasacademy.com | www.iasparliament.com
98

3. c  A company to be incorporated under this Act


as Nidhi shall be a Public Company with a
 Havana Syndrome was first reported in 2016
minimum paid up equity share capital of
in the Cuban city.
5,00,000/-
 The patients reported that they heard strange
 Recently, changes were suggested by
sounds and experienced odd physical
"Sabanayagam Committee " on policies and
sensations in their hotel rooms or homes.
regulatory framework of nidhi companies.
 Acute symptoms – loud symptoms, hearing
loss, intense pressure, nausea, loss of balance.
 Chronic symptoms – Headache, nausea,
27-08-2021
insomnia, depression etc. 1) The Reserve Bank of India (RBI) constituted an
Expert Committee on Primary Urban Co-operative
 No definitive cause has been found. Banks (UCBs) under the chairmanship of N S
 But scientific studies are consistent with Vishwanathan. The committee has released its report
exposure to directed radio frequency energy. recently. Which of the following is not one among its
recommendations?
4. c
a. It suggested for setting up an Umbrella
 Kadavur Reserve Forest is in Tamil Nadu. Organisation (UO) to oversee co-operative banks.
 There is a growing demand that the Tamil b. It suggested for a four-tier structure for the
Nadu government must declare the Kadavur urban cooperative banks (UCBs) depending upon
Reserve Forest a sanctuary for the slender the deposits.
loris.
c. It suggested that the Supervisory Action
 Slender loris (Loris tardigradus) is a shy, Framework should follow a triple indicator
secretive and reclusive species of primate, and approach considering asset quality, Profitability
has nocturnal habits. and capital instead of current, twin-indicator
approach.
 It is native to Southern India and Sri Lanka.
d. It prescribed separate ceilings for home loans
 It is commonly found in the tropical scrubs, and unsecured loans for different categories of
deciduous forests and scrubs as well as the UCBs.
dense hedgerow plantations bordering
farmlands.
1. IUCN - Endangered 2) Which of the following statement is incorrect?
2. CITES - Appendix II a. India has already achieved emission reduction
of 28% over 2005 levels, against the target of 35%
3. Wild Life Protection Act, 1972 - Schedule I to by 2030 committed in its Nationally Determined
provide the highest level of legal protection Contributions (NDC).
5. b b. India has already achieved 100 GW of installed
 Nidhi Company is registered under Renewable Energy Capacity and 38.5% of India‘s
the Companies Act, 2013. installed power generation capacity is based on
clean renewable energy sources.
 It is a non-banking finance company (NBFC)
doing the business of lending and borrowing c. Presently India stands at 4th position in the
with its members or shareholders. world in terms of installed RE capacity, 5th in
Solar and 4th in Wind energy capacity.
 As it is an NBFC, RBI has powers to issue
directives for them related to their deposit d. None of the above
acceptance activities.
 It works on the principle of mutual benefits 3) SUJALAM Campaign, sometimes seen in the news
that are regulated by the Ministry of Corporate recently, aims to?
Affairs.

www.shankariasacademy.com | www.iasparliament.com
99

a. Achieve more Open defecation free (ODF)-Plus 1. c


villages
 The Reserve Bank of India (RBI) constituted
b. Impart clean habits and good sanitation an Expert Committee on Primary Urban Co-
behaviour to school children operative Banks (UCBs) under the
chairmanship of N S Vishwanathan.
c. Achieve 100% internet connectivity in Indian
villages through local cable networks  The committee has submitted its report
d. Engaging NGOs to spread awareness about recently to the RBI.
empowerment schemes for the marginalised Highlights of the report
people
 Umbrella Organisation - The committee
suggested setting up an Umbrella Organisation
4) Which of the following countries will be (UO) to oversee co-operative banks.
participating in the multinational peacekeeping  UO should be financially strong and be well
exercise – ―Shared Destiny-2021‖? governed by a professional board and senior
1. China management, both of which are fit and proper.
2. Pakistan  As an alternative to mandatory consolidation,
the Committee preferred smaller banks
3. India acquiring scale via the network of the UO.
4. USA  Merger - The Committee has suggested that
5. Thailand the UCBs should be allowed to open more
branches if they meet all regulatory
Select the correct answer using the codes given below: requirements.
a. 1, 2 and 5 only  If the UCBs don‘t meet the prudential
b. 1, 3 and 5 only requirements, the RBI should mandatorily
merge or reconstruct the UCBs to resolve
c. 3, 4 and 5 only them.
d. 2, 4 and 5 only
 Four-Tiers - UCBs may be categorised into 4
tiers for regulatory purposes, based on the
banks‘ cooperativeness, availability of capital
5) Consider the following statements with respect to and other factors,
Slender Loris
1. Tier 1 with all unit UCBs and salary earner‘s
1. They are commonly found in the tropical UCBs (irrespective of deposit size) and all
scrubs, deciduous forests and scrubs. other UCBs having deposits up to Rs 100 crore,
2. They are listed under the Schedule I of the 2. Tier 2 with UCBs of deposits between Rs 100
Wildlife (Protection) Act of India, 197 crore - 1,000 crore,
3. The species is native to North-Eastern India 3. Tier 3 with UCBs of deposits between Rs
and Myanmar. 1,000 crore - 10,000 crore
Which of the statements given above are correct? 4. Tier 4 with UCBs of deposits more than Rs
a. 3 only 10,000 crore.
b. 1 and 2 only  The Committee has suggested that the
minimum Capital to Risk-Weighted Assets
c. 2 and 3 only Ratio (CRAR) for them could vary from 9% to
d. 1, 2 and 3 15% and for Tier-4 UCBs the Basel III
prescribed norms.
 It has also prescribed separate ceilings for
Answers home loans, loan against gold ornaments and

www.shankariasacademy.com | www.iasparliament.com
100

unsecured loans for different categories of particularly through creation of 1 million Soak-
UCBs. pits and also other Grey water management
activities.
 Supervisory Action Framework (SAF) -
The SAF should follow a twin-indicator  The Campaign has started from today i.e. 25th
approach instead of triple indicators at present August, 2021 and will continue to run for the
in order to find a time-bound remedy to the next 100 days.
financial stress of a bank.
The key activities that will be organised in the villages
 So, the asset quality and capital should be under this campaign include:
measured only through two indicators NNPA
1. Organizing Community consultations, Khuli
and CRAR.
Baithaks and Gram Sabha meetings to analyze
 If a UCB remains under more stringent stages the current situation
of SAF for a prolonged period, it may have an 2. Pass resolution to maintain ODF sustainability
adverse effect on its operations and may and achieve needed number of soak pits to
further erode its financial position. manage the grey water
2. d 3. Develop a 100 days‘ plan to undertake
 Union Minister of Power and New & sustainability and soak pit construction related
Renewable Energy, and President of activities
International Solar Alliance, Shri RK Singh 4. Construct requisite number of soak pits
attended the ‗INDIA-ISA Energy Transition
5. Retrofit toilets where needed through IEC and
 Dialogue 2021‘ organized by the International community mobilization and
Solar Alliance (ISA) and the Union Ministry of
New and Renewable Energy (MNRE). 6. Ensure all newly emerging Households in the
village have access to toilets.
 In his address he stated that, India has already
achieved emission reduction of 28% over 2005  The disposal of waste water and clogging of
levels, against the target of 35% by 2030 waterbodies in the villages or on the outskirts
committed in its NDC (Nationally determined of the villages remain one of the major
contributions). problems.

 He also stated that the Indian Power Sector  The Campaign would help in management of
have achieved the coveted milestone of 100 the wastewater and in turn would help to
GW of installed Renewable Energy Capacity. revive the waterbodies.

 While 100 GW of capacity has been installed  Furthermore, the campaign would boost the
and operationalized, 50 GW of additional momentum of SBMG phase II activities
capacity is under installation and another 27 through community participation and it will
GW is under tendering process. increase awareness about ODF-plus activities.

 As on 31st July 2021, 38.5% of India‘s installed 4. a


power generation capacity is based on clean Shared Destiny – 2021
renewable energy sources and with this pace
we will reach the target of 40% by 2023.  It is a multinational peacekeeping exercise.

 Presently India stands at 4th position in the  The militaries of China, Pakistan, Mongolia
world in terms of installed RE capacity 5th in and Thailand will take part in this exercise to
Solar and 4th in Wind energy capacity. be organised by the People's Liberation Army
in the central Henan province from September
3. a 6 to 15, 2021.
 The Ministry of Jal Shakti began ‗SUJALAM‘, a  The scenario of the exercise is joint operations
‗100 days campaign‘ as part of the ‗Azadi Ka of multinational peacekeeping forces and the
Amrit Mahotsav‘ celebrations to create more exercise will be held in a close-to-real
and more ODF Plus villages by undertaking battlefield environment set in accordance with
waste water management at village level

www.shankariasacademy.com | www.iasparliament.com
101

the international, professional and realistic  Many parts of this forest has been afforested
combat standards. under the Tamil Nadu Afforestation Project
(TAP) program aided by Japanese
5. b
government.
 A proposal has been sent to the State
government of Tamil Nadu to declare the
Kadavur Reserve Forest in Karur district as a 28-08-2021
sanctuary for Slender Loris.
1) Consider the following statements with respect to e-
Slender loris (Loris tardigradus) Shram Portal
 It is a shy, secretive and reclusive species of 1. It will be the National Database for
primate, and has nocturnal habits. Unorganised workers (Shram Yogis).
 Habit - They are usually solitary but 2. It was launched by the Ministry of Labour and
sometimes found in pairs. Employment.
 It usually travels from the canopy of one tree to Which of the statement(s) given above is/are correct?
another. At times, it comes down to the bushes
a. 1 only
at the ground level to feed.
b. 2 only
 It sleeps by day in the foliage or in a hole or
crevice. It comes out at dusk in search of prey. c. Both 1 and 2
 Location - It is native to Southern India and d. Neither 1 nor 2
Sri Lanka.
 It is commonly found in the tropical scrubs, 2) Consider the following statements with respect to
deciduous forests and scrubs as well as the Stop TB Partnership
dense hedgerow plantations bordering
farmlands. 1. It was setup following the meeting of the First
Session of the Ad Hoc Committee on the
 Food - It eats lantana berries, insects, lizards, Tuberculosis Epidemic held in London in
small birds, tree frogs, tender leaves and buds. March 1998.
 It plays a key role in controlling agricultural 2. India is the current Chair of Stop TB
crop pests in organic farms. Partnership Board.
 Threats - Habitat loss, electrocution of live 3. Amsterdam Declaration, which aims to Stop
wires, road accidents and illegal smuggling. TB, was conceived by the Stop TB initiative in
2000.
Conservation
Which of the statement(s) given above is/are correct?
 IUCN - Endangered
a. 2 only
 CITES - Appendix II
b. 1 and 2 only
 Wild Life Protection Act, 1972 - Schedule I to
provide the highest level of legal protection c. 1 and 3 only
Kadavur Reserve Forest d. 1, 2 and 3

 It is located in Tamil Nadu. It consists of hills


and dense dry areas. 3) Which of the following is the objective of Mission
 Indian bison, spotted deer, mouse deer, Karmayogi?
slender loris, jackal, mongoose, black naped a. Civil Services Capacity Building
hare, wild boar, porcupine, monitor lizard,
pangolin, monkeys, pythons, etc are found in b. Elimination of manual scavenging
the forests. c. Sustaining agricultural productivity

www.shankariasacademy.com | www.iasparliament.com
102

d. Pro-Active Governance and Timely c. 1 and 3 only


Implementation
d. 1, 2 and 3

4) Veligonda Project, sometimes seen in the news


7) Enhanced Access and Service Excellence (EASE), a
recently, aims to irrigate drought-affected areas in
common reform agenda unveiled by the government is
which of the following states?
related which of the following sectors?
a. Telangana
a. Railways
b. Karnataka
b. Banking
c. Maharashtra
c. Micro, Small and Medium Enterprises
d. Andhra Pradesh
d. Start-up

5) Consider the following statements with respect to


8) Consider the following statements about rate of
Supermassive Black Holes
desertification in the country
1. They are difficult to detect because they do not
1. Mizoram in the North East ranks first in the
emit any light.
rate of desertification in the country in the last
2. When the dust and gas fall onto a 15 years.
supermassive black hole, some of the mass is
2. Overall, Central India records the highest
swallowed by the black hole, but some of it is
desertification in terms of Total Geographical
converted into energy.
Area (TGA) in the country.
3. They reveal their presence by emitting
Which of the statements given above is/are correct?
electromagnetic radiation.
a. 1 only
Which of the statements given above are correct?
b. 2 only
a. 1 and 2 only
c. Both 1 and 2
b. 1 and 3 only
d. Neither 1 nor 2
c. 2 and 3 only
d. 1, 2 and 3
9) Consider the following statements about Manthan
2021
6) Consider the following statements about Fair and
1. It is an online hackathon organised by the All
Remunerative Price (FRP)
India Council for Technical Education
1. The concept of FRP was introduced in 2009-10 (AICTE).
by amending Sugarcane (Control) Order, 1966.
2. It is an initiative to identify new digital
2. FRP is linked to a basic recovery rate of sugar technology innovations for online education in
in which farmers are being paid higher India.
premium for higher recoveries of sugar from
Which of the statements given above is/are not
sugarcane.
correct?
3. The farmers are required to wait till the end of
a. 1 only
the season or announcement of profits by
sugar mills or the Government. b. 2 only
Which of the statements given above are correct? c. Both 1 and 2
a. 1 and 2 only d. Neither 1 nor 2
b. 1 and 2 only

www.shankariasacademy.com | www.iasparliament.com
103

10) Which of the following statements is not correct  Founded in 2001, the Partnership's mission is
about the Rubber Census? to serve every person who is vulnerable to TB
and ensure that high-quality diagnosis,
a. The first ever Rubber Census is launched by the
treatment and care is available to all who need
Rubber Board covering using Digitalized Mobile
it.
Application called ‗RUBAC‘.
b. The Rubber Act of 1947 stipulates mandatory  The organization was conceived following the
registration of rubber plantations in the country. meeting of the First Session of the Ad Hoc
Committee on the Tuberculosis Epidemic held
c. Natural Rubber production in the country in London in March 1998.
recorded a downward momentum due to
lockdowns and a fall in consumption.  In March 2000, the Stop TB Initiative
produced the Amsterdam Declaration to Stop
d. Kerala alone accounts for about 60% of the TB, which called for action from ministerial
Rubber area in the country. delegations of 20 countries with the highest
burden of TB.
 It has 1500 partner organizations which
Answers
include international, non-governmental and
1. c governmental organizations and patient
groups.
E-Shram Portal
 The Secretariat is hosted by United Nations
 It was launched by the Ministry of Labour and
Office for Project Services (UNOPS), based at
Employment.
Geneva, Switzerland.
 E-Shram Portal will be the National Database Stop TB Partnership Board
of Unorganised workers (Shram Yogis).
 It provides leadership and direction, monitors
 The portal will do the targeted identification of the implementation of agreed policies, plans
the unorganized workers, and will help take and activities of the Partnership, and ensures
social security schemes to their doorstep. coordination among Stop TB Partnership
 An estimate says that 92% of India's workforce components.
of 500 million is unorganised, often deprived  India is the current chair of Stop TB
of minimum wages and any form of social Partnership chaired by Union Minister for
security. Health and Family Welfare Shri Mansukh
 The database will include construction Mandaviya.
workers, migrant workers, gig and platform India’s Targets
workers, street vendors, domestic workers,
agriculture workers, migrant workers and  India has committed to eliminating TB in the
similar other unorganised workers. country by 2025, five years ahead of the global
deadline of 2030.
 Workers can register themselves on the portal,
and they will be provided with an e-SHRAM  The Government of India's National Strategic
card that will have a 12 digit unique number. Plan for TB Elimination 2017-2025 outlines an
ambitious agenda and targets which exceed
 This move will integrate the Centre‘s social even the aspirational ones set by the World
security schemes. Health Organization's (WHO) End TB Strategy
2. d for the world.
Stop TB Partnership 3. a

 The ‗Stop TB Partnership‘ is mandated to National Programme for Civil Services


eliminate Tuberculosis as a public health Capacity Building (NPCSCB) (Mission
problem. Karmayogi)
 It was approved by the Government on
2nd September, 2020 with the objective of

www.shankariasacademy.com | www.iasparliament.com
104

enhancing governance through Civil Services  They are difficult to detect because they do not
Capacity Building. emit any light.
 Mission Karmayogi aims to prepare the Indian  But they can reveal their presence by
Civil Servant for the future by making him interacting with their surroundings.
more creative, constructive, imaginative,
innovative, proactive, professional,  When the dust and gas from the surroundings
progressive, energetic, enabling, transparent fall onto a supermassive black hole, some of
and technology-enabled. the mass is swallowed by the black hole, but
some of it is converted into energy and emitted
 NPCSCB has been carefully designed to lay the as electromagnetic radiation that makes the
foundations for capacity building for Civil black hole appear very luminous.
Servants so that they remain entrenched in
Indian Culture and sensibilities and remain  They are called active galactic nuclei (AGN)
connected, with their roots, while they learn and release huge amounts of ionized particles
from the best institutions and practices across and energy into the galaxy and its
the world. environment.

 The training of Civil Servants at various  Both of these ultimately contribute to the
Academies is being restructured to include growth of the medium around the galaxy and
optimum use of the digital learning platform of ultimately the evolution of the galaxy itself.
Integrated Government Online Training  Many Active Galactic Nuclei (AGN,
(iGOT). supermassive black hole at the centre of a
4. d galaxy) pairs have been detected in the past,
but triple AGN are extremely rare.
 The Telangana government had recently wrote
to the Union water resources ministry 6. a
opposing the Centre‘s funding for Andhra Fair and Remunerative Price (FRP)
Pradesh‘s Veligonda project.
 Union Cabinet has recently approved the
 They claimed that the project supposedly highest ever FRP for sugarcane for the next
diverts water from the Krishna river outside its marketing year starting October 2021.
basin.
 With the amendment of the Sugarcane
 The Veligonda project in Andhra will be (Control) Order, 1966 in 2009, the concept of
undertaken in the Prakasam district with the Statutory Minimum Price (SMP) of sugarcane
hope to irrigate drought-affected areas in the was replaced with the ‗Fair and Remunerative
districts of Prakasam, Nellore, and Kadapa by Price (FRP)‘ of sugarcane for 2009-10.
diverting floodwater from the Krishna river to
the Srisailam reservoir.  It is decided on the basis of the
recommendations of the Commission for
5. d Agricultural Costs and Prices (CACP) in
 Indian researchers have discovered three consultation with the State Governments and
supermassive black holes from three galaxies after taking feedback from associations of
merging together to form a triple active sugar industry.
galactic nucleus, a compact region at the center  Under the FRP system, the farmers are not
of a newly discovered galaxy that has a much- required to wait till the end of the season or for
higher-than-normal luminosity. any announcement of the profits by sugar mills
 This rare occurrence in our nearby Universe or the Government.
indicates that small merging groups are ideal  The FRP is linked to a basic recovery rate of
laboratories to detect multiple accreting sugar, with a premium payable to farmers for
supermassive black holes and increases the higher recoveries of sugar from sugarcane.
possibility of detecting such rare occurrences.
Supermassive Black Holes

www.shankariasacademy.com | www.iasparliament.com
105

 This is to ensure that higher sugar recoveries  According to the recent report by the ISRO,
are adequately rewarded and to account for North Eastern States are desertifying most
variations amongst sugar mills. rapidly in the country.
7. b  Mizoram has been desertifying at the fastest
rate in the country.
EASE 4.0
 Land degradation and desertification
 Enhanced Access and Service Excellence
increased 2.8 times in the state in the 15-year
(EASE) is a common reform agenda for Public
period studied.
Sector Banks (PSBs), recently unveiled by the
Ministry of Finance. 9. b
 It is aimed at institutionalizing clean and Manthan 2021
smart banking.
 It is an online hackathon organised by The
 The first edition was launched in Bureau of Police Research and Development
January,2018 which aimed at enhancing the (BPR&D), in coordination with the All India
ease of banking in all customer experiences, Council for Technical Education (AICTE).
using technology, alternate data, as well as
analytics.  It is for identification of innovative concepts
and technology solutions to address the
8. a challenges faced by intelligence agencies.
Desertification  It will be held in 2 phases in which the
participants will have to develop digital
 Land degradation is defined as decline in
solutions under six themes using new
productivity of land in terms of biodiversity
technologies like artificial intelligence, deep
and economy, resulting from various causes,
learning, augmented reality and machine
including climate and human dominance,
learning.
leading to loss of ecosystem.
10. c
 Desertification is a type of land degradation in
which a relatively dry region becomes Rubber Census
increasingly arid, typically losing its water
bodies as well as vegetation and wildlife.  The first ever nationwide census on Rubber is
launched by the Rubber Board.
 It covers 16 states to prepare a comprehensive
database on rubber plantations and rubber
growers using a mobile application called
RUBAC.
 It include all seven states in the North East,
Kerala, Tamil Nadu, Karnataka, Andhra
Pradesh, Odisha, Maharashtra, Goa and West
Bengal.
 Objectives - ascertaining the actual area under
rubber, new-planted area, re-planted area, the
age profile of trees, discarded area over the
years, level of adoption of new clones, size of
holdings and details of tappers.
 Kerala accounts for 67 % of rubber area in the
country, of which kottayam alone accounts for
1/5th of the area.

www.shankariasacademy.com | www.iasparliament.com
106

30-08-2021 Which of the statement(s) given above is/are correct?


1) With respect to Bharat Series (BH-series), launched a. 1 only
recently, consider the following statements: b. 2 only
1. It facilitates free movement of personal c. Both 1 and 2
vehicles across States/UTs of India upon
relocation to a new State/UT. d. Neither 1 nor 2
2. This facility will be available to all Indian
Citizens and not available to foreigners. 4) With reference to Real Estate Investment Trusts
3. Under section 47 of the Motor Vehicles Act, (REITs), which of the following statement is incorrect?
1988, a person is allowed to keep the vehicle a. These are investment vehicles that pool investor
for not more than 12 months in any state other money and use it to buy a portfolio of real estate
than the state where the vehicle is registered. assets
Which of the statement(s) given above is/are correct? b. The structure of a REIT is similar to a mutual
a. 1 only fund
b. 1 and 2 only c. The minimum investment amount in a REIT
should be Rs. 50,000
c. 1 and 3 only
d. REITs need to mandatorily distribute 90% of
d. 1, 2 and 3 their income to unit-holders

2) With respect to Association of Renewable Energy 5) With respect to National Technical Advisory Group
Agencies (AREAS), consider the following statements: on Immunization (NTAGI), consider the following
1. Union Minister for New & Renewable Energy statements:
is the ex-officio President of the Association. 1. It is India‘s apex advisory body on
2. All State Nodal Agencies (SNAs) are member immunization established in 200
of the Association. 2. It will be chaired by the Union Minister for
3. Transaction of business will be carried out by Health & Family Welfare.
an Executive Committee (EC), presided over Which of the statement(s) given above is/are correct?
by the Union Minister for New & Renewable
Energy. a. 1 only
Which of the statement(s) given above is/are correct? b. 2 only
a. 2 only c. Both 1 and 2
b. 1 and 2 only d. Neither 1 nor 2
c. 2 and 3 only
d. 1, 2 and 3 6) Consider the following statements about
Refrigerants
1. Carbondioxide is a natural refrigerant and it
3) With reference to EBITDA, consider the following becomes supercritical at 310c which makes it
statements: viable to be used as a refrigerant.
1. It is a measure of a company's overall financial 2. Synthetic refrigerants have low global warming
performance and in some circumstances can potential but consume more electricity making
be used as an alternative to net income. them less efficient.
2. The measure includes interest, taxes and Which of the statements given above is/are correct?
excludes depreciation and amortization.
a. 1 only

www.shankariasacademy.com | www.iasparliament.com
107

b. 2 only Which of the statements given above is/are correct?


c. Both 1 and 2 a. 1 only
d. Neither 1 nor 2 b. 2 only
c. Both 1 and 2
7) Consider the following pairs with respect to Military d. Neither 1 nor 2
Logistics Support Agreements
1. Russia - Acquisition and Cross-Servicing
10) Consider the following statements about Morheen
Agreement (ACSA)
Yoga Mat
2. Japan - Reciprocal Exchange of Logistics
1. It is a 100% biodegradable and compostable
agreement (RELOS)
mat made from Mangrove species.
3. USA - Logistics Exchange Memorandum of
2. It is named after a resident bird of Deepor Beel
Understanding (LEMOA)
Wildlife sanctuary.
4. France - Basic Exchange & Cooperation
Which of the Statements given above is/are correct?
Agreement for Geo-spatial Cooperation
(BECA) a. 1 only
Which of the above given pairs are incorrectly b. 2 only
matched?
c. Both 1 and 2
a. 1, 2 and 4 only
d. Neither 1 nor 2
b. 2 and 3 only
c. 1 and 3 only
Answers
d. 2 and 4 only
1. c
Re-registration of a vehicle
8) Recently, fossil of amphibious four-legged whale
dates back 43 million years has been discovered in a  Under section 47 of the Motor Vehicles Act,
site called Whale Valley. It is a UNESCO World 1988, a person is allowed to keep the vehicle
Heritage site which contains fossil remains of for not more than 12 months in any state other
archaeoceti, a suborder of whales. It represents the than the state where the vehicle is registered,
major story of evolution of whales from a land based but a new registration with the new state-
animal to ocean-going mammal. The site is located in? registering authority has to be made within the
stipulated time of 12 months.
a. Madagascar
 A passenger vehicle user takes the following
b. Egypt steps to re-register a vehicle:
c. Israel 1. No Objection Certificate from the Parent State
d. Greece for assignment of a new registration mark in
another state.
2. Assignment of new registration mark after the
9) Consider the following statements about Nine Dash road tax on prorata basis is paid in the new
Line State
1. It is a straight line representing China‘s claim 3. Application for refund of the road tax in the
over 90% areas in South China Sea that traces parent State on pro rata basis.
its origin back to 1947.
 This provision to get refund from the parent
2. It is a continuous line meaning that other State on pro rata basis is a very cumbersome
countries cannot pass through the lines freely process and varies from one State to another.
without its permission

www.shankariasacademy.com | www.iasparliament.com
108

BH Series Vehicle Marking Association of Renewable Energy Agencies of


States (AREAS)
 In order to facilitate seamless transfer of
vehicles, the Ministry of Road Transport &  It has been formed and registered as a society
Highways has introduced a new registration in 2014 under Society Registration Act 1860.
mark for new vehicles i.e. ―Bharat series (BH-
series)‖.  Union Minister for New & Renewable Energy
is the Ex-Officio Patron of the Association.
 This scheme will facilitate free movement of
personal vehicles across States/UTs of India  Secretary, MNRE is the ex-officio President of
upon relocation to a new State/UT. the Association.

 A vehicle bearing this registration mark shall  All SNAs are member of the Association.
not require assignment of a new registration  Transaction of business of the AREAS to be
mark when the owner of the vehicle shifts from carried out by an Executive Committee (EC)
one State to another. which is presided over by the President of the
 This vehicle registration facility will AREAS.
be available on voluntary basis to Defense  Vice President to be elected by the members
personnel, employees of Central Government/ and Joint Secretary, MNRE to be the ex-officio
State Government/ Central/ State Public member of EC.
Sector Undertakings and private sector
companies/organizations, which have their  In addition, Executive Director, two SNAs
offices in four or more States/Union from each zone (North, South, East, West and
territories. North-East, and UTs) are also members of the
executive committee.
 The motor vehicle tax will be levied for two
years or in multiple of two. 3. a

 After completion of the fourteenth year, the Earnings before interest, taxes, depreciation,
motor vehicle tax shall be levied annually and amortization (EBITDA)
which shall be half of the amount which was  It is a measure of a company's overall financial
charged earlier for that vehicle. performance and is used as an alternative to
net income in some circumstances.
 EBITDA is now commonly used to compare
the financial health of companies and to
evaluate firms with different tax rates and
depreciation policies.
 EBITDA, however, can be misleading because
2. a it strips out the cost of capital investments like
property, plant, and equipment.
 Ministry of New & Renewable Energy (MNRE)
is the nodal agency at the central level for  Among its drawbacks, EBITDA is not a
promotion of grid-connected and off-grid substitute for analyzing a company's cash flow
renewable energy in the country. and can make a company look like it has more
money to make interest payments than it really
 Ministry‘s programmes are implemented in does.
close coordination with State Nodal Agencies
(SNAs) for renewable energy (RE).  EBITDA also ignores the quality of a
company's earnings and can make it look
 MNRE initiated the Association of Renewable cheaper than it really is.
Energy Agencies of States (AREAS) for SNAs
to interact and learn from each other‘s 4. c
experiences.  SEBI has reduced the minimum application
value of REITs and InvITs, and revised trading
lot to one unit for these emerging investment

www.shankariasacademy.com | www.iasparliament.com
109

instruments to make them attractive for retail  The NTAGI Secretariat was established in
investors. 2013, under MoHFW to provide
technomanagerial support to NTAGI and STSC
 The minimum application value has been cut
and its working groups.
down to the range of Rs 10,000-15,000 for
both REITs and InvITs, compared to the  The NTAGI secretariat is tasked with
earlier requirement of Rs 50,000 for REITs undertaking technical review of scientific
and Rs 1 lakh for InvITs. evidence on matters related to immunization
policy and programmes.
Real Estate Investment Trust (REIT)
6. b
 These are investment vehicles that pool
investor money like mutual funds and use it to Refrigerants
buy a portfolio of real estate assets.
 There are two kinds of refrigerants.
 They manage these assets to generate a regular
1. Natural refrigerants - Ammonia,
income and capital appreciation.
Hydrocarbons such as propane and
 The structure of a REIT is similar to a mutual butane, and carbon dioxide.
fund.
2. Synthetic refrigerants - r-134a, r-152a.
 It has three-tier structure — a sponsor, who is
 Among the natural refrigerants, carbon dioxide
responsible for promoting the REIT with his
is the best because hydrocarbons can catch
own capital; a fund management company
fire.
which is responsible for selecting and
operating the properties; and the trustee, who  However, Carbon dioxide has a problem. It
ensures that the money is managed in the becomes ‗supercritical‘ at 310c, meaning that it
interest of unit-holders. exists in a state where there is no distinction
between gaseous and liquid states.
 As per SEBI‘s guidelines, REITs need to
mandatorily distribute 90 per cent of their  A refrigerant must be liquefiable but Carbon
income to unit-holders. The distribution could dioxide cannot liquefy, no matter how much
be in the form of dividend or interest income pressure is applied.
or both.
 Thus, inorder to be used as refrigerant, heat
 Investors‘ awareness and participation had exchangers must be arranged in such a way
been slowly improving since the listing of the that cold temperature available within the
first REIT in the country. system is used to liquefy the carbon dioxide.
5. a 7. a
National Technical Advisory Group on Military Logistics Support Agreements
Immunization (NTAGI)
 India has signed the following 3 foundational
 It was established by an order of the Ministry pacts with USA
of Health and Family Welfare (MoHFW) in
2001. 1. Logistics Exchange Memorandum of
Agreement (LEMOA)
 As India‘s apex advisory body on
2. Communications Interoperability and Security
immunization, the NTAGI provides guidance
Memorandum of Agreement (CISMOA)
and advice to the MoHFW on provision of
vaccination and immunization services for the 3. Basic Exchange and Cooperation Agreement
effective control of vaccine preventable for Geo-spatial Cooperation (BECA)
diseases in the country.
 India and Japan has signed Acquisition and
 It is chaired by Secretary Health, MoHFW and Cross-Servicing Agreement (ACSA) in 2020.
Co-chaired by Secretary Department of
Biotechnology and Secretary Department of  India is expected to sign Reciprocal Exchange
Health Research. of Logistics agreement (RELOS) with Russia
during the upcoming India-Russia meet.

www.shankariasacademy.com | www.iasparliament.com
110

8. b
Whale Valley
 Wadi Al-Hitan, Whale Valley, is in the Western
Desert of Egypt.
 It contains invaluable fossil remains of the
earliest, and now extinct, suborder of whales,
the archaeoceti.
 The fossils represent one of the major stories
of evolution as the emergence of the whale as
an ocean-going mammal from a previous life
as a land-based animal.
 Other fossil material in the site makes it
possible to reconstruct the surrounding
environmental and ecological conditions of the
time.
 It was inscribed in the UNESCO World
Heritage site in 2005.
9. d
10. b
Nine Dash Line
Morheen Yoga Mat
 It is a U-shaped line that owes its origin back
to 1947, demarcated by Chinese geographer  It is a 100% biodegradable and compostable
Yang Huairen. yoga mat developed from water hyacinth.

 It is a discontinuous line meaning that other  The intervention was triggered through an
countries can pass through it freely. initiative by North East Centre for Technology
Application and Reach (NECTAR), an
 It includes the disputed Paracel and Spratly autonomous body under Department of
islands. Science & Technology (DST).
 It represents China‘s historical claim over 90%  It is named after Kam Sorai (Purple moorhen,
of the regions in South China Sea. a resident bird of Deepor Beel Wildlife
sanctuary in Assam).
 China included it in its map in 2009 when it
submitted the document in UN during a
dispute with Vietnam.
31-08-2021
 It is deemed by most countries as being
inconsistent with the United Nations 1) Consider the following statements with respect to
Convention on the Law of the Sea (UNCLOS). POSHAN Abhiyaan
1. It is a flagship programme to improve
 UNCLOS gives States the right to establish a
nutritional outcomes for children, adolescent
territorial sea up to only 12 nautical miles.
Girls, pregnant women and lactating mothers.
2. The programme will be funded 50% by the
Government of India and 50% by World Bank
or other Multinational Development Banks.
3. Every year, the month of October is celebrated
as POSHAN Maah across the country.
Which of the statements given above are correct?
a. 1 only

www.shankariasacademy.com | www.iasparliament.com
111

b. 1 and 2 only 5) Consider the following statements with respect to


Geological Survey of India (GSI)
c. 2 and 3 only
1. It was set up in 1851 primarily to find coal
d. 1, 2 and 3
deposits for the Railways.
2. Presently, GSI is an attached office to the
2) Which of the following is not an initiative of Ministry of Mines.
Ministry of Micro, Small & Medium Enterprises
Which of the statement(s) given above is/are correct?
(MSME)?
a. 1 only
1. Champions Portal
b. 2 only
2. Udyam Registration
c. Both 1 and 2
3. National SC-ST Hub
d. Neither 1 nor 2
4. Niryat Bandhu Scheme
Select the correct answer using the codes given below:
6) Consider the following statements about New rules
a. 1 only
on poultry farms
b. 4 only
1. It defines large farmers as one who have more
c. 3 and 4 only than 1 lakh birds.
d. 1, 2 and 3 only 2. It states that a poultry farm should be set up
500m away from the residential area and
100m away from rivers, lakes and canals.
3) Which of the following are the applications of 3. Poultry is considered as ―Green‖ by Central
Hydrogels? Pollution Control Board (CPCB) and exempt
1. Soft robotics from air, water and environmental protection
laws.
2. Tissue engineering
Which of the statements given above is/are correct?
3. Wastewater treatment
a. 2 only
4. Regenerative medicine
b. 1 and 2 only
Select the correct answer using the codes given below:
c. 1 and 3 only
a. 1 and 2 only
d. 1, 2 and 3
b. 3 and 4 only
c. 2, 3 and 4 only
7) Consider the following statements about Pashmina
d. 1, 2, 3 and 4 Shawls
1. Pashmina comes from an animal fibre
4) India‘s Highest Herbal Park was recently Cashmere, derived from the Shahtoosh wool of
inaugurated in which of the following states? Chiru Antelope.
a. Tamil Nadu 2. Changpa tribes from Changthang region of
Ladakh rear the goat for its wool.
b. Uttarakhand
3. Kashmiri Pashmina was given the GI tag in the
c. Arunachal Pradesh year 2013 owing to its unique methods of
d. Jammu & Kashmir making.
Which of the statements given above is/are correct?
a. 1 only

www.shankariasacademy.com | www.iasparliament.com
112

b. 2 only b. 2 only
c. 2 and 3 only c. 1 and 2 only
d. 1, 2 and 3 d. 1, 2 and 3

8) Consider the following statements about Leaded Answers


Petrol
1. b
1. According to UNEP, World has completely
eradicated the use of Leaded Petrol. PM’s Overarching Scheme for Holistic
Nourishment (POSHAN) Abhiyan
2. Leaded Petrol mainly affects the emission
control system such as catalytic converters.  It is a flagship programme to improve
nutritional outcomes for children, pregnant
3. India‘s phase down of leaded petrol got women and lactating mothers.
completed way back in 2000.
 It targets to reduce stunting, undernutrition,
Which of the statements given above is/are correct? anemia and reduce low birth weight by 2%,
a. 2 only 2%, 3% and 2% per annum respectively.
b. 1 and 2 only  It will be funded 50% by the Government of
India and 50% by World Bank or other
c. 1 and 3 only Multinational Development Banks.
d. 1, 2 and 3
 The Executive Committee is the Apex body for
all Nutrition related activities under the
POSHAN Abhiyaan.
9) The Tiger Reserve is located in Indo-Nepal Border.
The river Gandak and its tributaries inundate and  National Council on India‘s nutritional
causes flood every year in this reserve. It is the only challenges under chairmanship of Vice
tiger reserve from the State. It comprises of both Chairman, NITI Aayog, has been set up under
National Park and a wildlife sanctuary. The above the POSHAN Abhiyaan.
statements describe which of the following Tiger
Reserves?  In order to ensure community mobilization
and bolster people‘s participation, every year,
a. Pilibhit Tiger Reserve the month of September is celebrated as
POSHAN Maah across the country.
b. Dudhwa Tiger Reserve
c. Valmiki Tiger Reserve  The POSHAN Maah intends to achieve the
holistic goals of PoshanAbhiyaan with
d. Jim Corbett Tiger Reserve swiftness in a harmonized manner.
2. b
10) One District One Product Scheme aims to promote  Ministry of MSME has undertaken
exports and improve market accessibility by identifying interventions to enhance MSME ecosystem in
and solving problems in its supply chain. A bunch of India.
products has been selected for its phase-1. Consider the
following pairs of products with its respective States Some of the key reforms introduced by Ministry of
MSME are:
1. Markhana Marbels – Rajasthan
 Revision of MSME definition: In line with
2. Jamnagari Bandhani (Handloomed Textile Government of India's top focus on energizing
Product) - Gujarat MSMEs in the country, Government of India
3. Lakadong Turmeric – Odisha approved the upward revision of MSME
definition in 2020 under the Aatmanirbhar
Which of the above pairs is/are correctly matched? Bharat Package.
a. 1 only

www.shankariasacademy.com | www.iasparliament.com
113

 The Government revised the MSME one place, Enterprise Development Centres
classification by inserting composite criteria of (EDCs) have been conceptualized.
both investment and annual turnover.
 Till date Ministry of MSME has set up 102
 Udyam Registration: Udyam is an online EDCs across India.
and simplified procedure of filing of
 The aim of these centers is to build a network
registration which enables MSMEs to obtain
of entrepreneurial leaders by providing
registration without any documentation and
professional mentoring and handholding
fees.
support services to existing as well as aspiring
 It is a globally benchmarked process and a MSMEs with special focus on rural enterprises
revolutionary step towards Ease of Doing on continuous basis.
Business.
3. d
 Ministry of MSME has also commenced API
 A group of researchers from Indian Institute of
integration of Udyam Registration portal with
Technology (IIT), Guwahati have developed
GeM so that MSMEs can participate in
hydrogel-based electrodes that could enhance
Government procurement easily.
the performance of an energy storage device.
 Champions Portal: It is an online platform
 They have developed the hydrogel electrodes
to help and handhold the MSMEs especially in
by simple room temperature process in which
this difficult time.
graphene and MXene spontaneously assemble
 It is an ICT based technology system aimed at themselves over metal plates within a water
making the smaller units big by solving their medium.
grievances, encouraging, supporting, helping
 Graphene, a single atom thin carbon sheet,
and handholding throughout the business
stores charge on its surface via physical
lifecycle.
adsorption, known as electrical double layer
 The platform facilitates a single window mechanism (ELDC).
solution for all needs of MSMEs.
 Whereas, MXene, nanosheets of titanium
 National SC-ST Hub (NSSH): It has been carbide, stores charge via both ELDC and
launched to promote entrepreneurship culture chemical reaction on its surface, known as
in the SC-ST community and fulfill the 4% pseudo-capacitance.
procurement target mentioned in the Public Hydrogels
Procurement Policy order, 2018.
 They are three-dimensional (3D) cross-linked
 To boost entrepreneurship among SC/ST polymer networks, which can absorb and
population and for maximum on-ground retain large amount of water.
penetration, several interventions have been
undertaken to cater to the challenge of market  A hydrogel is a porous framework of
linkages, finance facilitations, capacity interconnected materials, in which water
building etc. remains stably locked within the pores.
 Procurement Policy: For providing  They have been applied in a wide range of
marketing support to MSMEs, all Central biomedical and engineering applications,
Ministries/Government Departments and ranging from tissue engineering and
CPSEs are required to procure 25% of their regenerative medicine to wastewater treatment
annual requirements of goods and services and soft robotics.
from MSEs including 4% from MSEs owned by
4. b
SC/ST and 3% from MSEs owned by women
entrepreneurs under the Public Procurement India’s Highest Herbal Park
Policy.
 India‘s Highest Herbal Park was recently
 Establishment of Enterprise inaugurated at Mana, Uttarakhand.
Development Centers (EDCs): With a view
to provide Information related to MSMEs at

www.shankariasacademy.com | www.iasparliament.com
114

 The main aim of the herbal park is to conserve New Rules on Poultry Farms
medicinally and culturally important alpine
 It classifies poultry farmers based on the
species, and to facilitate a study on the
number of birds in the farm
propagation of these species, as well as their
ecology. o Small - 5,000-25,000 birds
 Most of the species of herbal plants conserved o Medium - 25,000 – 1 lakh birds
in this park built by Uttarakhand government
o Large – more than 1 lakh birds
are,
1. Found in high alpine areas in the Himalayan  A farm should be set up 500 metres away from
region, a residential area, 100 metres from rivers,
lakes, canals and drinking water sources, 100
2. Included in the ―red list‖ of the IUCN, and metres from national highways and 10-15
metres from village footpaths and rural roads.
3. Declared ―endangered and threatened‖ by
State Biodiversity board.  In 2015, Central Pollution Control Boards
(CPCB) has classified Poultry, hatchery and
 The park is categorised into 4 sections -
piggery as green and exempted them from air,
Sections for ―Ashtavarga‖ species, Saussurea
water and environmental protection laws.
species, species associated with Badrinath
(Lord Vishnu), and for assorted alpine species.  In the new guideline, a farmer of medium-
sized poultry farm will have to obtain a
 The land for the project was provided by Mana
certificate of Consent from state pollution
Van panchayat under the Compensatory
control board or Committee under the Water
Afforestation Fund Act (CAMPA).
Act, 1974 and the Air Act, 1981.
5. c
 The permission will be valid for 15 years.
Geological Survey of India (GSI)
 Other guidelines include –
 It was founded in 1851 primarily to find coal
deposits for the Railways.  Poultry farm need to have ventilated room

 One of the oldest Surveys of the world, the  Burials of dead birds to be done 3m above the
history of Geological Survey of India is ground water level
synonymous with history of development of  Proper space between birds and protection
the infrastructure and industries in India. from rodents and flies
 The main functions of GSI relate to creation  Water from the farm must be collected in a
and updation of national geo-scientific tank to be later used for horticulture
information and mineral resource assessment.
 Poultry faeces, which emit gaseous ammonia,
 These objectives are achieved through ground hydrogen sulphide and methane, do not mix
surveys, air-borne and marine surveys, with running water or any other pesticide
mineral exploration, multidisciplinary
geoscientific, geo-technical, geo-environmental 7. c
and natural hazards studies, glaciology, Pashmina Shawls
seismotectonics, and carrying out fundamental
research.  Pashmina comes from an animal fibre
Cashmere, derived from the Changthangi goat
 GSI, headquartered at Kolkata, has six of Ladakh.
Regional offices located at Lucknow, Jaipur,
Nagpur, Hyderabad, Shillong and Kolkata and  Changthang is the land of the nomads, located
State Unit offices in almost all States of the east of Leh, about 14600 m above sea level and
country. winter temperature can drop to −400c.
 Geological Survey of India is an attached office  Changpa are herder nomadic tribes belong to
of the Ministry of Mines. the sub-sect of larger Buddhist community in
the Ladakh.
6. b

www.shankariasacademy.com | www.iasparliament.com
115

 They rear sheep in harsh climates for meat and  It is situated in Champaran District, North
Pashmina goats for wool primarily. West corner of the State of Bihar.
 A GI Tag for Pashmina shawls was given in the  In north, the protected areas are bordered by
year 2013 owing to its geographic origin and Nepal while the Indian state Uttar Pradesh
unique methods of making. bounds the Wildlife Sanctuary from western
side.
 Recently, initiatives have been taken to re-
engage women artisans in critical production  It comprises of the Valmiki National Park and
process by doubling wages and announcing Valmiki Wildlife Sanctuary.
MSPs for GI-certified products by the
Directorate of Handicrafts and Handlooms,  The river Gandak and its tributaries inundate
Kashmir. the reserve every year.
10. c
 Shahtoosh refers to the fine wool made from
the undercoat of the Chiru Goat, a Tibetan Lakadong Turmeric
antelope, which is an endangered species
under CITES.  It is the world‘s finest turmeric from the State
of Meghalaya.
 Therefore, Shahtoosh is banned in most of the
countries in the world.  It has the average curcumin content
(ingredient that gives turmeric its healing
8. d properties) of a high 7%.
Leaded Petrol  Other Selected Products –
 According to UNEP, globally, automotive fuel  Kolhapuri Chappals from Karnataka and
is completely lead-free now. Not a single fuel Maharashtra
outlet sells leaded petrol anymore anywhere.
 Kashmiri Papier Mâche
 Algeria was the last country to use the fuel.
 Blue Pottery from Jaipur
 This feat is achieved after UNEP-led global
Partnership for Clean Fuels and Vehicles  Markhana Marbels from Nagaur in Rajasthan
(PCFV) began its campaign in 2002 to  Naga Mircha, hot chilli from Nagaland
eliminate lead in petrol.
 Machine parts from Tamil Nadu, and
 Fumes from petrol vehicles using leaded petrol
have been a significant source of lead  Pharmaceutical produce from Andhra Pradesh
exposures which affects multiple body systems
and is particularly harmful to young children.
 It affects the brain, liver, kidneys and bones.
Lead is measured in blood to understand
exposure.
 Lead in bone is released into blood during
pregnancy and becomes a source of exposure
to the developing foetus.
 It affects emissions control systems of vehicles
particularly catalytic converters.
 India started its phase down in 1994 and
completed in 2000.
9. c
Valmiki Tiger Reserve

www.shankariasacademy.com | www.iasparliament.com

You might also like